Sunteți pe pagina 1din 377

3rd Asian Physics Olympiad

Singapore
Experimental Competition PART B
(The Experimental Competition consists of two parts, Part A and Part B. The
following experiment represents Part B only)

Date: May 10, 2002


Time Available: 2_ hours for Part B
Read the following instructions carefully
1. In this experiment, you are not expected to indicate uncertainties of your
experimental results.
2. Use only the pen provided.
3. Use only the front side of the answer sheets and graph papers.
4. In your answers please use as little text as possible; express yourself
primarily in equations, numbers and figures. If the required result is a
numerical number, underline your final result with a wavy line.
5. Write on the blank sheets of paper the results of all your measurements
and whatever else you consider is required for the solution of the question
and that you wish to be marked.
6. It is absolutely essential that you enter in the boxes at the top of each
sheet of paper used, including graph paper, your Name, your Country,
your student number (Student No.), the part number of the question
(Question Part No.), the progressive number of each sheet (including
graph sheets) (Page No.) and the total number of sheets that you have
used and wish to be marked for each question (Total pages). If you use
some sheets of paper for notes that you do not wish to be marked, put a
large cross through the whole sheet and do not include them in your
numbering. Do not write anything on the right column which is reserved
for Examiners use only.
7. At the end of 2_ hours for this half of the examination, please staple your
answer and graph sheets in the order of their page numbers, before
proceeding to the other half of the examination.

Objective
To

study how the frequency of vibration of a tuning fork varies with an equal
mass clamped on each of its prongs (at a definite point near the prong tip), and
hence to determine the pair of unknown masses X similarly attached to the
prongs.
The Stroboscope
The experiment will make use of a stroboscope (strobe) which is a simple
electronic device consisting of a discharge lamp which can be made to flash for
a short duration with a high intensity at highly regular intervals. The strobe
enables the frequency of a rotating or vibrating object to be measured without
the need for any direct physical contact with the moving object.
Caution: The strobe has a finite life time, specified in maximum number of
flashes obtainable. Do not leave it running idly when you are not using it.
Consider a particle rotating with uniform circular motion being illuminated by a
strobe. If the flash frequency is a multiple or sub-multiple of that of the motion,
the particle will appear stationary. It follows that the periodicity of the circular
motion of the particle can be determined by tuning the frequency of the light
flash.
Suppose the frequency of rotation of the particle is x Hz, and that of flashing is y
Hz. Then, in the time interval of 1/y s between two successive flashes the
particle would have moved through an angle 2x/y.
If y/x is an irrational number so that it cannot be expressed as a ratio of two
integers, then the particle would not appear stationary but would appear to rotate
slowly in the forward or backward direction depending on whether y/x is just
slightly smaller or larger than some rational number nearby.
If y/x = q/p where p and q are integers, then the strobe would flash q times for
every p complete cycles. Furthermore, if p and q have no common factors
between them (assumed throughout this write-up), then each flash would show a
different position of the particle. Thus the particle will exhibit q stationary
positions under the strobe flashlight.
If q becomes too large it might be difficult to count the number of stationary
positions displayed by the rotating particle.
The above theory applied to the rotating particle can be similarly applied to that
of a tuning fork vibrating in simple harmonic motion if we regard the
vibrational motion as equivalent to the motion of the projection of the rotating

case, because the vibrating object retraces the same path in the opposite
direction every half cycle, there is a chance, though very remote, that an image
in one half of a vibration cycle coincides with that in the next half cycle. It
would result in only one image (but of double the intensity) being recorded,
instead of two. This freak coincidence should be guarded against in an
experimental observation.
Identification of Fundamental Synchronism
Fundamental synchronism is obtained when the lamp flashes once for every
cycle of rotation or vibration of the mechanism under observation, so that the
object appears to stop at one stationary position. However, it will be
appreciated that a similar and indistinguishable result will also occur when the
flash frequency is a sub-multiple (1/2, 1/3, 1/4, etc) of the object movement
frequency. Thus if the object movement frequency is totally unknown, when
adjusting for fundamental synchronism, a safe procedure is to start at a high
flash frequency, when multiple images are obtained, and then slowly reduce the
flash frequency until the first single image appears. This procedure should be
adopted in all measurements to check for fundamental synchronism.
Multiples of Fundamental Frequency
Multiples of fundamental frequency occur when the strobe is flashing at a
higher rate than the cyclical frequency of the object under observation. The
converse when the strobe flashing rate is lower than that of the moving object is
referred to as sub-multiples of fundamental frequency.
If the lamp is flashed at a frequency q times the rotational frequency of the
particle, multiple images can be seen. In such a situation, a rotating particle will
appear as several stationary images spaced equally around the circumference.
Twice this frequency, or q/p=2, will produce two such images at radians apart,
and three times this frequency, or q/p=3, will yield three images at 2/3 radians
spacing, etc. The particle rotational frequency is then given by the flash
frequency divided by the number of images seen. In general, if q>p>1, then the
strobe would flash q times for every p cycles of the particle motion, and so there
will still be q stationary positions.

Sub-multiples of Fundamental Frequency


Here, q/p is less than one. If the strobe frequency is exactly 1/p times that of the
object movement, where p>1, then the object would have moved through p
cycles for every flash, and only one stationary image is seen. If p>q>1, then the

strobe would have flashed q times for every p full cycles of the object
movement, and the number of stationary images seen would be q.
The Tuning Fork
A tuning fork is designed to vibrate at a fundamental frequency with no
harmonics after it is struck. The two prongs of the fork are symmetrical in
every respect so that they move in perfect anti-phase and exert, at any instant,
equal and opposite forces on the central holder. The net force on the holder is
therefore always zero so that the holder does not vibrate, and hence holding it
firmly will not cause any undesirable damping. For the same reason the prongs
of a tuning fork cannot vibrate in like phase as this will result in a finite
oscillatory force on its holder which would cause the vibration to dampen away
very quickly.
It is possible to lower the fundamental frequency of the tuning fork by loading
an equal weight on each arm. The loading on the arms has to be symmetrical in
order to minimise damping of vibration.
For such a loaded tuning fork, the period T of vibration is given by:
2

T = A(m + B)
where A is a constant depending on the size, shape and mechanical properties of
the tuning fork material and B a constant depending on the effective mass of
each vibrating arm.
Items of Apparatus provided:
1. A stroboscope with digital readout.
2. A mini-torch light.
3. A tuning fork with a 31.6g weight loaded symmetrically on each prong
and with the centre-of-mass of the weight coinciding with the point P
marked clearly on each prong.
4. Two paper clamps with two detachable levers. The levers are used only
to open the clamps, and they should be removed when you are doing
the experiment.
5. A pair of equal unknown masses X.
6. A series of the following known masses (in pairs): 5g, 10g, 15g, 20g,
25g.
7. Regular graph papers (5 sheets).

Experimental Steps
Step 1: Fundamental synchronism and measurements of multiple frequencies
(2.7 marks)
(a) Obtain fundamental synchronism between the strobe flash and the
vibrating tuning fork loaded with the original 31.6g mass on each
prong. By dislodging the mass temporarily, check to make sure that
the mass is pre-clamped with its centre-of-mass located at the point P
(which is marked on the prong but hidden by the mass). Record its
fundamental flash frequency.
(b) Keeping the flash frequency above the fundamental frequency, try to
discover as many readings of flash frequencies as possible which
yield observable stationary images of the (31.6g-loaded) tuning fork
frequency. Identify their different q/p values.
(c) Tabulate your data (in the order of increasing q/p) as follow,
keeping q/p as a rational fraction:
Strobe Reading

Number of Stationary Images

q/p value

Plot a straight-line graph of all the observed strobe flash frequencies


against the corresponding multiple of the tuning fork frequency.
Identify each data point on the graph with its q/p value.

Step 2:

Measurements of sub-multiple frequencies (2.3 marks)

(a) Keeping the strobe frequency below the fundamental frequency of


the (31.6g-loaded) tuning fork, obtain readings of all observable
strobe frequencies which yield stationary images.
(b) Tabulate your readings as in question 1, but in the order of
decreasing q/p, and plot a straight-line graph of all the observed
strobe frequencies against the corresponding sub-multiple of the
(31.6g-loaded) tuning fork frequency. Identify each data point on
the graph with its q/p value.

Step 3: Determination of the pair of unknown masses X (5 marks)


(a)

Remove the 31.6g loading mass from each prong (which would
also reveal the point P marked on the prong) and obtain the
resulting vibrational frequency of the unloaded tuning fork.

(b)

Next, obtain the vibrational frequencies of the tuning fork with


each prong loaded with known masses m of 5g, 10g, 15g, 20g and
25g respectively. Ensure that in each case the centre-of-mass of the
load coincides with the point P. Note that the value of m as
labelled on the mass is the total mass of both the mass itself and
that of the given paper clamp (with both its levers removed) used to
clamp it.

(c)

Tabulate your results using your data obtained in (b) and plot a
2
graph of T against m. Obtain the slope, and the intercept on the
m-axis.

(d)

Replace the known loading masses with the unknown masses X,


and obtain the vibrational frequency under this loading. Deduce X.
Again, note that X also includes the mass of the paper clamp (with
both its levers removed).

IPhO2002

II. OPTICAL BLACK BOX


Description
In this problem, the students have to identify the unknown optical components inside the
cubic box. The box is sealed and has only two narrow openings protected by red plastic
covering. The components should be identified by means of optical phenomena observed in
the experiment. Ignore the small thickness effect of the plastic covering layer.
A line going through the centers of the slits is defined as the axis of the box. Apart from the
red plastic coverings, there are three (might be identical or different) elements from the
following list:

Mirror, either plane or spherical

Lens, either positive or negative

Transparent plate having parallel flat surfaces (so called plane-parallel plate)

Prism

Diffraction grating.
The transparent components are made of material with a refractive index of 1.47 at the
wavelength used.

Apparatus available:

A laser pointer with a wavelength of 670 nm. CAUTION: DO NOT LOOK


DIRECTLY INTO THE LASER BEAM.
An optical rail
A platform for the cube, movable along the optical rail
A screen which can be attached to the end of the rail, and detached from it for other
measurements.
A sheet of graph paper which can be pasted on the screen by cellotape.
A vertical stand equipped with a universal clamp and a test tube with arbitrary scales,
which are also used in the Problem I.
Note that all scales marked on the graph papers and the apparatus for the experiments are
of the same scale unit, but not calibrated in millimeter.

IPhO2002

The Problem
Identify

each

of

the

three

components

and

give

its

respective

specification:

Possible type of component

Specification required

mirror

radius of curvature, angle between the mirror axis and


the axis of the box

lens*

positive or negative,
box

plane-parallel plate

its focal le ngth, and its position inside the

thickness, the angle between the plate and the axis of the box

prism

apex angle, the angle between one of its deflecting sides and
the axis of the box

diffraction grating*

line spacing, direction of the lines, and its position inside the
box

implies that its plane is at right angle to the axis of the box

Express your final answers for the specification parameters of each component (e.g. focal
length, radius of curvature) in terms of millimeter, micrometer or the scale of graph paper.
You dont have to determine the accuracy of the results.

IPhO2002
Country

Student No. Experiment No. Page No. Total Pages

ANSWER FORM
1.

2.

Write down the types of the optical components inside the box :
no.1.

[0.5 pts]

no.2..

[0.5 pts]

no.3.

[0.5 pts]

The cross section of the box is given in the figure below. Add a sketch in the figure
to show how the three components are positioned inside the box. In your sketch,
denote each component with its code number in answer 1 .
[0.5 pts for each correct position]

axis of the box

direction of the slit

direction of the slit

IPhO2002

Country

Student No. Experiment No. Page No. Total Pages

3. Add detailed information with additional sketches regarding arrangement of the optical
components in answer 2, such as the angle, the distance of the component from the slit, and
the orientation or direction of the components. [1.0 pts]

IPhO2002

Country

Student No. Experiment No. Page No. Total Pages

4. Summarize the observed data [0.5 pts], determine the specification of the optical
component no.1 by deriving the appropriate formula with the help of drawing [1.0 pts],
calculate the specifications in question and enter your answer in the box below [0.5 pts].

Name of component no.1

Specification

IPhO2002

Country

Student No. Experiment No.


No. Page No. Total Pages

5. Summarize the observed data [0.5 pts], determine the specification of the optical
component no.2 by deriving the appropriate formula with the help of drawing [1.0 pts],
calculate the specifications in question and enter your answer in the box below [0.5 pts].

Name of component no.2

Specification

10

IPhO2002

Country

Student No. Experiment No. Page No. Total Pages

6. Summarize the observed data [0.5 pts], determine the specification of the optical
component no.3 by deriving the appropriate formula with the help of drawing [1.0 pts],
calculate the specifications in question and enter your answer in the box below [0.5 pts].

Name of component no.3

Specification

11

4th Asian Physics Olympiad

Experimental Competition

25 April 2003

Experimental Competition
II. Cylindrical Bore
Background
There are many techniques to study the object with a bore inside. Mechanical
oscillation method is one of the non-destructive techniques. In this problem, you are
given a brass cube of uniform density with cylindrical bore inside. You are required to
perform non-destructive mechanical measurements and use these data to plot the
appropriate graph to find the ratio of the radius of the bore to the side of the cube.
The cube of sides a has a cylindrical bore of radius b along the axis of
symmetry as shown in Fig. 2.1. This bore is covered by very thin discs of the same
material. A, B and C represent small holes at the corners of the cube. These holes can be
used for suspending the cube in two configurations. Fig. 2.2(a) shows the suspension
using B and C; the other suspension is by using A and B as shown in Fig 2.2(b).

B
C

2b

a
Fig. 2.1 Geometry of cube with cylindrical bore

4th Asian Physics Olympiad

Experimental Competition

25 April 2003

Top of stand
I2

I1

B
C

C
2.2(a)

2.2(b)

Fig. 2.2 Two configurations of cubes suspension

Students may use the following in their derivation of necessary formulae:


Y

For a solid cube of side a


1
I = Ma 2 about both axes
6

c.m.
c.m. = centre of mass
Y

For a solid cylinder of radius b length a


1
IY =
mb 2
2
1
1
IX =
ma 2 + mb 2
12
4

4th Asian Physics Olympiad

Experimental Competition

25 April 2003

Materials and apparatus


1.
2.
3.
4.
5.
6.

brass cube
stop watch
stand
thread
ruler/ centimeter stick
linear graph papers

Experiment
a)

Choose only one of the two bifilar suspensions as shown in Fig. 2.2, and derive the
expressions for the moment of inertia and the period of oscillation about the vertical
axis through the centre of mass in terms of A, d , b, a and g where A is the length of
each thread and d is the separation between threads.
(2 points)

b) Perform necessary non-destructive mechanical measurements and use these data to


b
plot an appropriate graph and then find the value of .
(8 points)
a
The value of g for Bangkok = 9.78 m/s2.
---------------------------------

Experimental Competition - Problem No. 2

Black box
APPARATUS AND MATERIALS
1. A double beam oscilloscope.
2. A function generator capable to generate sine, triangle and square waves over the
0.02 Hz to 2 MHz range.
3. A "Black box" with two groups of connectors: the ABCD group and A'B'C'D' group.
Besides, there are also two connectors for the standard resistor Rn = 5 k, which is
isolated from the two groups.
4. Conductors of negligible resistance.
5. Graph paper.
Warning: You are not allowed to open the black box.
EXPERIMENT
In the black box, there are two groups of passive
elements (that are elements of the types: resistor R,
capacitor C or inductor (induction coil) L). The first
group consists of three elements Z1 , Z 2 , Z 3
connected in a star circuit as shown in Figure 1. The B
elements are led out to the connectors A, B, C and
D, with A - the common connector of the ABCD
group. The second group consists of three elements
Z'1 , Z'2 , Z'3 connected in the same manner to
connectors A', B', C' and D', with A'- the common
connector of the A'B'C'D' group (see Figure 2).
1. By using the oscilloscope and the function
generator, determine the type and the parameter (that
is resistance of R, capacity of C, inductivity of L) of
each of the elements Z1 , Z 2 , Z 3 and Z'1 , Z'2 , Z'3 .
[5.0 pts]

C
Z2
D
Z1

Z3
A
Figure 1

C
Z2
B

2. Connect five points B, C, B', C' and D' together.


We obtain a new black box with terminals DDA
(called DDA).
a. Draw the electric circuit of this black box.
b. Apply a sine wave from the generator to
connectors D and A'.
Plot a graph of the ratio of the voltage amplitudes K =

D
Z1

Z3

A
Figure 2

U D'A '
and the phase shift
U DA '

between these voltages as functions of the frequency f of the signal.


c. The graphs possess a particular point at a certain frequency f0. Determine the value
of the frequency f0, the ratio K =

U D'A '
and the phase shift at this frequency.
U DA '

d. Derive the relation between f0 and the parameters of the elements in the black box
and calculate the values of f0.. [5.0 pts]

Experimental Competition

Before attempting to assemble


your apparatus, read the problem text completely!
Please read this first:
1. The time available for the Experimental problem 1 is 2 hours and 45 minutes;
and that for the Experimental problem 2 is 2 hours and 15 minutes.
2. Use only the pen and equipments provided.
3. Use only the one side of the provided sheets of paper.
4. In addition to blank sheets where you may write freely, there is a set of Answer
sheets where you must summarize the results you have obtained. Numerical
results must be written with as many digits as appropriate; do not forget the units.
5. Please write on the blank sheets the results of all your measurements and
whatever else you deem important for the solution of the problem that you wish to
be evaluated during the marking process. However, you should use mainly
equations, numbers, symbols, graphs, figures, and as little text as possible.
6. It is absolutely imperative that you write on top of each sheet: your student code
as shown on your identification tag, and additionally on the blank sheets: your
student code, the progressive number of each sheet (Page n. from 1 to N) and the
total number (N) of blank sheets that you use and wish to be evaluated (Page
total).
7. The student should start with a new page for each section. It is also useful to write
the number of the section you are answering at the beginning of each such section.
If you use some sheets for notes that you do not wish to be evaluated by the
marking team, just put a large cross through the whole sheet and do not number it.
8. When you have finished, turn in all sheets in proper order (answer sheet first, then
used sheets in order, the unused sheets and problem text at the bottom) and put
them all inside the envelope provided; then leave everything on your desk. You
are not allowed to take anything out of the room.
9. are not allowed to take anything out of the room.

Note: The AC voltmeter black port is connected with two spare terminals

Experimental problem 2
Measurement of liquid electric conductivity10 points

1. Experimental instructions
In the apparatus of present experiment to measure the conductivity of liquid (i.e.,
water with salt), the sensor deals with ac signal without any contact potential involved
to interfere with the desired experimental results. Meanwhile, since the sensor
(detective windingdoes not directly touch the liquid to be measured, no chemical
reaction would happen during the experiments to damage any part of the apparatus.
Therefore it can be used repeatedly for a long time.
As shown in Fig. 1, the sensor designed for measuring the conductivity of liquid
consists of two circular loops with the same radius, made of soft-iron-based alloy.
Each loop is wound with winding. The numbers of circles of the two windings are
equal to each other. The two alloy loops are aligned along the same axis and
connected closely as one airproof hollow cylinder, as shown in Fig. 2.

Fig. 1

Fig. 2

The sensor is immersed in the liquid to be measured. Winding 11 is connected to


sine signal generator of frequency about 2.5kHz. The amplitude of its output signal
might drift somewhat. If the drift exceeds certain value, it should be adjusted in time
to keep the output amplitude remain at certain value. Winding 22 is connected to ac
voltmeter used to measure the induced signal voltage. With the measured magnitude
of the signal voltage, the conductivity of the liquid can be calculated.

2. Experimental principles
The operation principle of the present experimental apparatus can be simply
explained as follows. The ac sine current from the signal generator induces an ac
magnetic field in loop 11. In turn the magnetic field induces an ac current in the

conducting liquid. Such induced current induces back a time-varying magnetic field in
loop 22, which induces an electromotive force in the same loop 22, being the output
signal of the sensor.
Neglecting the magnetic hysteresis effect, output voltage
function of input voltage

. When input voltage

is a monotonical

and the conductivity

of the

liquid are respectively within certain range, a proportional relation holds between
and the ratio of

:
=

where

(1)

is the proportionality constant.

In the present apparatus, the liquid container can contain so much liquid to be
measured that the resistance of the liquid outside the cylinder-shaped sensor is
negligible. Therefore the output voltage

of the sensor depends mainly on the

liquid within the hollow cylinder (referred as liquid cylinder hereafter). Thus, it
is possible to use the liquid cylinder to calculate the liquid conductivity. Resistance of
the liquid cylinder is
(2)
where L is the length of the liquid cylinder along its axis, and S is the area of its cross
section. Combination of (1) and (2) leads to
(3)
where

or alternatively

With Eq.(2) and (3) we obtain


.

(4)

Equation (4) shows that, when using the present sensor to measure the liquid
conductivity,

is related to L (length of the hollow cylinder), S (area of its cross section),

, and B as well.
Remark Essentially in the present experiment, in order to obtain the
proportionality constant K and then B accurately, various kinds of liquid with known

should be required and prepared. Obviously this is not an easy task. Therefore, for
the sake of both convenience and correctness, instead of the various liquids of known
, we use externally connected standard resistors. The two ends of the standard
resistor are connected to the two ends of a conducting thread passing through the
hollow cylinder of the sensor to form a resistor circuit, as shown in Fig. 3.

Fig. 3
3. Experimental content
1.

Draw the experimental circuit diagram for scaling the sensor of the liquid

conductivity (1.0 points) , and complete the connection of the circuit in order to
measure both the input voltage

(<2.000V) and induced output voltage

according to the above circuit diagram (1.0 points).


2.

According to the range of resistance of standard resistors: 09.9

, measure

for various resistances. Record the data in the data Table designed by
yourself.

(2.0points)

Control the amplitude of

at any moment to make sure that its effective value

is within the range of [1.700V, 1.990V] and its variation should not be higher
than 0.03V. You can also fix the input voltage at a single value within this range.
(1.0 points)
3-1. Take

as ordinate and the reciprocal of resistance R of the standard resistor

1/R as abscissa. Draw the curve of

versus 1/R. The number of

measurement points should be greater than 20 within the whole output voltage
range, and you are not required to add error (uncertainty) bars to the graph, but
should estimate the uncertainties from the scatter points.

(1.0 points)

3-2 It can be seen that at some region of less induced current the curve is linear.
Graph this linear part and use the graphical method to obtain the slope B of the
straight part of the curve and its relative uncertainty u(B) or u(B)/B.

(1.5 points)

4. With the given axis length of the sensor L=(30.5000.025) mm and diameter of the
liquid cylinder d=(13.9000.025)mm, calculate the value of
u(K)/K.

and u(K) or

(1.0 points)

5Work out the conductivity of the liquid in the container and write the result.
According to the uncertainties of L, d, and B, estimate the uncertainty of the
conductivity. The measurement of the conductivity should be done for six times,
during

which

the

liquid

should

be

stirred

for

each

time.

(1.5 points)
4. Instruments and materials
1.

Sensor of liquid conductivity


The sensor has four ports of connection terminals: two terminal ports are
connected to winding 11 and two terminal ports are connected to the other
winding 22.

2.

Container filled with the yet-to-be-determined liquid and stirring rod.

3.

The instrument for measuring the liquid conductivity.


On the instrument panel there are:

Signal generator:
Two ports of connection terminals connected to the signal generator, the
red one for signal output, and the black one for grounding. The amplitude
of output signal can be adjusted by turning the knob.

ac digital voltmeter.

Inserting-type resistor box:


On the panel, there are many ports of connection terminals, between every
two adjacent ports, there is a resistor with relative resistance error of 0.001.
Resistance of these resistors is 0.1,0.2, 0.5,1,2, and 5 respectively.

Switch 1 2single-pole double throw.

4 Some leads
5.

Two pieces of graph paper20cm25cmcalculator, recording paper, ruler, and


pen.

EXPERIMENTAL PROBLEM 2
BIREFRINGENCE OF MICA
In this experiment you will measure the birefringence of mica (a crystal widely used in
polarizing optical components).

MATERIAL
In addition to items 1), 2) and 3), you should use,
14) Two polarizing films mounted in slide holders, each with an additional acrylic
support (LABEL J). See photograph for mounting instructions.
15) A thin mica plate mounted in a plastic cylinder with a scale with no numbers;
acrylic support for the cylinder (LABEL K). See photograph for mounting
instructions.
16) Photodetector equipment. A photodetector in a plastic box, connectors and foam
support. A multimeter to measure the voltage of the photodetector (LABEL L).
See photograph for mounting and connecting instructions.
17) Calculator.
18) White index cards, masking tape, stickers, scissors, triangle squares set.
19) Pencils, paper, graph paper.

Polarizer mounted in slide holder with


acrylic support (LABEL J).

Thin mica plate mounted in cylinder with


a scale with no numbers, and acrylic
support (LABEL K).

A photodetector in a plastic box, connectors and foam support. A multimeter to measure the
voltage of the photodetector (LABEL L). Set the connections as indicated.

DESCRIPTION OF THE PHENOMENON


Light is a transverse electromagnetic wave, with its electric field lying on a plane
perpendicular to the propagation direction and oscillating in time as the light wave travels.
If the direction of the electric field remains in time oscillating along a single line, the wave
is said to be linearly polarized, or simply, polarized. See Figure 2.1.

Figure 2.1 A wave travelling in the y-direction and polarized in the z-direction.

A polarizing film (or simply, a polarizer) is a material with a privileged axis parallel to its
surface, such that, transmitted light emerges polarized along the axis of the polarizer. Call
(+) the privileged axis and (-) the perpendicular one.

Figure 2.2 Unpolarized light normally incident on a polarizer. Transmitted light is polarized
in the (+) direction of the polarizer.
Common transparent materials (such as window glass), transmit light with the same
polarization as the incident one, because its index of refraction does not depend on the
direction and/or polarization of the incident wave. Many crystals, including mica, however,
are sensitive to the direction of the electric field of the wave. For propagation perpendicular
to its surface, the mica sheet has two characteristic orthogonal axes, which we will call Axis
1 and Axis 2. This leads to the phenomenon called birefringence.

Figure 2.3 Thin slab of mica with its two axes, Axis 1 (red) and Axis 2 (green).
Let us analyze two simple cases to exemplify the birefringence. Assume that a wave
polarized in the vertical direction is normally incident on one of the surfaces of the thin
slab of mica.

Case 1) Axis 1 or Axis 2 is parallel to the polarization of the incident wave. The trasmitted
wave passes without changing its polarization state, but the propagation is characterized as if
the material had either an index of refraction n1 or n 2 . See Figs. 2.4 and 2.5.

Figure 2.4 Axis 1 is parallel to polarization of incident wave. Index of refraction is n1.

Figure 2.5 Axis 2 is parallel to polarization of incident wave. Index of refraction is n 2 .


Case 2) Axis 1 makes an angle with the direction of polarization of the incident wave.
The transmitted light has a more complicated polarization state. This wave, however, can be
seen as the superposition of two waves with different phases, one that has polarization
parallel to the polarization of the incident wave (i.e. "vertical") and another that has
polarization perpendicular to the polarization of the incident wave (i.e. "horizontal").

Figure 2.6 Axis 1 makes and angle with polarization of incident wave
Call IP the intensity of the wave transmitted parallel to the polarization of the incident
wave, and IO the intensity of the wave transmitted perpendicular to polarization of the
incident wave. These intensities depend on the angle , on the wavelength of the light
source, on the thickness L of the thin plate, and on the absolute value of the difference of
the refractive indices, n1 n 2 . This last quantity is called the birefringence of the material.
The measurement of this quantity is the goal of this problem. Together with polarizers,
birefringent materials are useful for the control of light polarization states.
We point out here that the photodetector measures the intensity of the light incident on it,
independent of its polarization.
The dependence of IP ( ) and IO ( ) on the angle is complicated due to other effects not
considered, such as the absorption of the incident radiation by the mica. One can obtain,
however, approximated but very simple expressions for the normalized intensities IP ( ) and
IO ( ) , defined as,

IP ( ) =

IP ( )
IP ( ) + IO ( )

(2.1)

and

IO ( )
(2.2)
IP ( ) + IO ( )
It can be shown that the normalized intensities are (approximately) given by,
IO ( ) =

IP ( ) = 1
and

1
(1 cos )sin 2 (2 )
2

(2.3)

IO ( ) =

1
(1 cos )sin2 (2 )
2

(2.4)

where is the difference of phases of the parallel and perpendicular transmitted waves.
This quantity is given by,

2L

n1 n 2

(2.5)

where L is the thickness of the thin plate of mica, the wavelength of the incident
radiation and n1 n 2 the birefringence.

EXPERIMENTAL SETUP
Task 2.1 Experimental setup for measuring intensities. Design an experimental setup for
measuring the intensities IP and IO of the transmitted wave, as a function of the angle of
any of the optical axes, as shown in Fig. 2.6. Do this by writing the LABELS of the different
devices on the drawing of the optical table. Use the convention (+) and (-) for the direction
of the polarizers. You can make additional simple drawings to help clarify your design.
Task 2.1 a) Setup for IP (0.5 points).
Task 2.1 b) Setup for IO (0.5 points).
Laser beam alignment. Align the laser beam in such a way that it is parallel to the table and
is incident on the center of the cylinder holding the mica. You may align by using one the
white index cards to follow the path. Small adjustments can be made with the movable
mirror.
Photodetector and the multimeter. The photodetector produces a voltage as light impinges
on it. Measure this voltage with the multimeter provided. The voltage produced is linearly
proportional to the intensity of the light. Thus, report the intensities as the voltage produced
by the photodetector. Without any laser beam incident on the photodetector, you can
measure the background light intensity of the detector. This should be less than 1 mV. Do
not correct for this background when you perform the intensity measurements.
WARNING: The laser beam is partially polarized but it is not known in which direction.
Thus, to obtain polarized light with good intensity readings, place a polarizer with either its
(+) or () axes vertically in such a way that you obtain the maximum transmitted intensity in
the absence of any other optical device.

MEASURING INTENSITIES
Task 2.2 The scale for angle settings. The cylinder holding the mica has a regular
graduation for settings of the angles. Write down the value in degrees of the smallest interval
(i.e. between two black consecutive lines). (0.25 points).
Finding (approximately) the zero of and/or the location of the mica axes. To facilitate
the analysis, it is very important that you find the appropriate zero of the angles. We suggest
that, first, you identify the location of one of the mica axes, and call it Axis 1. It is almost
sure that this position will not coincide with a graduation line on the cylinder. Thus,
consider the nearest graduation line in the mica cylinder as the provisional origin for the
angles. Call the angles measured from such an origin. Below you will be asked to provide
a more accurate location of the zero of .
Task 2.3 Measuring IP and IO . Measure the intensities IP and IO for as many angles as
you consider necessary. Report your measurements in Table I. Try to make the
measurements for IP and IO for the same setting of the cylinder with the mica, that is, for a
fixed angle . (3.0 points).
Task 2.4 Finding an appropriate zero for . The location of Axis 1 defines the zero of the
angle . As mentioned above, it is mostly sure that the location of Axis 1 does not coincide
with a graduation line on the mica cylinder. To find the zero of the angles, you may proceed
either graphically or numerically. Recognize that the relationship near a maximum or a
minimum may be approximated by a parabola where:
I( ) a 2 + b + c
and the minimum or maximum of the parabola is given by,
b
m = .
2a
Either of the above choices gives rise to a shift of all your values of given in
Table I of Task 2.3, such that they can now be written as angles from the appropriate zero,
= + . Write down the value of the shift in degrees. (1.0 points).
DATA ANALYSIS.
Task 2.5 Choosing the appropriate variables. Choose IP ( ) or IO ( ) to make an analysis
to find the difference of phases . Identify the variables that you will use. (0.5 point).

Task 2.6 Data analysis and the phase difference.

Use Table II to write down the values of the variables needed for their analysis.
Make sure that you use the corrected values for the angles . Include uncertainties.
Use graph paper to plot your variables. (1.0 points).

Perform an analysis of the data needed to obtain the phase difference . Report
your results including uncertainties. Write down any equations or formulas used in
the analysis. Plot your results. (1.75 points).

Calculate the value of the phase difference in radians, including its uncertainty.
Find the value of the phase difference in the interval [0, ]. (0.5 points).

Task 2.7 Calculating the birefringence n1 n 2 . You may note that if you add 2 N to the
phase difference , with N any integer, or if you change the sign of the phase, the values
of the intensities are unchanged. However, the value of the birefringence n1 n 2 would
change. Thus, to use the value found in Task 2.6 to correctly calculate the birefringence,
you must consider the following:

2 L

or

2 =

n1 n 2
2L

n1 n 2

if

L < 82 106 m

if

L > 82 106 m

where the value L of the thickness of the slab of mica you used is written on the cylinder
holding it. This number is given in micrometers (1 micrometer = 10-6 m). Assign 1 106 m
as the uncertainty for L . For the laser wavelength, you may use the value you found in
Problem 1 or the average value between 620 109 m and 750 109 m, the reported range
for red in the visible spectrum. Write down the values of L and as well as the
birefringence n1 n 2 with its uncertainty. Include the formulas that you used to calculate
the uncertainties. (1.0 points).

Question Number

Experimental Competition
27 April 2010

Page 1 of 8

__________________________________________________________________________________________

Experiment II

Understanding Semiconductor Lasers

The purpose of this experiment is to explore the basic characteristics of semiconductor


lasers. We will measure and calculate the fraction of the linear polarization of the
collimated laser beam by using a pair of polarizers and a photoconductor. Finally, we will
determine the maximum value of the power increase per current increment of the
collimated laser.

Safety Caution: Do not look directly into the laser beam, which can damage your eyes!!

Background Description

The photoconductor, the light-sensing device in this experiment, is made of


semiconductor, which has a band gap of EG = (EC EV) (see Fig. II-1). When the energy of
the incident photons is larger than that of the band gap, the photons can be absorbed by the
semiconductor to create free electrons and holes. The density of charge carriers, including
electrons and holes, is then increased, and so is the conductivity of the material. In this
experiment, the resistance (the inverse of conductance) is measured by using a multimeter.

Electron
EC (Conduction Band Edge)
Photon

EG (Energy Gap)
EV (Valence Band Edge)
Hole

Fig. II-1 Schematic diagram of an electron-hole pair generated by a


single photon in a semiconductor.

Question Number

Experimental Competition
27 April 2010

Page 2 of 8

__________________________________________________________________________________________

In the semiconductor laser, the light emitting device in this experiment, as the external
source injects electrons and holes into the device, they can combine to emit photons as
shown schematically in the Fig. II-2. Ideally, the combination of one pair of electron and
hole can generate one photon. Realistically, there are also nonradiative processes through
which an electron-hole pair recombines without generating a photon. Thus the number of
photons generated is not equal to the number of electron-hole pairs recombined. The
average fractional number of photons generated by an electron-hole pair is called the
quantum efficiency.

Electron
EC (Conduction Band Edge)
Photon

EG (Energy Gap)
EV (Valence Band Edge)
Hole

Fig. II-2 Schematic diagram of a single photon generated by


an electron-hole pair combined in a semiconductor.

The semiconductor laser can emit a monochromatic, partially polarized and coherent
light beam. The partially polarized light is composed of two parts linearly polarized and
unpolarized. The light intensity due to the former is denoted by Jp and the other by Ju.
When the partially polarized light is incident upon a polarizer, the transmittance of the
linearly polarized part depends on the angle between its polarized direction and the
direction of the polarizer. But for the unpolarized part, a constant portion is allowed to pass
through the polarizer and is independent of the angle.

Question Number

Experimental Competition
27 April 2010

Page 3 of 8

__________________________________________________________________________________________

Experiments and Procedures

Exp. II-ALight Response of the Photoconductor


The light source used in this part should be the collimated laser diode (CLD). Use the
circuit diagram shown in Fig. IIA-1 to provide current for the CLD. The symbols used in Fig.
IIA-1 are listed in Table IIA-1.

Fig. IIA-1 Circuit diagram used for the CLD.

Table IIA-1 Symbols used in Fig. IIA-1


Devices Collimated Laser
5V
Diode
dc-power

Variable
Resistors

Symbols

Label

Ammeter
(Multimeter)
A

II-L-#

C-D-#

C-E-#

II-X, II-Y

Operate the CLD with the maximum current. The laser intensity is detected by a
photoconductor (PC). When you shine light on a PC, the conductance increases with the light
intensity. You should minimize the ambient light effect. In this experiment, we actually
measure the resistance, which is the inverse of conductance. The intensity of the laser light
reaching the PC may be varied by using the supplied polarizers or filters. The symbols of
other optical components are given in the Table IIA-2. The partial polarization of the laser
light may be observed by using the experimental setup in the Fig. IIA-2.

Question Number

Experimental Competition
27 April 2010

Page 4 of 8

__________________________________________________________________________________________

Table IIA-2 Symbols of optical components.


Devices
Collimated Laser
Polarizers
Diode
Symbols

Label

P1

C-C-#

Fig.IIA-2

P2

II-P-#
II-Q-#

Photoconductor

Light filter

PC

II-W-#

Optional
II-U

P1

PC

Experimental setup for the preparation

Rotating P1, one should observe that the PC resistance varies. Adjust P1 so that the PC
resistance reaches a minimum. If the observed minimum resistance happens in a range, say
10 or larger of the rotation angle of P1, then the PC is saturated. In this case light filter(s)
should be used to avoid PC saturation near the maximum light intensity.
Fix the P1 position according to the description in previous paragraph. Characterize the
conductance of the PC versus the relative light intensity following the experimental setup
shown in the Fig. IIA-3.

Question Number

Experimental Competition
27 April 2010

Page 5 of 8

__________________________________________________________________________________________

P1

P2 PC

P1
CLD

P2
PC

Fig.IIA-3

Experimental setup for the PC characterization

(1) Define P to be the relative angle between polarization axes of P1 and P2. By varying
the angle P from 0 to 180 in step of 5 . Record the measured PC resistance and
P in the data table. Transform the measured PC resistance values into conductance values
and record them in the data table. No error analysis is required.

(1.2 points)

(2) Plot the PC conductance values as a function of P on a graph paper. No error analysis is
required.

(1.2 points)

Question Number

Experimental Competition
27 April 2010

Page 6 of 8

__________________________________________________________________________________________

Exp. II-BThe Fraction of Linearly Polarized Laser Light

The light source used in this part should be the collimated laser diode (CLD) with a 15
mA current from the dc power supply. The task in this part is to determine the fraction of the
laser light that is linearly polarized by using the setup in Fig. IIA-2, which is the same as the
previous section. No error analysis is required in this part.

J ( Linearly Polarized )
J J min
= max
J (Unpolarized ) + J ( Linearly Polarized ) J max + J min

Jmax and Jmin are the maximum and minimum light intensity detected by PC while rotating P1.

Fig.IIA-2

P1

PC

Experimental setup for the preparation

(1) Find the maximum and minimum values of PC resistance (Rmax and Rmin) by rotating P1
360 . Transform Rmax and Rmin into the minimum and maximum values of PC conductance
Cmin and Cmax. Record the data in the data table.
(0.8 points)
(2) Utilizing the conductance versus P graph in Exp. II-A-(2) to determine the relative
intensities J m a x and J m i n corresponding to Cmax and Cmin. Write down the result on the
answer sheet.
(3) Calculate and write down the result on the answer sheet.

(1.6 points)
(0.2 points)

Question Number

Experimental Competition
27 April 2010

Page 7 of 8

__________________________________________________________________________________________

Exp. II-C The Differential Quantum Efficiency of the


Collimated Laser Diode
The task of this part is to characterize the relative light intensity versus the current through
the collimated laser diode (CLD) and determine the Differential Quantum Efficiency ,
which will be defined below. Control the current of CLD in the range between 5 mA and 20
mA. Make sure that the PC is not saturated when the current is close to 20 mA. Filters or
polarizers can be used to avoid saturation.
(1) Control the CLD current and measure the corresponding PC resistance values. Record the
data in the data table. Transform your data and plot the PC conductance versus CLD
current on a graph paper. No error analysis is required.
(1.3 points)
(2) Based on the graph of step (1), choose a region (I ~ 3 mA) centered around the
maximum slope. By using the conductance versus P graph in Part II-A-(2), transform
and record the data of this region in the table of step (1) into the relative light intensity (J).
Plot the relative light intensity (J) versus CLD current (I) on a graph paper. No error
analysis is required.
(0.8 points)
(3) The maximum radiating power of the CLD is assumed to be exactly Pmax = 3 .0 mW.
Extract the maximum slope from the graph in step (2) and transfer it to the value of
P
G
, which is the maximum ratio of the increased amount of radiating power and
I max
the increase amount of input current. Write down your analysis and the calculated value G
on the answer sheet. Estimate the error of G. Do not include the error of the Pmax. Write
down your analysis and the calculated value G on the answer sheet.
(2.0 points)
(4) The Quantum Efficiency equals the probability of one photon being generated per
electron injected. From a particular bias current of the laser, a small increment of electrons
injected would cause a corresponding increment of photons. The Differential Quantum
Efficiency is defined as the ratio of the increased number of photons and the increased
number of injected electrons. Determine of your CLD by using the value of G obtained
in step (3). Write down your analysis and the calculated value on the answer sheet.

Question Number

Experimental Competition
27 April 2010

Page 8 of 8

__________________________________________________________________________________________

Estimate the error of . Write down your analysis and the calculated value on the
answer sheet. (Laser wavelength = 650 nm. Plancks constant = 6.63 1034 J s . Light
speed = 3.0 108 m s )

(0.9 points)

41st International Physics Olympiad Experimental Competition, July 21st, 2010

1/4

Experimental problem 2
There are two experimental problems. The setup on your table is used for both problems. You have 5
hours to complete the entire task (1&2).

Experimental problem 2: Forces between magnets, concepts of


stability and symmetry
Introduction
Electric current I circulating in a loop of area S creates a magnetic moment of magnitude m IS
[see Fig. 1(a)]. A permanent magnet can be thought of as a collection of small magnetic moments of
iron (Fe), each of which is analogous to the magnetic moment of a current loop. This (Ampres)
model of a magnet is illustrated in Fig. 1(b). The total magnetic moment is a sum of all small magnetic
moments, and it points from the south to the northern pole.
(a)

(b)

Figure 1. (a) Illustration of a current loop and the produced magnetic field. (b) Ampres model of a
magnet as a collection of small current loops.

Forces between magnets


To calculate the force between two magnets is a nontrivial theoretical task. It is known that like poles
of two magnets repel, and unlike poles attract. The force between two current loops depends on the
strengths of the currents in them, their shape, and their mutual distance. If we reverse the current in
one of the loops, the force between them will be of the same magnitude, but of the opposite
direction.
In this problem you experimentally investigate the forces between two magnets, the ring-magnet
and the rod-magnet. We are interested in the geometry where the axes of symmetry of the two
magnets coincide (see Fig. 2). The rod-magnet can move along the z - axis from the left, through the
ring-magnet, and then towards the right (see Fig. 2). Among other tasks, you will be asked to
measure the force between the magnets as a function of z . The origin
corresponds to the
case when the centers of the magnets coincide.

41st International Physics Olympiad Experimental Competition, July 21st, 2010

2/4

Figure 2. The rod- and the ring-magnet are aligned. The force between them changes as the rodmagnet moves along the z - axis.

To ensure motion of the rod-magnet along the axis of symmetry ( z - axis), the ring-magnet is firmly
embedded in a transparent cylinder, which has a narrow hole drilled along the z - axis. The rodmagnet is thus constrained to move along the z - axis through the hole (see Fig. 3). The
magnetization of the magnets is along the z - axis. The hole ensures radial stability of the magnets.

Figure 3. Photo of two magnets and a transparent hollow cylinder; the rod-magnet moves through
the cylinders hole.

Experimental setup (2nd problem)


The following items (to be used for the 2nd problem) are on your desk:
1.
2.
3.
4.
5.

Press (together with a stone block); see separate instructions if needed


Scale (measures mass up to 5000 g, it has TARA function, see separate instructions if needed)
A transparent hollow cylinder with a ring-magnet embedded in its side.
One rod-magnet.
One narrow wooden stick (can be used to push the rod magnet out of the cylinder).

The setup is to be used as in Fig. 4 to measure the forces between the magnets. The upper plate of
the press needs to be turned up-side-down in comparison to the first experimental problem. The
narrow Aluminum rod is used to press the rod-magnet through the hollow cylinder. The scale
measures the force (mass). The upper plate of the press can be moved downwards and upwards by
using a wing nut. Important: The wing nut moves 2mm when rotated 360 degrees.

41st International Physics Olympiad Experimental Competition, July 21st, 2010

3/4

Figure 4. Photograph of the setup, and the way it should be used for measuring the force between
the magnets.

Tasks
1. Determine qualitatively all equilibrium positions between the two magnets, assuming that
the z - axis is positioned horizontally as in Fig. 2 , and draw them in the answer sheet. Label
the equilibrium positions as stable (S)/unstable (U), and denote the like poles by shading, as
indicated for one stable position in the answer sheet. You can do this Task by using your
hands and a wooden stick. (2.5 points)
2. By using the experimental setup measure the force between the two magnets as a function
of the z - coordinate. Let the positive direction of the z - axis point into the transparent
cylinder (the force is positive if it points in the positive direction). For the configuration when
the magnetic moments are parallel, denote the magnetic force by F (z ) , and when they
are anti-parallel, denote the magnetic force by F (z ) . Important: Neglect the mass of the
rod-magnet (i.e., neglect gravity), and utilize the symmetries of the forces between
magnets to measure different parts of the curves. If you find any symmetry in the forces,
write them in the answer sheets. Write the measurements on the answer sheets; beside
every table schematically draw the configuration of magnets corresponding to each table (an
example is given). (3.0 points)
3. By using the measurements from Task 2, use the millimeter paper to plot in detail the
functional dependence F (z ) for z 0 . Plot schematically the shapes of the curves

F (z ) and F (z ) (along the positive and the negative z - axis). On each schematic graph

41st International Physics Olympiad Experimental Competition, July 21st, 2010

4/4

denote the positions of the stable equilibrium points, and sketch the corresponding
configuration of magnets (as in Task 1). (4.0 points)
4. If we do not neglect the mass of the rod magnet, are there any qualitatively new stable
equilibrium positions created when the z - axis is positioned vertically? If so, plot them on
the answer sheet as in Task 1. (0.5 points)

Experimental Question 2: An Optical Black Box

TV and computer screens have advanced significantly in recent years. Today, most displays consist of a color LCD
filter matrix and a uniform white backlight source. In this experiment, we will study a sample of plastic material which
was considered for use as an ingredient in the backlight illumination of LCD screens.
Equipment
On your desk, you have the following items (see Figure 1):
(1)
(2)
(3)
(4)

(5)
(6)
(7)
(8)
(9)
(10)
(11)

The sample - a piece of plastic material fixed in a slide frame. The sample is sensitive - do not touch it. To
adjust the samples position, use its holder and stand.
A holder and stand for the slide frame. The stand includes a handle which can be used for fine rotations of the
sample. Do not remove the slide frame from its holder.
A white LED flashlight. The flashlight can be turned on and off using a button at its rear end. Do not confuse
it with the laser (see Figure 2).
A red laser pointer. The laser is marked with a warning label. Do not confuse it with the white flashlight
(see Figure 2). The laser may be turned on and off by moving its black cap back and forth. Dont try to
remove the cap it may be dangerous, and you may break the laser. The lasers battery will weaken after
about an hour - do not keep it turned on longer than necessary. The lasers wavelength is
.
A single stand to be used for the two light sources. At the start of the experiment, the flashlight is fixed to
the stand, while the laser lies on the desk.
A white screen on wooden legs, covered with millimeter graph paper. There is a hole near the middle of the
screen. You are allowed to make markings on the screen.
A wooden stake that can be moved back and forth on a wooden bench. You are allowed to make markings on
the bench.
A tape measure.
A ruler.
Millimeter graph paper.
A desktop lamp which can be turned on or off for your convenience.

LASER SAFETY:
1.
2.
3.
4.

Do not stare into the laser beam!


Beware of reflections from metallic surfaces.
Do not point the laser at others.
Do not try to repair or disassemble the laser.
Call a supervisor if you require assistance.

Page 1 of 6

(6)
(7)
(1)
(3)
(2)

(4)
(8)

(5)

Figure 1: Summary of the equipment.


(1) The plastic sample.
(2) Sample holder and stand.
(3) White LED flashlight.
(4) Red laser.
(5) Stand for light source.
(6) Screen covered with millimeter graph paper.
(7) Wooden stake on bench.
(8) Tape measure.

Figure 2: Close-up of the two light sources.

Page 2 of 6

(4) Red
laser

(3) White
flashlight

Part I Theory (0.4 points)

a. (0.4 pts.) A light ray is reflected from two mirrors which meet at an angle (Figure 3). Find the angle
between the incoming and outgoing rays. Assume that all light rays lie in the plane perpendicular to the
mirrors intersection line.

Figure 3: A light ray


reflected from two mirrors.

Part II Measurements with white light (6.1 points)

Using the white flashlight as your light source, you may observe both the transmission and the reflection properties of
the sample. Figure 4 illustrates the suggested setups for both types of observation. Note: you may observe different
results when illuminating the two sides of the sample.
Transmission:

Reflection:
White
flashlight

White
flashlight

Screen
Figure 4: Suggested
observation setups
for white light.

Sample
Eye

Sample

CAUTION: For viewing transmitted light, you will have to look directly into the flashlight beam through the sample.
Dont do this with the laser! Also, avoid looking directly into the flashlight itself for long periods of time.

Page 3 of 6

b. (0.5 pts.) Figure 5 illustrates schematically four possibilities for the samples microscopic structure. stands
for the refractive index of the plastic. Choose the structure that best fits your observations. Note: the 5 periods
shown in the figure are for illustration only. In reality, is small, and the sample contains many periods.
(A)

(B)

Figure 5: Different
possibilities for the
samples structure.

(C)

(D)

c. (0.8 pts.) Find the angle

for the sample and estimate its error.

d. (0.5 pts.) When a perpendicular white light beam is incident on the sample from one of its sides, the following
faint pattern can be observed in the transmitted light, slightly to the right from the source (Figure 6). R, G
and B stand for red, green and blue respectively. Note: this pattern may be difficult to observe, and
measurements on it are not required.

Source
BGR

BGR

BGR

BGR

Further to the right you may observe a much brighter pattern (Figure 7):

Page 4 of 6

Figure 6: Faint
pattern near the
light source.

Figure 7: Bright pattern


farther to the right from
the light source.

Choose the correct option:


A. All the colored patterns result from interference.
B. All the colored patterns result from the dependence of on the wavelength.
C. The patterns depicted in Figure 6 result from interference, while the pattern depicted in Figure 7 results
from dependence of on the wavelength.
D. The patterns depicted in Figure 6 result from the dependence of on the wavelength, while the pattern
depicted in Figure 7 results from interference.
e. (1.4 pts.) With the white light set up as in part (d), measure the deflection angle of violet light (at the far
blue end of the spectrum) for the dominant peak depicted in Figure 7. The deflection angle is defined in Figure
8. Record all intermediate measurements. Provide error estimates.

Sample

f.

Figure 8: The
deflection angle

(1.4 pts.) Illumination of the sample at different angles of incidence results in different deflection angles for the
dominant transmitted peaks. Measure the minimal deflection angle
of the dominant peak for transmitted
violet light (there is only one such minimal angle). Record all intermediate measurements. Provide error
estimates.

g. (0.8 pts.) Using the angle from part (c), express the refraction index of the sample in terms of either
. You may use the reversibility of light propagation and the fact that there is only one minimal angle
h. (0.7 pts.) Find the refraction index

or

of the sample for violet light and its error estimate.

Part III - Laser measurements (3.5 points)

Remove the flashlight from the light-source stand, and replace it with the laser. You can use the white screen to view
both transmission and reflection patterns, as illustrated in Figure 9. The laser has a limited battery life - do not keep it
turned on longer than necessary. When aligning the components, it may help to rotate the laser around its axis.
WARNING: Do not look directly into the laser beam or its reflections! Do not look at the laser light through
the sample use the provided screen.
Page 5 of 6

Transmission

Reflection
Laser

Screen

Screen

Figure 9: Suggested
observation setups
for laser light.

Sample

Sample
Laser

Observe the alternating pattern of bright and dim fringes on the screen as you slightly rotate the sample. The dimming
of some of the fringes is due to destructive interference between different regions of each tooth on the sample.
i.

(1 pt.) Use one of the setups in Figure 9, with the sample illuminated perpendicularly by the laser beam.
Record the deflection angles of the observed fringes as a function of the fringe number . Define the center
of the pattern as
. Use the provided table on the answer form. Record all intermediate measurements.
Provide error estimations.

j.

(1.5 pts.) Using a linear graph, find the spacing between two adjacent teeth of the sample. Error bars on the
graph are not required. Provide error estimation for .

k. (1 pt.) Using the formula you derived in part (g), find the refraction index
of the sample for the lasers red
wavelength. Record any additional measurements. Provide error estimates. WARNING: Do not look through
the sample! Use the provided screen.

Page 6 of 6

Experimental Competition:
Problem 2

14 July 2011
Page 1 of 2

2. Mechanical Blackbox: a cylinder with a ball inside


A small massive particle (ball) of mass m is fixed at distance z below the top of a long hollow
cylinder of mass M . A series of holes are drilled perpendicularly to the central axis of the cylinder.
These holes are for pivoting so that the cylinder will hang in a vertical plane.
Students are required to perform necessary nondestructive measurements to determine the numerical
values of the following with their error estimates:
i.

position of centre of mass of cylinder with ball inside.


Also provide a schematic drawing of the experimental set-up for measuring the centre of
mass.
[1.0 points]

ii.
iii.
iv.

distance z
M
ratio
.
m
the acceleration due to gravity, g .

[3.5 points]
[3.5 points]
[2.0 points]

Equipment: a cylinder with holes plus a ball inside, a base plate with a thin pin, a pin cap, a ruler, a
stopwatch, thread, a pencil and adhesive tape.

z
O

pivot

xCM

Base plate
to be clamped
to a table top

CM

L
Thin pin
for pivoting

xCM is the distance from the top of the cylinder to the


centre of mass.
R is the distance from the pivoting point to the centre of
mass.

Experimental Competition:
Problem 2

14 July 2011
Page 2 of 2

Cylinder with holes


plus ball inside

Adhesive
tape

Pin cap

Stopwatch

Thread
(for balancing)

Base plate

Ruler

Caution: The thin pin is sharp. When it is not in use, it should be protected with a pin cap for safety.

Useful information:

d2
g M m R , where ICM is
dt 2
the moment of inertia of the cylinder with a ball about the centre of mass and is the angular
displacement.
2. For a long hollow cylinder of length L and mass M , the moment of inertia about the centre of
1. For such a physical pendulum,

m R2

ICM

1
L
mass with the rotational axis perpendicular to the cylinder can be approximated by M
.
3
2
3. The parallel axis theorem: I

I centre of mass

Mx 2 , where x is the distance from the rotation

point to the centre of mass, and M is the total mass of the object.
4. The ball can be treated as a point mass and it is located on the central axis of the cylinder.
5. Assume that the cylinder is uniform and the mass of the end-caps is negligible.

Solar cells

2.0 Introduction
Equipment used for this experiment is displayed in Fig. 2.1.

Figure 2.1 Equipment used for experiment E2.

List of equipment (see Fig. 2.1):


A: Solar cell
B: Solar cell
C: Box with slots for the mounting of light source, solar cells, etc.
D: LED-light source in holder
E: Power supply for light source D
F: Variable resistor
G: Holder for mounting single solar cell in the box C
H: Circular aperture for use in the box C
I: Holder for mounting two solar cells in the box C

Page 1 of 9

E2

Solar cells
J: Shielding plate for use in the box C
K: Digital multimeter
L: Digital multimeter
M: Wires with mini crocodile clips
N: Optical vessel (large cuvette)
O: Measuring tape
P: Scissors
Q: Tape
R: Water for filling the optical vessel N
S: Paper napkin for drying off excess water
T: Plastic cup for water from the optical vessel N (not shown in Fig. 2.1)
U: Plastic pipette (not shown in Fig. 2.1)
V: Lid for the box C (not shown in Fig. 2.1)

Data sheet: table of fundamental constants


Speed of light in vacuum
Elementary charge
Boltzmanns constant

A solar cell transforms part of the electromagnetic energy in the incident light to electric energy by
separating charges inside the solar cell. In this way an electric current can be generated. Experiment
E2 intents to examine solar cells with the use of the supplied equipment. This equipment consists of
a box with holders for light source and solar cells along with various plates and a lid. The variable
resistor should be mounted in the box, see Fig. 2.2. One of the three terminals on the resistor has
been removed, since only the two remaining terminals are to be used. Also supplied are wires with
mini crocodile clips and two solar cells (labeled with a serial number and the letter A or B) with
terminals on the back. The two solar cells are similar but can be slightly different. The two
multimeters have been equipped with terminals for designated use as ammeter and voltmeter,
respectively, see Fig. 2.3. Finally, the experiment will make use of an optical vessel together with
some drinking water from the bottle.

Page 2 of 9

E2

Solar cells

Figure 2.2 (a) Box with light source and resistor for mounting. (b) The resistor mounted in the box. Notice
that the small pin on the resistor fits in the hole to the right of the shaft.

Figure 2.3 Multimeters equipped with terminals for use as ammeter (left) and voltmeter (right),
respectively. The instrument is turned on by pressing POWER in the top left corner. The instrument turns
off automatically after a certain idle time. It can measure direct current and voltage
as well as alternating
current and voltage
. The internal resistance in the voltmeter is 10 M regardless of the measuring range.
The potential difference over the ammeter is 200 mV at full reading, regardless of the measuring range. In
case of overflow the display will show l, and you need to select a higher measuring range. The HOLD
button (top right corner) should not be pushed, except if you want to freeze a measurement.

Page 3 of 9

E2

Solar cells
WARNING: Do not use the multimeter as an ohmmeter on the solar cells since the measuring
current can damage them. When changing the measuring range on the multimeters, please turn
the dial with caution. It can be unstable and may break. Check whether there is a number under
the decimal point when measuring if the dial is not fully in place, the multimeter will not
measure, even if there are digits in the display.
Notice: Do not change the voltage on the power supply. It must be 12 V throughout the
experiment. (The power supply for the light source should be connected to the outlet (230 V ~) at
your table.)
Notice: Uncertainty considerations are only expected when explicitly mentioned.
Notice: All measured and calculated values must be given in SI units.
Notice: For all measurements of currents and voltages in this experiment, the LED-light
source is supposed to be on.

Page 4 of 9

E2

Solar cells

E2

2.1 The dependence of the solar cell current on the distance to the light source
For this question you will measure the current, , generated by the solar cell when in a circuit with
the ammeter, and determine how it depends on the distance, , to the light source. The light is
produced inside the individual light diodes and is therefore to be measured as shown in Fig. 2.4.

Figure 2.4 Top view of setup for question 2.1. Note the aperture a immediately in front of the solar cell A.
The distance is measured from inside the light diode to the surface of the solar cell.

Do not change the measuring range on the ammeter in this experiment: the internal resistance of the
ammeter depends on the measuring range and affects the current that can be drawn from the solar
cell. State the serial numbers of the light source and of solar cell A on your answer sheet. Mount the
light source in the U-shaped holder (the light source has a tight fit in the holder, so be patient when
mounting it. Mount solar cell A in the single holder and place it together with the circular aperture
immediately in front of the solar cell. The current as a function of the distance to the light source
can, when is not too small, be approximated by

where

and

are constants.

2.1a Measure I as a function of r, and set up a table of your measurements.

1.0

2.1b Determine the values of Ia and a by the use of a suitable graphical method.

1.0

Page 5 of 9

Solar cells

E2

2.2 Characteristic of the solar cell


Remove the circular aperture. Mount the variable resistor in the box as shown on Fig. 2.2. Place the
light source in slot number 0, furthest away from the resistor. Mount solar cell A in the single
holder without the circular aperture in slot number 10. Build a circuit as shown in Fig. 2.5, so that
you can measure the characteristic of the solar cell, i.e. the terminal voltage U of the solar cell as a
function of the current I in the circuit consisting of solar cell, resistor and ammeter.

Figure 2.5 Electrical diagram for measuring the characteristic in question 2.2.

2.2a Make a table of corresponding measurements of U and I.

0.6

2.2b Graph voltage as function of current

0.8

2.3 Theoretical characteristic for the solar cell


For the solar cells in this experiment, the current as function of the voltage is given by the equation
(

where the parameters


, and are constant at a given illumination. We take the temperature to
be
. The fundamental constants and
are the elementary charge and Boltzmanns
constant, respectively.
2.3a Use the graph from question 2.2b to determine

0.4

The parameter can be assumed to lie in the interval from 1 to 4. For some values of the potential
difference , the formula can be approximated by
(
2.3b

Estimate the range of values of


for which the mentioned approximation is good.
1.2
Determine graphically the values of and for your solar cell.

Page 6 of 9

Solar cells

E2

2.4 Maximum power for a solar cell


2.4a

The maximum power that the solar cell can deliver to the external circuit is denoted
. Determine
for your solar cell through a few, suitable measurements. (You 0.5
may use some of your previous measurements from question 2.2).

Estimate the optimal load resistance


, i.e. the total external resistance when the
2.4b solar cell delivers its maximum power to
. State your result with uncertainty and 0.5
illustrate your method with suitable calculations.

2.5 Comparing the solar cells


Mount both solar cells (A and B) in the double holder in slot number 15, see Fig. 2.6.

Figure 2.6 Top view of light source and solar cells in question 2.5.

Measure, for the given illumination:


- The maximum potential difference that can be measured over solar cell A.
2.5a
- The maximum current that can be measured through solar cell A.
Do the same for solar cell B.
2.5b

0.5

Draw electrical diagrams for your circuits showing the wiring of the solar cells and the
0.3
meters.

Page 7 of 9

Solar cells
2.6 Couplings of the solar cells
The two solar cells can be connected in series in two different ways as shown in Fig. 2.7. There are
also two different ways to connect them in parallel (not shown in the figure).

Figure 2.7 Two ways to connect the solar cells in series for question 2.6. The two ways to connect them in
parallel are not shown.

Determine which of the four arrangements of the two solar cells yields the highest
possible power in the external circuit when one of the solar cells is shielded with the
shielding plate (J in Fig. 2.1). Hint: You can estimate the maximum power quite well by
2.6
1.0
calculating it from the maximum voltage and maximum current measured from each
configuration.
Draw the corresponding electrical diagram.

2.7 The effect of the optical vessel (large cuvette) on the solar cell current
Mount the light source in the box and place solar cell A in the single holder with the circular
aperture immediately in front, so that there is approximately 50 mm between the solar cell and the
light source. Place the empty optical vessel immediately in front of the circular aperture as shown in
Fig. 2.8.

Page 8 of 9

E2

Solar cells

E2

Figure 2.8 Experimental set-up for question 2.7.

2.7a

Measure the current I, now as a function of the height, h, of water in the vessel, see Fig.
1.0
2.8. Make a table of the measurements and draw a graph.

2.7b Explain with only sketches and symbols why the graph looks the way it does.

1.0

Mount the light source in the box and place solar cell A in the single holder so that the distance
between the solar cell and the light source is maximal. Place the circular aperture immediately in
front of the solar cell.

For this set-up do the following:


- Measure the distance between the light source and the solar cell and the current .
- Place the empty vessel immediately in front of the circular aperture and measure the
2.7c
0.6
current .
- Fill up the vessel with water, almost to the top, and measure the current .
Use your measurements from 2.7c to find a value for the refractive index
for water.
2.7d Illustrate your method with suitable sketches and equations. You may include additional 1.6
measurements.

Page 9 of 9

Experimental problem:

Semiconductor element
), an adjustable resistor (up to 140 ),

In this experiment a semiconductor element (

a fixed resistor (300 ), a 9-V-direct voltage source, cables and two multimeters are at disposal.
It is not allowed to use the multimeters as ohmmeters.
a) Determine the current-voltage-characteristics of the semiconductor element taking into
account the fact that the maximum load permitted is 250 mW. Write down your data in
tabular form and plot your data. Before your measurements consider how an overload of the
semiconductor element can surely be avoided and note down your thoughts. Sketch the
circuit diagram of the chosen setup and discuss the systematic errors of the circuit.
b) Calculate the resistance (dynamic resistance) of the semiconductor element for a current of
25 mA.
c) Determine the dependence of output voltage U2 from the input voltage U1 by using the
circuit described below. Write down your data in tabular form and plot your data.

The input voltage U1 varies between 0 V and 9 V. The semiconductor element is to be


placed in the circuit in such a manner, that U2 is as high as possible. Describe the entire
circuit diagram in the protocol and discuss the results of the measurements.
d) How does the output voltage U2 change, when the input voltage is raised from 7 V to 9 V?
Explain qualitatively the ratio U1 / U2.
e) What type of semiconductor element is used in the experiment? What is a practical
application of the circuit shown above?
Hints: The multimeters can be used as voltmeter or as ammeter. The precision class of these
instruments is 2.5% and they have the following features:
measuring range

50 A 300 A 3 mA

internal resistance 2 k

1 k

100

30 mA 300 mA

0,3 V 1 V

3V

10

6 k 20 k

60 k 200 k

11

10 V

Experimental Problems
Problem 4: Lens experiment
The apparatus consists of a symmetric biconvex lens, a plane mirror, water, a meter stick,
an optical object (pencil), a supporting base and a right angle clamp. Only these parts may
be used in the experiment.
a) Determine the focal length of the lens with a maximum error of 1 %.
b) Determine the index of refraction of the glass from which the lens is made.
The index of refraction of water is nw = 1.33. The focal length of a thin lens is given by
1 1
1
= ( n 1) ,
f
r1 r2
where n is the index of refraction of the lens material and r1 and r2 are the curvature
radii of the refracting surfaces. For a symmetric biconvex lens we have r1 = - r2 = r, for a
symmetric biconcave lens r1 = - r2 = - r .
Solution of problem 4:
a) For the determination of fL , place the lens on the mirror
and with the clamp fix the pencil to the supporting base.
Lens and mirror are then moved around until the
vertically downward looking eye sees the pencil and its
image side by side.
In order to have object and image in focus at the same
time, they must be placed at an equal distance to the eye.
In this case object distance and image distance are the
same and the magnification factor is 1 .
It may be proved quite accurately, whether magnification 1 has in fact been obtained, if
one concentrates on parallatical shifts between object and image when moving the eye:
only when the distances are equal do the pencil-tips point at each other all the time.
The light rays pass the lens twice because they are reflected by the mirror. Therefore
the optical mapping under consideration corresponds to a mapping with two lenses
placed directly one after another:
11

1.2

Experimental competition
Exercise A

Follow the acceleration and the deceleration of a brass disk, driven by an


AC electric motor. From the measured times of half turns, plot the angle,
angular velocity and angular acceleration of the disk as functions of time.
Determine the torque and power of the motor as functions of angular
velocity.
Instrumentation
1. AC motor with switch and brass disk
2. Induction sensor
3. Multichannel stop-watch (computer)

Instruction
The induction sensor senses the iron pegs, mounted on the disk, when
they are closer than 0.5 mm and sends a signal to the stop-watch. The
stop-watch is programmed on a computer so that it registers the time
at which the sensor senses the approaching peg and stores it in memory. You run the stop-watch by giving it simple numerical commands,
i. e. pressing one of the following numbers:
5 MEASURE.
The measurement does not start immediately. The stop-watch waits
until you specify the number of measurements, that is, the number
of successive detections of the pegs:
3 30 measurements
6 60 measurements
Either of these commands starts the measurement. When a measurement is completed, the computer displays the results in graphic
form. The vertical axis represents the length of the interval between
detection of the pegs and the horizontal axis is the number of the
interval.
4

7 display results in numeric form.


The first column is the number of times a peg has passed the detector, the second is the time elapsed from the beginning of the measurement and the third column is the length of the time interval
between the detection of the two pegs.
In the case of 60 measurements:
8 displays the first page of the table
2 displays the second page of the table
4 displays the results graphically.
A measurement can be interrupted before the prescribed number of measurements by pressing any key and giving the disk another half turn.
The motor runs on 25 V AC. You start it with a switch on the mounting
base. It may sometimes be necessary to give the disk a light push or to
tap the base plate to start the disk.
The total moment of inertia of all the rotating parts is: (14.0 0.5)
106 kgm2 .
Exercise B
Locate the position of the centers and determine the orientations of a
number of identical permanent magnets hidden in the black painted block.
A diagram of one such magnet is given in Figure 1. The coordinates x, y
and z should be measured from the red corner point, as indicated in Figure 2.
~ in the
Determine the z component of the magnetic induction vector B
(x, y) plane at z = 0 by calibrating the measuring system beforehand.
Find the greatest magnetic induction B obtainable from the magnet supplied.

Instrumentation
1. Permanent magnet given is identical to the hidden magnets in the
block.
2. Induction coil; 1400 turns, R = 230
5

Fig. 1

Fig. 2

3. Field generating coils, 8800 turns, R = 990 , 2 pieces


4. Black painted block with hidden magnets
5. Voltmeter (ranges 1 V, 3 V and 10 V recommended)
6. Electronic circuit (recommended supply voltage 24 V)
7. Ammeter
8. Variable resistor 3.3 k
9. Variable stabilized power supply 0 25 V, with current limiter
10. Four connecting wires
11. Supporting plate with fixing holes
12. Rubber bands, multipurpose (e. g. for coil fixing)
13. Tooth picks
14. Ruler
15. Thread

Instructions
For the magnet-search only nondestructive methods are acceptable. The
final report should include results, formulae, graphs and diagrams. The
diagrams should be used instead of comments on the methods used wherever possible.
The proper use of the induced voltage measuring system is shown in Figure 3.
6

This device is capable of responding to the magnetic field. The peak voltage is proportional to the change of the magnetic flux through the coil.
The variable stabilized power supply is switched ON (1) or OFF (0) by the
lower left pushbutton. By the (U) knob the output voltage is increased
through the clockwise rotation. The recommended voltage is 24 V. Therefore switch the corresponding toggle switch to the 12 V 25 V position.
With this instrument either the output voltage U or the output current I is
measured, with respect to the position of the corresponding toggle switch
(V,A). However, to get the output voltage the upper right switch should be
in the Vklop position. By the knob (I) the output current is limited bellow
the preset value. When rotated clockwise the power supply can provide
1.5 A at most.

Fig. 3 0 zero adjust dial, 1 push reset button


Note: permeability of empty space 0 = 1.2 106 Vs/Am.

&RQVLGHUDQHXWURQVWDUZLWK



D

DPDVVRI

DQDYHUDJHUDGLXVRI
DQG DURWDWLRQSHULRGRI

NJ
P
V

&DOFXODWHWKHIODWWHQLQJIDFWRUJLYHQWKDWWKHJUDYLWDWLRQDOFRQVWDQWLV
1PNJ

,QWKHORQJUXQ RYHUPDQ\\HDUV WKHURWDWLRQRIWKHVWDUVORZVGRZQGXHWRHQHUJ\ORVV


DQGWKLVOHDGVWRDGHFUHDVHLQWKHIODWWHQLQJ7KHVWDUKDVKRZHYHUDVROLGFUXVWWKDW
IORDWVRQDOLTXLGLQWHULRU7KHVROLGFUXVWUHVLVWVDFRQWLQXRXVDGMXVWPHQWWRHTXLOLEULXP
VKDSH,QVWHDGVWDUTXDNHVRFFXUZLWKVXGGHQFKDQJHVLQWKHVKDSHRIWKHFUXVWWRZDUGV
HTXLOLEULXP'XULQJDQGDIWHUVXFKDVWDUTXDNHWKHDQJXODUYHORFLW\LVREVHUYHGWR
FKDQJHDFFRUGLQJWRILJXUH

DQJXODUYHORFLW\
RIWKHFUXVW V

Figure 1

time (days) -->

A sudden change in the shape of the crust of a


neutron star results in a sudden change of the
angular velocity.
E

&DOFXODWHWKHDYHUDJHUDGLXVRIWKHOLTXLGLQWHULRUXVLQJWKHGDWDRI)LJ0DNH
WKHDSSUR[LPDWLRQWKDWWKHGHQVLWLHVRIWKHFUXVWDQGWKHLQWHULRUDUHWKHVDPH
,JQRUHWKHFKDQJHLQVKDSHRIWKHLQWHULRU 

4XHVWLRQ

'HWHUPLQDWLRQRIWKHHIILFLHQF\RID/('

,QWURGXFWLRQ
,QWKLVH[SHULPHQWZHVKDOOXVHWZRPRGHUQVHPLFRQGXFWRUVWKHOLJKWHPLWWLQJGLRGH
/(' DQGWKHSKRWRGLRGH 3' ,QD/('SDUWRIWKHHOHFWULFDOHQHUJ\LVXVHGWRH[FLWH
HOHFWURQVWRKLJKHUHQHUJ\OHYHOV:KHQVXFKDQH[FLWHGHOHFWURQIDOOVEDFNWRDORZHU
HQHUJ\OHYHODSKRWRQZLWKHQHUJ\(SKRWRQLVHPLWWHGZKHUH

+HUHKLV3ODQFN
VFRQVWDQWFLVWKHVSHHGRIOLJKWDQG LVWKHZDYHOHQJWKRIWKHHPLWWHG
OLJKW7KHHIILFLHQF\RIWKH/('LVGHILQHGWREHWKHUDWLREHWZHHQWKHUDGLDWHGSRZHU
DQGWKHHOHFWULFDOSRZHUXVHG3/('

,QDSKRWRGLRGHUDGLDQWHQHUJ\LVWUDQVIRUPHGLQWRHOHFWULFDOHQHUJ\:KHQOLJKWIDOOVRQ
WKHVHQVLWLYHVXUIDFHRIDSKRWRGLRGHVRPH EXWQRWDOO RIWKHSKRWRQVIUHHVRPH EXW
QRWDOO RIWKHHOHFWURQVIURPWKHFU\VWDOVWUXFWXUH7KHUDWLREHWZHHQWKHQXPEHURI
LQFRPLQJSKRWRQVSHUVHFRQG1SDQGWKHQXPEHURIIUHHGHOHFWURQVSHUVHFRQG1HLV
FDOOHGWKHTXDQWXPHIILFLHQF\TS

7KHH[SHULPHQW
7KHSXUSRVHRIWKLVH[SHULPHQWLVWRGHWHUPLQHWKHHIILFLHQF\RID/('DVDIXQFWLRQRI
WKHFXUUHQWWKDWIORZVWKURXJKWKH/('7RGRWKLVZHZLOOPHDVXUHWKHLQWHQVLW\RIWKH
HPLWWHGOLJKWZLWKDSKRWRGLRGH7KH/('DQGWKH3'KDYHEHHQPRXQWHGLQWZR
ER[HVDQGWKH\DUHFRQQHFWHGWRDFLUFXLWSDQHO )LJ %\PHDVXULQJWKHSRWHQWLDO
GLIIHUHQFHDFURVVWKH/('DQGDFURVVWKHUHVLVWRUV5DQG5RQHFDQGHWHUPLQHERWK
WKHSRWHQWLDOGLIIHUHQFHVDFURVVDQGWKHFXUUHQWVIORZLQJWKURXJKWKH/('DQGWKH3'
:HXVHWKHPXOWLPHWHUWRPHDVXUH92/7$*(6RQO\7KLVLVGRQHE\WXUQLQJWKHNQRE
WRSRVLWLRQ
9
7KHPHWHUVHOHFWVWKHDSSURSULDWHVHQVLWLYLW\UDQJHDXWRPDWLFDOO\,IWKH
GLVSOD\LVQRWRQ$872VZLWFKRIIDQGSXVKRQ9DJDLQ&RQQHFWLRQ&20DQG
96
7KHER[FRQWDLQLQJWKHSKRWRGLRGHDQGWKHER[FRQWDLQLQJWKH/('FDQEHPRYHG
IUHHO\RYHUWKHERDUG,IERWKER[HVDUHSRVLWLRQHGRSSRVLWHWRHDFKRWKHUWKHQWKH/('
WKH3'DQGWKHKROHLQWKHER[FRQWDLQLQJWKH3'UHPDLQLQDVWUDLJKWOLQH
'DWD7KHTXDQWXPHIILFLHQF\RIWKHSKRWRGLRGH
7KHGHWHFWLRQVXUIDFHRIWKH3'LV
7KHZDYHOHQJWKRIWKHOLJKWHPLWWHGIURPWKH/('LV
7KHLQWHUQDOUHVLVWDQFHRIWKHYROWPHWHULV

TS 
[PP
QP
06LQWKHUDQJHXSWR
P9
06LQWKHRWKHUUDQJHV
7KHUDQJHLVLQGLFDWHGE\VPDOOQXPEHUVRQWKHGLVSOD\
3ODQFN
VFRQVWDQW
K -V
7KHHOHPHQWDU\TXDQWXPRIFKDUJH
H &
7KHVSHHGRIOLJKWLQYDFXR
F PV

Figure 1.
5 6
5 YDULDEOHUHVLVWRU
5 06
7KHSRLQWVODEHOOHGDQG
DUHPHDVXULQJSRLQWV
Figure 2 The experimental setup: a board and the
two boxes containing the LED and the photo-diode.

,QVWUXFWLRQV


%HIRUHZHFDQGHWHUPLQHWKHHIILFLHQF\RIWKH/('ZHPXVWILUVWFDOLEUDWHWKH
SKRWRGLRGH7KHSUREOHPLVWKDWZHNQRZQRWKLQJDERXWWKH/('
6KRZH[SHULPHQWDOO\WKDWWKHUHODWLRQEHWZHHQWKHFXUUHQWIORZLQJWKURXJKWKH
SKRWRGLRGHDQGWKHLQWHQVLW\RIOLJKWIDOOLQJRQLW,>-VP@LVOLQHDU



'HWHUPLQHWKHFXUUHQWIRUZKLFKWKH/('KDVPD[LPDOHIILFLHQF\



&DUU\RXWDQH[SHULPHQWWRPHDVXUHWKHPD[LPDO DEVROXWH HIILFLHQF\RIWKH/('

1RPDUNV SRLQWV ZLOOEHDOORFDWHGIRUDQHUURUDQDO\VLV LQ7+,6H[SHULPHQWRQO\ 


3OHDVHVXPPDUL]HGDWDLQWDEOHVDQGJUDSKVZLWKFOHDULQGLFDWLRQVRITXDQWLWLHV DQG
XQLWV 

4XHVWLRQ

'HWHUPLQDWLRQRIWKHUDWLRRIWKHPDJQHWLFILHOGVWUHQJWKVRI
WZRGLIIHUHQWPDJQHWV

,QWURGXFWLRQ
:KHQDFRQGXFWRUPRYHVLQDPDJQHWLFILHOGFXUUHQWVDUHLQGXFHGWKHVHDUHWKH
VRFDOOHGHGG\FXUUHQWV$VDFRQVHTXHQFHRIWKHLQWHUDFWLRQEHWZHHQWKHPDJQHWLFILHOG
DQGWKHLQGXFHGFXUUHQWVWKHPRYLQJFRQGXFWRUVXIIHUVDUHDFWLYHIRUFH7KXVDQ
DOXPLQLXPGLVNWKDWURWDWHVLQWKHQHLJKERXUKRRGRIDVWDWLRQDU\PDJQHWH[SHULHQFHVD
EUDNLQJIRUFH
0DWHULDODYDLODEOH









$VWDQG
$FODPS
$QKRPRJHQLXVDOXPLQLXPGLVNRQDQD[OHLQDKROGHUWKDWFDQURWDWH
7ZRPDJQHWV7KHJHRPHWU\RIHDFKLVWKHVDPH XSWR HDFKFRQVLVWVRIDFOLS
FRQWDLQLQJWZRVPDOOPDJQHWVRILGHQWLFDOPDJQHWL]DWLRQDQGDUHDWKHZKROH
SURGXFLQJDKRPRJHQLXVILHOG%RU%
7ZRZHLJKWV2QHZHLJKWKDVWZLFHWKHPDVV XSWR RIWKHRWKHU
$VWRSZDWFK
$UXOHU

Figure 1.
7KHH[SHULPHQW
7KHDOXPLQLXPGLVNLVIL[HGWRDQD[OHDURXQGZKLFKDFRUGLVZUDSSHG$ZHLJKWKDQJV
IURPWKHFRUGDQGZKHQWKHZHLJKWLVUHOHDVHGWKHGLVNDFFHOHUDWHVXQWLODFRQVWDQW
DQJXODUYHORFLW\LVUHDFKHG7KHWHUPLQDOVSHHGGHSHQGVDPRQJRWKHUWKLQJVRQWKH
PDJQLWXGHRIWKHPDJQHWLFILHOGVWUHQJWKRIWKHPDJQHW
7ZRPDJQHWVRIGLIIHUHQWILHOGVWUHQJWKV%RU%DUHDYDLODEOH(LWKHUFDQEHILWWHGRQWR
WKHKROGHUWKDWFDUULHVWKHDOXPLQLXPGLVNWKH\PD\EHLQWHUFKDQJHG
,QVWUXFWLRQV


7KLQNRIDQH[SHULPHQWLQZKLFKWKHUDWLRRIWKHPDJQHWLFILHOGVWUHQJWKV%DQG%
RIWKHWZRPDJQHWVFDQEHPHDVXUHGDVDFFXUDWHO\DVSRVVLEOH



*LYHDVKRUWWKHRUHWLFDOWUHDWPHQWLQGLFDWLQJKRZRQHFDQREWDLQWKHUDWLRIURP
WKHPHDVXUHPHQWV



&DUU\RXWWKHH[SHULPHQWDQGGHWHUPLQHWKHUDWLR



*,9($1(5525(67,0$7,21

8VHRIWKHVWRSZDWFK

Figure 2.
7KHVWRSZDWFKKDVWKUHHEXWWRQV66DQG6 VHH)LJ 
%XWWRQ6WRJJOHVEHWZHHQWKHGDWHWLPHDQGWKHVWRSZDWFKPRGHV6ZLWFKWRWKH
VWRSZDWFKPRGH2QHVKRXOGVHHWKLV

2QSUHVVLQJ6RQFHWKHVWRSZDWFKEHJLQVWLPLQJ7RVWRSLWSUHVV6DVHFRQGWLPH
7KHVWRSZDWFKFDQEHUHVHWWR]HURE\SUHVVLQJ6RQFH

Experimental Question 1
Terminal velocity in a viscous liquid
An object falling in a liquid will eventually reach a constant velocity, called the terminal velocity. The
aim of this experiment is to measure the terminal velocities of objects falling through glycerine.
For a sphere of radius r falling at velocity v through a viscous liquid, the viscous force F is given by
F = 6rv. Here is a property of the liquid called the viscosity. In this experiment you will measure
the terminal velocity of metal cylinders (because cylinders are easier to make than spheres). The diameter
of each cylinder is equal to its length, and we will assume the viscous force on such a cylinder is similar
to the viscous force on a sphere of the same diameter, 2r:
Fcyl = 6r m v

(1)

where = 1, m = 1 for a sphere.

Preliminary
Calculation of terminal velocity (2 marks)
If is the density of the culinder and 0 is the density of the liquid, show that the terminal velocity vT
of the cylinder is given by
vT = Cr 3m ( 0 )
(2)
where C is a constant and derive a expression for C.

Experiment
Use the equipment available to determine the numerical value of the exponent m (10 marks) and the
density of glycerine (8 marks).

Notes
For consistency, try to ensure that the cylinders fall in the same orientation, with the axis of the
cylinder horizontal.
The tolerances on the diameter and the length of the cylinders are 0.05 mm (you need not measure
them yourself).
There is a brass sieve inside the container that you should use to retrieve the metal cylinders.
Important: make sure the sieve is in place before dropping objects into the glycerine, otherwise you
will not be able to retrieve them for repeat measurements.
When glycerine absorbs water from the atmosphere, it becomes less viscous. Ensure that the
cylinder of glycerine is covered with the plastic film provided when not in use.
Do not mix cylinders of different size and different material after the experiment.
Density (kgm3 )
2.70 103
4.54 103
7.87 103
8.96 103

Material
Aluminium
Titanium
Stainless steel
Copper

THE PHYSICAL PENDULUM


A physical pendulum is an extended physical object of arbitrary shape that can
rotate about a fixed axis. For a physical pendulum of mass M oscillating about
a horizontal axis a distance, l, from the centre of mass, the period, T, for small
angle oscillations is
I
2
T=
+l
(1)
g Ml
Here g is the acceleration of gravity, and I is the moment of inertia of the pendulum about an axis parallel to the rotation axis but through the centre of mass.

146

Figure 2 shows a schematic drawing of the physical pendulum you will be using. The pendulum consists of a cylindrical metal rod, actually a long screw,
having length L, average radius R, and at least one nut. The values of various
dimensions and masses are summarised in Table 1. By turning the nut you can
place it at any position along the rod. Figure 2 defines two distances, x and l,
that describe the position of the rotation axis relative to the end of the rod and
the centre of mass, respectively.

Figure 2: Schematic drawing of the pendulum


with definition of important quantities.

Rod
Length
Average radius
Mass
Distance between screw threads
Nut
Height
Depth of groove
Mass

L
R
MROD

(400.0 0.4) mm
(4.4 0.1) mm
(210.2 0.2) x10-3 kg
(1.5000 0.0008) mm

h
d
MNUT

(9.50 0.05) mm
(0.55 0.05) mm
(4.89 0.03) x10-3 kg

Table 1: Dimensions and weights of the pendulum


A reminder from the front page: No points will be given for error estimates except in 2c. However, it is expected that the correct number of significant figures are given.
147

Section 1 : Period of oscillation versus rotation axis position


(4 marks)
a) Measure the oscillation period, T, as a function of the position x, and present
the results in a table.
b) Plot T as a function of x in a graph. Let 1 mm in the graph correspond to
1 mm in x and 1 ms in T. How many positions give an oscillation period equal
to T = 950 ms, T = 1000 ms and T = 1100 ms, respectively?
c) Determine the x and l value that correspond to the minimum value in T.

Section 2 : Determination of g (5 marks)


For a physical pendulum with a fixed moment of inertia, I, a given period, T,
may in some cases be obtained for two different positions of the rotation axis.
Let the corresponding distances between the rotation axis and the centre of
mass be l1 and l2 . Then the following equation is valid:
I
l1 l2 =
(2)
M

a) Figure 6 on the last page in this set illustrates a physical pendulum with an
axis of rotation displaced a distance l1 from the centre of mass. Use the information given in the figure caption to indicate all positions where a rotation axis
parallel to the drawn axis can be placed without changing the oscillation period.
b) Obtain the local Oslo value for the acceleration of gravity g as accurately as
possible. Hint: There are more than one way of doing this. New measurements
might be necessary. Indicate clearly by equations, drawings, calculations etc.
the method you used.
c) Estimate the uncertainty in your measurements and give the value of g with
error margins.

Section 3 : Geometry of the optical timer (3 marks)

148

a) Use direct observation and reasoning to characterise, qualitatively as well as


quantitatively, the shape of the reflecting surface of the nut (the mirror). (You
may use the light from the light bulb in front of you).
Options (several may apply):
1. Plane mirror
2. Spherical mirror
3. Cylindrical mirror
4. Cocave mirror
5. Convex mirror
In case of 2-5: Determine the radius of curvature.
b) Consider the light source to be a point source, and the detector a simple photoelectric device. Make an illustration of how the light from the emitter is reflected by the mirror on the nut in the experimantal setup (side view and top
view). Figure 7 on the last page in this set shows a vertical plane through the
timer display (front view). Indicate in this figure the whole region where the
reflected light hits this plane when the pendulum is vertical.

Section 4 : Measurement of magnetic field (4 marks)


You will now use an electronic sensor (Hall-effect sensor) to measure magnetic
field. The device gives a voltage which depends linearly on the vertical field
through the sensor. The field-voltage coefficient is V / B = 22.6 V/T (Volt/
Tesla). As a consequence of its design the sensor gives a non-zero voltage
(zero-offset voltage) in zero magnetic field. Neglect the earths magnetic field.

Figure 3: Schematics of the magnetic field detector system


a) Connect the sensor to the battery and voltmeter as shown above. Measure
the zero-offset voltage, V0 .
A permanent magnet shaped as a circular disk is mounted on a separate stand.
The permanent magnet can be displaced vertically by rotating the mount screw,
which is threaded identically to the pendulum rod. The dimensions of the permanent magnet are; thickness t = 2.7 mm, radius r = 12.5 mm.
b) Use the Hall sensor to measure the vertical magnetic field, B, from the permanent magnet along the cylinder axis, see Figure 4. Let the measurements
cover the distance from y = 26 mm (use the spacer) to y = 3.5 mm, where
y = 1 mm corresponds to the sensor and permanent magnet being in direct
contact. Make a graph of your data for B versus y.

Figure 4: Definition of the distance y between top of magnet and the active part
of the sensor.

149

c) It can be shown that the field along the axis of a cylindrical magnet is given
by the formula

y+t
y
B( y) = B0

(3)
2
2
y 2 + r 2
( y + t ) + r
where t is the cylinder thickness and r is the radius. The parameter B0 characterizes the strength of the magnet. Find the value of B 0 for your permanent
magnet. Base your determination on two measured B-values obtained at different y.

Section 5 : Determination of magnetic dipole moment (4 marks)

150

A tiny magnet is attached to the white end of the pendulum rod. Mount the pendulum on the stand with its magnetic end down and with x = 100 mm. Place
the permanent magnet mount under the pendulum so that both the permanent
magnet and the pendulum have common cylinder axis. The alignment should
be done with the permanent magnet in its lowest position in the mount. (Always avoid close contact between the permanent magnet and the magnetic end
of the pendulum.)
a) Let z denote the air gap spacing between the permanent magnet and the
lower end of the pendulum. Measure the oscillation period, T, as function of the
distance, z. The measurement series should cover the interval from z = 25 mm
to z = 5.5 mm while you use as small oscillation amplitude as possible. Be
aware of the possibility that the period timer might display 2T (see remark regarding the timer under Instrumentation above). Plot the observed T versus z.
b) With the additional magnetic interaction the pendulum has a period of oscillation, T, which varies with z according to the relation
B0
1
1+
f ( z)
(4)
2
Mgl
T
Here stand for proportional to, and is the magnetic dipole moment of
the tiny magnet attached to the pendulum, and
is the parameter determined
in section 4c. The function f(z) includes the variation in magnetic field with
distance. In Figure 5 on the next page you find the particular f(z) for our experiment, presented as a graph.
Select an appropirate point on the graph to determine the unknown magnetic
moment .

2 B0 is a material property called remanent magnetic induction, Br .

60

50

f(z)

40

30

151

20

10

0
0

10

15

20

25

30

z (mm)
Figure 5. Graph of the dimension-less function f(z) used in section 5b.

152

Figure 6. For use in section 2a. Mark all positions where a rotation axis
(orthogonal to the plane of the paper) can be placed without changing the
oscillation period. Assume for this pendulum (drawn on scale, 1:1) that
I/M = 2100 mm2. (Note: In this booklet the size of this figure is about 75% of
the size in the original examination paper.)

Figure 7. For use in section 3b. Indicate the whole area where the reflected
light hits when the pendulum is vertical.

Include this page in your report!

29th International Physics Olympiad


Reykjavik, Iceland

Experimental competition
Monday, July 6th, 1998

Time available: 5 hours

Read this first:

Use only the pen provided.


1. Use only the front side of the answer sheets and paper.
2. Use as little text as possible in your answers; express yourself primarily with equations, numbers
and figures. Summarize your results on the answer sheets.
3. Please indicate on all sheets the name of your team, your student number, the page number and the
total number of pages.
4. At the end of the exam please put your answer sheets, pages and graphs in numerical order and
leave them on your table.
5. Use of a calculator is allowed.

This set of problems consists of 6 pages.

Examination prepared at: University of Iceland, Department of Physics.

Instrumentation provided:
A

Platform with 6 banana jacks

Pickup coil embedded into the


platform

Ferrite U-core with two coils


marked ``A'' and ``B''

Ferrite U-core without coils

Aluminium foils of thicknesses:


50 m, 100 m and 200 m

Function generator with output


leads

Two multimeters

Six leads with banana plugs

Two rubber bands and two


plastic spacers

Multimeters
The multimeters are two-terminal devices that in this experiment are used for measuring AC voltages, AC
currents, frequency and resistance. In all cases one of the terminals is the one marked COM. For the
voltage, frequency and resistance measurements the other terminal is the red one marked V-. For current
measurements the other terminal is the yellow one marked mA. With the central dial you select the meter
function (V~ for AC voltage, A~ for AC current, Hz for frequency and for resistance) and the
measurement range. For the AC modes the measurement uncertainty is (4% of reading + 10 units of the
last digit). To get accurate current measurements a change of range is recommended if the reading is
less than 10% of full scale.
Function generator
To turn on the generator you press in the red button marked PWR. Select the 10 kHz range by pressing the
button marked 10k, and select the sine waveform by pressing the second button from the right marked with
a wave symbol. No other buttons should be selected. You can safely turn the amplitude knob fully
clockwise. The frequency is selected with the large dial on the left. The dial reading multiplied by the range
selection gives the output frequency. The frequency can be verified at any time with one of the multimeters.
Use the output marked MAIN, which has 50 internal resistance.
Ferrite cores
Handle the ferrite cores gently, they are brittle!! Ferrite is a ceramic magnetic material, with low electrical
conductivity. Eddy current losses in the cores are therefore low.
Banana jacks
To connect a coil lead to a banana jack, you loosen the colored plastic nut, place the tinned end between the
metal nut and plastic nut, and tighten it again.

Figure 1: Experimental arrangement for part I.


Part I. Magnetic shielding with eddy currents
Time-dependent magnetic fields induce eddy currents in conductors. The eddy currents in turn produce a
counteracting magnetic field. In superconductors the induced eddy currents will expel the magnetic field
completely from the interior of the conductor. Due to the finite conductivity of normal metals they are not
as effective in shielding magnetic fields.
To describe the shielding effect of aluminium foils we will apply a phenomenological model

B = B0 e d

(1)
where B0 is the magnetic field in the absence of foils. B is the magnetic field beneath the foils, an
attenuation constant, and d the foil thickness.
Experiment
Put the ferrite core with the coils, with legs down, on the raised block such that coil A is directly above the
pickup coil embedded in the platform, as shown in Figure 1. Secure the core on the block by stretching the
rubber bands over the core and under the block recess.
1. Connect the leads for coils A and B to the jacks. Measure the resistance of all coils to make sure
you have good connections. You should expect values of less than 10 . Write your values in field
1 on the answer sheet.
2. Collect data to validate the model above and evaluate the attenuation constant for the
aluminum foils (50 - 300 m), for frequencies in the range of 5 - 20 kHz. Place the foils inside the
square, above the pickup coil, and apply a sinusoidal voltage to coil A. Write your results in field
2 on the answer sheet.
3. Plot versus frequency, and write in field 3 on the answer sheet, an expression describing the
function (f) .

Part II. Magnetic flux linkage


The response of two coils on a closed ferrite core to an external alternating voltage (Vg) from a sinusoidal
signal generator is studied.

Theory
In the following basic theoretical discussion, and in the treatment of the data, it is assumed that the ohmic
resistance in the two coils and hysteresis losses in the core have insignificant influence on the measured
currents and voltages. Because of these simplifications in the treatment below, some deviations will occur
between measured and calculated values.
Single coil
Let us first look at a core with a single coil, carrying a current I. The magnetic flux , that the current
creates in the ferrite core inside the coil, is proportional to the current I and to the number of windings N.
The flux depends furthermore on a geometrical factor g, which is determined by the size and shape of the
core, and the magnetic permeability =r0 , which describes the magnetic properties of the core material.
The relative permeability is denoted r and 0 is the permeability of free space.
The magnetic flux is thus given by

= gNI = cNI

(2)

where c=g. The induced voltage is given by Faraday's law of induction,

d(t )
dI (t )
= cN 2
dt
dt

(t ) = N

(3)

The conventional way to describe the relationship between current and voltage for a coil is through the self
inductance of the coil L, defined by,

(t ) = L

dI (t )
dt

(4)

A sinusoidal signal generator connected to the coil will drive a current through it given by

I (t ) = I 0 sin t

(5)

where is the angular frequency and I0 is the amplitude of the current. As follows from equation (3) this
alternating current will induce a voltage across the coil given by

(t ) = cN 2 I 0 cos t

(6)
The current will be such that the induced voltage is equal to the signal generator voltage Vg. There is a 90
degree phase difference between the current and the voltage. If we only look at the magnitudes of the
alternating voltage and current, allowing for this phase difference, we have

= cN 2 I

(7)

Two coils
Let us now assume that we have two coils on one core. Ferrite cores can be used to link magnetic flux
between coils. In an ideal core the flux will be the same for all cross sections of the core. Due to flux
leakage in real cores a second coil on the core will in general experience a reduced flux compared to the

flux-generating coil. The flux B in the secondary coil B is therefore related to the flux A
coil A through

B = k A

in the primary

(8)

in coil A. The
Similarly a flux component B created by a current in coil B will create a flux A =kB
factor k, which is called the coupling factor, has a value less than one.
The ferrite core under study has two coils A and B in a transformer arrangement. Let us assume that coil A
is the primary coil (connected to the signal generator). If no current flows in coil B (IB=0), the induced
voltage A due to IA is equal and opposite to Vg. The flux created by IA inside the secondary coil is
determined by equation (8) and the induced voltage in coil B is

B = kcN A N B I A

(9)

If a current IB flows in coil B, it will induce a voltage in coil A which is described by a similar expression.
The total voltage across the coil A will then be given by

V g = A = cN A2 I A kcN A N B I B

(10)

The current in the secondary coil thus induces an opposing voltage in the primary coil, leading to an
increase in IA. A similar equation can be written for B. As can be verified by measurements, k is
independent of which coil is taken as the primary coil.
Experiment
Place the two U-cores together as shown in Figure 2, and fasten them with the rubber bands. Set the
function generator to produce a 10 kHz, sine wave. Remember to set the multimeters to the most sensitive
range suitable for each measurement. The numbers of the windings of the two coils, A and B, are: NA =
150 turns and NB = 100 turns (1 turn on each coil).

Figure 2: A transformer with a closed magnetic circuit.


1.

2.

3.

Derive algebraic expressions for the self inductances LA and LB , and the coupling factor k, in
terms of measured and given quantities and write your results in field 1.a on the answer sheet.
Draw circuit diagrams in field 1.b on the answer sheet, showing how these quantities are
determined. Calculate the numerical values of LA , LB and k and write their values in field 1.c
on the answer sheet.
When the secondary coil is short-circuited, the current IP in the primary coil will increase. Use
the equations above to derive an expression giving IP explicitly and write your result in field 2.a
on the answer sheet. Measure IP and write your value in field 2.b on the answer sheet.
Coils A and B can be connected in series in two different ways such that the two flux
contributions are either added to or subtracted from each other.
3.1. Find the self inductance of the serially connected coils, LA+B , from measured quantities in
the case where the flux contributions produced by the current I in the two coils add to (strengthen)
each other and write your answer in field 3.1 on the answer sheet.

4.
5.
6.

3.2. Measure the voltages VA and VB when the flux contributions of the two coils oppose each
other. Write your values in field 3.2.a on the answer sheet and the ratio of the voltages in field
3.2.b. Derive an expression for the ratio of the voltages across the two coils and write it in field
3.2.c on the answer sheet.
Use the results obtained to verify that the self inductance of a coil is proportional to the square of
the number of its windings and write your result in field 4 on the answer sheet.
Verify that it was justified to neglect the resistances of the coils and write your arguments as
mathematical expressions in field 5 on the answer sheet.
Thin plastic spacers inserted between the two half cores (as shown in Figure 3) reduce the coil
inductances drastically. Use this reduction to determine the relative permeability r of the ferrite
material, given Ampere's law and continuity of the magnetic field B across the ferrite - plastic
interface.

Assume =0 =410-7 Ns2/C2 for the plastic spacers and a spacer thickness of 1.6 mm. The geometrical
factor can be determined from Ampere's law

Bdl = I

total

(11)

where Itotal is the total current flowing through a surface bounded by the integration path. Write your
algebraic expression for r in field 6.a on the answer sheet and your numerical value in field 6.b.

Figure 3: The ferrite cores with the two spacers in place.

30th International Physics Olympiad


Padua, Italy
Experimental competition
Tuesday, July 20th, 1999

Before attempting to assemble your equipment, read the problem text


completely!
Please read this first:
1.
2.
3.
4.

The time available is 5 hours for one experiment only.


Use only the pen provided.
Use only the front side of the provided sheets.
In addition to "blank" sheets where you may write freely, there is a set of Answer sheets
where you must summarize the results you have obtained. Numerical results must be
written with as many digits as appropriate; dont forget the units. Try whenever possible
to estimate the experimental uncertainties.
5. Please write on the "blank" sheets the results of all your measurements and whatever else
you deem important for the solution of the problem, that you wish to be evaluated during
the marking process. However, you should use mainly equations, numbers, symbols,
graphs, figures, and use as little text as possible.
6. It's absolutely imperative that you write on top of each sheet that you'll use: your name
(NAME), your country (TEAM), your student code (as shown on your identification tag,
CODE), and additionally on the "blank" sheets: the progressive number of each sheet (from
1 to N, Page n.) and the total number (N) of "blank" sheets that you use and wish to be
evaluated (Page total); leave the Problem field blank. It is also useful to write the
number of the section you are answering at the beginning of each such section. If you use
some sheets for notes that you dont wish to be evaluated by the marking team, just put a large
cross through the whole sheet, and dont number it.
7. When you've finished, turn in all sheets in proper order (answer sheets first, then used
sheets in order, unused sheets and problem text at the bottom) and put them all inside the
envelope where you found them; then leave everything on your desk. You are not allowed
to take anything out of the room.
This problem consists of 11 pages (including this one and the answer sheets).
This problem has been prepared by the Scientific Committee of the 30th IPhO, including professors at the Universities
of Bologna, Naples, Turin and Trieste.

28/09/09

Experimental problem

Page 1

Torsion pendulum
In this experiment we want to study a relatively complex mechanical system a torsion
pendulum and investigate its main parameters. When its rotation axis is horizontal it
displays a simple example of bifurcation.

Available equipment
1. A torsion pendulum, consisting of an outer body (not longitudinally uniform) and an inner threaded
rod, with a stand as shown in figure 1
2. A steel wire with handle
3. A long hexagonal nut that can be screwed onto the pendulum threaded rod (needed only for the
last exercise)
4. A ruler and a right triangle template
5. A timer
6. Hexagonal wrenches
7. A3 Millimeter paper sheets.
8. An adjustable clamp
9. Adhesive tape
10. A piece of T-shaped rod

The experimental apparatus is shown in figure 1; it is a torsion pendulum that can oscillate
either around a horizontal rotation axis or around a vertical rotation axis. The rotation axis is
defined by a short steel wire kept in tension. The pendulum has an inner part that is a threaded
rod that may be screwed in and out, and can be fixed in place by means of a small hexagonal
lock nut. This threaded rod can not be extracted from the pendulum body.
When assembling the apparatus in step 5 the steel wire must pass through the brass
blocks and through the hole in the pendulum, then must be locked in place by keeping it
stretched: lock it first at one end, then use the handle to put it in tension and lock it at the
other end.
Warning: The wire must be put in tension only to guarantee the pendulum stability. It's
not necessary to strain it with a force larger than about 30 N. While straining it,
don't bend the wire against the stand, because it might break.

28/09/09

Experimental problem

Page 2

steel wire

handle

Figure 1: Sketch of the experimental apparatus when its rotation axis is horizontal.

The variables characterizing the pendulum oscillations are:


the pendulum position defined by the angle of deviation from the direction
perpendicular to the plane of the stand frame, which is shown horizontal in figure 1.
the distance x between the free end of the inner threaded rod and the pendulum rotation
axis
the period T of the pendulum oscillations.
The parameters characterizing the system are:
the torsional elastic constant (torque = angle) of the steel wire;
the masses M1 and M2 of the two parts of the pendulum (1: outer cylinder 1 and 2:
threaded rod);

1 Including the small hex locking nut.

28/09/09

Experimental problem

Page 3

the distances R1 and R2 of the center of mass of each pendulum part (1: outer cylinder and
2: threaded rod) from the rotation axis. In this case the inner mobile part (the threaded rod)
is sufficiently uniform for computing R2 on the basis of its mass, its length and the
distance x. R2 is therefore a simple function of the other parameters;
the moments of inertia I1 and I2 of the two pendulum parts (1: outer cylinder and 2:
threaded rod). In this case also we assume that the mobile part (the threaded rod) is
sufficiently uniform for computing I2 on the basis of its mass, its length and the
distance x. I2 is therefore also a simple function of the other parameters;
the angular position 0 (measured between the pendulum and the perpendicular to the
plane of the stand frame) where the elastic recall torque is zero. The pendulum is locked
to the rotation axis by means of a hex screw, opposite to the threaded rod; therefore 0
varies with each installation of the apparatus.

Summing up, the system is described by 7 parameters: , M1, M2, R1, I1, , 0, but 0
changes each time the apparatus is assembled, so that only 6 of them are really constants and
the purpose of the experiment is that of determining them, namely , M1, M2, R1, I1, ,
experimentally. Please note that the inner threaded rod cant be drawn out of the pendulum
body, and initially only the total mass M1 + M2 is given (it is printed on each pendulum).
In this experiment several quantities are linear functions of one variable, and you
must estimate the parameters of these linear functions. You can use a linear fit, but alternative
approaches are also acceptable. The experimental uncertainties of the parameters can be
estimated from the procedure of the linear fit or from the spread of experimental data about
the fit.
The analysis also requires a simple formula for the moment of inertia of the inner
part (we assume that its transverse dimensions are negligible with respect to its length, see
figure 2):
I 2 ( x) =

s 2 ds =

(x
3

) 3 (3x

( x )3 =

3 2 x + 3

(1)

where = M 2 / is the linear mass density, and therefore


I 2 ( x ) = M 2 x 2 M 2 x +

M2 2

(2)

rotation
axis

x-

s
x

Figure 2: In the analysis of the experiment we can use an equation (eq. 2) for the moment of inertia of
a bar whose transverse dimensions are much less than its length. The moment of inertia must be
computed about the rotation axis that in this figure crosses the s axis at s=0.

Now follow these steps to find the 6 parameters M1, M2, , R1, , I1:

28/09/09

Experimental problem

1.

2.

Page 4

The value of the total mass M1+M2 is given (it is printed on the pendulum), and you can
find M1 and M2 by measuring the distance R(x) between the rotation axis and the center
of mass of the pendulum. To accomplish this write first an equation for the position R(x)
of the center of mass as a function of x and of the parameters M1, M2, R1, .
[0.5
points]
Now measure R(x) for several values of x (at least 3) 2. Clearly such measurement must
be carried out when the pendulum is not attached to the steel wire. Use these
measurements and the previous result to find M1 and M2.
[3 points]

Figure 3: The variables and x and the parameters 0 and are shown here.

3.
4.

Find an equation for the pendulum total moment of inertia I as a function of x and of the
parameters M2, I1 and .
[0.5 points]
Write the pendulum equation of motion in the case of a horizontal rotation axis, as a
function of the angle (see figure 3) and of x, , 0, M1, M2, the total moment of
inertia I and the position R(x) of the center of mass.
[1 point]

2 The small hex nut must be locked in place every time you move the threaded rod. Its mass is included in M .
1

This locking must be repeated also in the following, each time you move the threaded rod.

28/09/09

Experimental problem

5.

Page 5

In order to determine , assemble now the pendulum and set it with its rotation axis
horizontal. The threaded rod must initially be as far as possible inside the pendulum.
Lock the pendulum to the steel wire, with the hex screw, at about half way between the
wire clamps and in such a way that its equilibrium angle (under the combined action of
weight and elastic recall) deviates sizeably from the vertical (see figure 4). Measure the
[4 points]
equilibrium angle e for several values of x (at least 5).

Figure 4: In this measurement set the pendulum so that its equilibrium position deviates from the
vertical.

6.
7.

Using the last measurements, find .


[4.5 points]
Now place the pendulum with its rotation axis vertical 3 , and measure its oscillation
[4
period for several values of x (at least 5). With these measurements, find I1 and .
points]

At this stage, after having found the system parameters, set the experimental
apparatus as follows:
pendulum rotation axis horizontal
threaded rod as far as possible inside the pendulum
pendulum as vertical as possible near equilibrium
finally add the long hexagonal nut at the end of the threaded rod by screwing it a few
turns (it cant go further than that)
In this way the pendulum may have two equilibrium positions, and the situation
varies according to the position of the threaded rod, as you can also see from the generic
graph shown in figure 5, of the potential energy as a function of the angle .
The doubling of the potential energy minimum in figure 5 illustrates a phenomenon
known in mathematics as bifurcation; it is also related to the various kinds of symmetry
breaking that are studied in particle physics and statistical mechanics.

3 In order to stabilize it in this position, you may reposition the stand brackets.

28/09/09

Experimental problem

Page 6

Figure 5: Graph of the function U ( ) = a ( 0 ) 2 + cos (which is proportional to the


2

potential energy of this problem) as a function of , with 0 0. The various curves


correspond to different a values as labeled in the figure; smaller values of a (a < 1)
correspond to the appearence of the bifurcation. In our case the parameter a is associated
with the position x of the threaded rod.

We can now study this bifurcation by measuring the period of the small oscillations
about the equilibrium position:
8.

Plot the period 4 T as a function of x. What kind of function is it? Is it increasing,


decreasing or is it a more complex function?
[2.5 points]

4 You may be able to observe two equilibrium positions, but one of them is more stable than the other (see
figure 5). Report and plot the period for the more stable one.

28/09/09

1st ASIAN PHYSICS OLYMPIAD


KARAWACI, INDONESIA

EXPERIMENTAL COMPETITION

APRIL 27, 2000

Time available : 5 hours

READ THIS FIRST :


1. Use only the pen provided.
2. Use only the marked side of the paper
3. Each problem should be answered on separate sheets
4. In your answers please user primarily symbols, equations,
numbers, graphs, tables and as little text as possible.
5. Write at the top of every sheet in your report:
Your candidate number (APhO identification number).
The problem number and section identification, e.g.2/a.
Number each sheet consecutively.
6. Write on the front page the total number of sheets in your report

This set of problems consists of . pages

Problem 1
Determination of the Density of Oil

Listed below are the only apparatus and materials available for your experiment :
1.
2.
3.
4.
5.
6.
7.
8.
9.

Test tube with uniform cross-section over most of its length between its to ends.
Vessel
Ruler
Eye dropper
Graph papers
Drying cloth/tissue papers
Rubber band for level marking
Distilled water with density 1.00 g/cm3
Oil in a plastic cup

In this experiment, you are to determine the density of the oil without measuring the
dimensions of the tube. You should not put both oil and water in the tube at the same
time.
Include the following in your report:
a. The theoretical basis for the analysis
b. A description of the method and procedure of the experiment
c. Final value for the density of oil
The errors and their sources

Problem 2
Determination of the Stefan-Boltzmann Constant
Listed below are the only apparatus and materials available for your experiment:
1.
2.
3.
4.
5.

DC power supply
Heater mounted on a ceramic base
Digital voltmeter (labeled V) and ammeter (labeled A)
Caliper
Aluminum cylinder with polished surface and a hole to house the heater. The
cylinder is fitted with a thermocouple (iron constantan) for measuring its
temperature.
6. Thermally isolated vessel containing water and ice for maintaining the cold
(reference) junction of the thermocouple at the constant temperature of 00 C.
7. Digital mV-meter (labeled mV) to be connected with the thermocouple
8. A table listing the calibrated thermoelectric characteristics of the thermocouple
for converting the mV readings into the corresponding temperatures
9. Electric cables
10. Candle and safety matches for blackening the cylinder
A note on the theoretical principle:
The effective radiation of power by an object with surface S at absolute temperature T in
equilibrium with its surrounding is given by the formula
P = e S (T 4 T04 )
where is the Stefan-Boltzmann constant, T0 I the absolute temperature of the
surrounding, and e=1 for an ideal blackbody while e=0 for an ideal reflector. The room
temperature will be given.
In this experiment, you are to determine the Stefan-Boltzmann constant .
Include the following in your report
a.
b.
c.
d.

The theoretical basis for the experiment


A description of the method and procedure of the experiment
The final value of Stefan-Boltzmann constant
The errors and their sources

Warnings:
1. Be careful in handling some of the elements during the experiment
As they may become very hot ( > 100 0 C ) at some stages
2. Be sure that the power supply current for the heater never exceeds
2A at all stages of the experiment.

April/18/2001

2nd Asian Physics Olympiad


TAIPEI, TAIWAN
Experimental Competition
Thursday, April 26, 2001
Time Available : 5 hours
Read This First:
1. Use only the pen provided.
2. Use only the front side of the answer sheets and paper.
3. In your answers please use as little text as possible ; express yourself
primarily in equations, numbers and figures. If the required result is a
numerical number, underline your final result with a wavy line.
4. Write on the blank sheets of paper the results of all your measurements
and whatever else you consider is required for the solution of the
question and that you wish to be marked.
5. It is absolutely essential that you enter in the boxes at the top of each
sheet of paper used your Country and your student number (Student
No.). In addition, on the blank sheets of paper used for each question,
you should enter the number of the question (Question No.), the
Section label, the progressive number of each sheet (Page No.) and the
total number of blank sheets that you have used and wish to be marked
for each question (Total No. of pages). If you use some blank sheets of
paper for notes that you do not wish to be marked, put a large cross
through the whole sheet and do not include them in your numbering.
6. At the end of the exam please put your answer sheets and graphs in
order.

-1-

April/18/2001

Basic Characteristics of Solar Cells


I. Background description
The purpose of this experiment is to explore the basic characteristics of solar cells.
Solar cells can absorb electromagnetic waves and convert the absorbed photon
energy into electrical energy. A solar cell mainly consists of a diode, whose
forward dark current-voltage relationship (i.e., I-V curve under no light
illumination) can be expressed as
I = Io(eV 1),
where Io and are constants.
The diode is made up of a semiconductor with a band gap of Ec Ev (see Fig. 1).
When the energy of the incident photon is larger than the band gap, the photon
can be absorbed by the semiconductor to create an electron-hole pair. The
electrons and holes are then driven by the internal electric field in the diode to
produce a photocurrent (light-generated current). There are several important
parameters other than light-generated current involved in a solar cell.

electron

(conduction band)

photon
energy gap

(valence band)
hole
Fig. 1
Brief explanations of the terminologies and basic principles are listed below:
(1) Short-circuit current (Isc) is the output current of the solar cell when the
external circuit is shorted, i.e., zero load resistance.
(2) Open-circuit voltage (Voc) is the output voltage of the solar cell when the
external circuit is open, i.e., infinite load resistance. Voc is also referred to as
photovoltaic voltage.
(3) Pm is the maximum output power of the solar cell, i.e., the maximum value
of IV.
(4) The filling factor (FF) is defined to be Pm/(IscVoc), which represents an
important parameter used to evaluate the quality of the solar cell.
-2-

April/18/2001

(5)

(6)

Because the photocurrent is produced by photon absorption by the


semiconductor, the spectral response of the photocurrent can be used to
determine the semiconductor band gap. From the band gap value, one can
infer the particular semiconductor material used.
Any incident photon with photon energy larger than the semiconductor band
gap can contribute to the photocurrent (Iph) of the solar cell, thus

where N( ) is the number of electrons per unit wavelength produced by


photons with wavelength ,
is the cut-off wavelength of the optical
filter (see Fig. 2, Fig. 3, and Fig. 4), and o is the longest wavelength capable
of producing a photocurrent. Here N( ) is approximately constant in the
visible spectral range, and each optical filter provided in this experiment cuts
off all light with wavelengths shorter than a certain cut-off wavelength
.
Therefore, the spectral response of Iph with an optical filter can be simplified
to be
Iph (o ).
(7)

The photon energy E is related to the photon wavelength as E = 1240/; the


unit for is nm (10-9m) and the unit for E is eV (electron-volt) in this
equation.
II. Equipments and materials
(1) A solar cell in a black box with pre-installed electrical connections
(2) Two digital electrical multimeters
(3) A set of dry cell batteries (1.5 V 2)
(4) One precision variable resistor (0-5 k)
(Caution: Do not connect the central connecting lead (the red wire) of
the variable resistor directly to the battery, it may damage the resisitor.)
(5) A white light source with power supply
(6) Two polarizers (Note: these polarizers are less effective for light with
wavelengths shorter than that of yellow light,that is the cut-off wavelength
of the yellow filter.)
(7) Red, orange, and yellow optical filters, one of each (their spectral
specifications are shown in Fig. 2, Fig. 3, and Fig. 4)
(8) An optical mount for the optical filters or polarizers (Note: optical filters and
polarizers can be mounted together on the same optical mount.)
(9) One optical bench
(10) Wire joining devices: 6 small springs
(11) One 45 cm ruler
-3-

April/18/2001

(12) Regular graph paper (10 sheets), semi-log graph paper (5 sheets)
(13) Two light-shielding boards
(Note: To avoid deterioration due to heat, polarizers and filters should be set at a
distance as far away from the light source as possible.)

Fig. 2

Fig. 3

-4-

April/18/2001

Fig. 4
III. Experimental steps
(1) (Question (1) : 3 points)
Measure the dark I-V characteristic of the forward biased solar cell.
a. Draw a diagram of the electrical circuit you used.
b. Plot the I-V curve and determine the values of and Io using the I-V data
you obtained..
(2) (Question (2) : 7 points)
Measure the characteristics of the solar cell, without electrical bias under
white light illumination. (Note: the distance between the light source and the
solar cell box should be kept at 30 cm as shown in Fig. 5.)
light source

solar cell box

30 cm

optical bench

a.

Fig. 5
Draw the circuit diagram you used.
-5-

April/18/2001

(3)

b. Measure the short-circuit current , Isc.


c. Measure the open-circuit voltage, Voc.
d. Measure the I vs. V relationship of the solar cell with varying load
resistance and plot the I-V curve.
e. Determine the maximum output power of the solar cell.
f. Determine the load resistance for the maximum output power.
g. Calculate the filling factor, FF Pm/(IscVoc)
(Question (3) : 3.0 points)
Assume that the solar cell can be modeled as a device consisting of an ideal
current source (light-generated current source), an ideal diode, a shunt
resistance Rsh, and a series resistance Rs,
a. Draw an equivalent circuit diagram for the solar cell under light
illumination.
b. Derive the I-V relationship for the equivalent circuit. Express the result
in terms of
,
, Iph (light-generated current), and Id (the current
c.

passing through the diode).


Assuming that
= infinity and

= 0 and can be neglected, find

the I-V relationship and prove that it can be written in the form as given
below:

where Voc is the open-circuit voltage, Isc is the short-circuit current, and
I0 ,
are constants.
(4)

(5)

(Question (4) : 4 points)


Find effects of irradiance.
a. Measure and plot the Isc vs. relative light intensity curve, and determine
the approximate functional relationship between Isc and the relative light
intensity.
b. Measure and plot the Voc vs. relative light intensity curve, and determine
the approximate functional relationship between Voc and the relative
light intensity.
(Question (5) : 3.0 points)
Find the wavelength response of the solar cell.
a. Measure and plot the Isc vs. different cut-off wavelengths using the three
optical filters.
b. Estimate the longest wavelength for which the solar cell can function
properly.
-6-

April/18/2001

c.

Infer which semiconductor material the solar cell is made of. (Hint: the
band gaps for commonly used semiconductors are InAs: 0.36 eV, Ge:
0.67 eV, Si: 1.1 eV, amorphous Si(a-Si : H): 1.7 eV, GaN: 3.5 eV)

-7-

IPhO2001-experimental competition

Experimental Competition
Saturday, June 30th, 2001

Please read this first:


1.
2.
3.
4.
5.

The time available is 5 hours for the experimental competition.


Use only the pen provided.
Use only the front side of the paper.
Begin each part of the problem on a separate sheet.
For each question, in addition to the blank sheets where you may write, there is an answer form
where you must summarize the results you have obtained. Numerical results should be written
with as many digits as are appropriate to the given data.
6. Write on the blank sheets of paper the results of all your measurements and whatever else you
consider is required for the solution of the question. Please use as little text as possible; express
yourself primarily in equations, numbers, figures and plots.
7. Fill in the boxes at the top of each sheet of paper used by writing your Country no and Country
code, your student number (Student No.), the number of the question (Question No.), the
progressive number of each sheet (Page No.) and the total number of blank sheets used for each
question (Total No. of pages). Write the question number and the section label of the part you
are answering at the beginning of each sheet of writing paper. If you use some blank sheets of
paper for notes that you do not wish to be marked, put a large X across the entire sheet and do
not include it in your numbering.
8. At the end of the exam, arrange all sheets in the following order;
answer form
used sheets in order
the sheets you do not wish to be marked
unused sheets and the printed question
Place the papers inside the envelope and leave everything on your desk. You are not allowed

to take any sheets of paper and any material used in the experiment out of the
room.

IPhO2001-experimental competition
ROTATING LIQUID
This experiment consists of three basic parts:
1. investigation of the profile of the rotating liquids surface and the determination of the
acceleration due to gravity,
2. investigation of the rotating liquid as an optical system,
3. determination of the refractive index of the liquid.
When a cylindrical container filled with a liquid rotates about the vertical axis passing through its
center with a uniform angular velocity , the liquids surface becomes parabolic (see Figure 1). At
equilibrium, the tangent to the surface at the point P(x, y) makes an angle with the horizontal such
that
2 x
tan =
for x R
(1)
g
where R is the radius of the container and g is the acceleration due to gravity.
It can further be shown that for <max (where max is the angular speed at which the center of the
rotating liquid touches the bottom of the container)
R
at x=x0=
, y(x0)=h0
(2)
2
that is; the height of the rotating liquid is the same as if it were not rotating.
The profile of the rotating liquids surface is a parabola defined by the equation
x2
y = y0 +
(3)
4C
where the vertex is at V(0, y0) and the focus is at F(0, y0+C). When optical rays parallel to the axis
of symmetry (optical axis) reflect at the parabolic surface, they all focus at the point F (see Fig.1).

IPhO2001-experimental competition
Apparatus

A cylindrical rigid plastic cup containing liquid glycerin. Millimetric scales are attached to
the bottom and the sidewall of this cup.
A turntable driven by a small dc electric motor powered by a variable voltage supply, which
controls the angular velocity.
A transparent horizontal screen on which you can put transparent or semi-transparent
millimetric scales. The location of the screen can be adjusted along the vertical and
horizontal directions.
A laser pointer mounted on a stand. The position of the pointer can be adjusted. The head of
the pointer can be changed.
Additional head for the laser pointer.
A ruler.
A highlighter pen.
A stopwatch. Push the left button to reset, the middle button to select the mode, and the right
button to start and stop the timing.
Transmission gratings with 500 or 1000 lines/mm.
Bubble level.
Glasses.

IMPORTANT NOTES

DO NOT LOOK DIRECTLY INTO THE LASER BEAM. BE AWARE THAT LASER
LIGHT CAN ALSO BE DANGEROUS WHEN REFLECTED OFF A MIRROR-LIKE
SURFACE. FOR YOUR OWN SAFETY USE THE GIVEN GLASSES.

Throughout the whole experiment carefully handle the cup containing glycerin.

The turntable has already been previously adjusted to be horizontal. Use bubble level only
for horizontal alignment of the screen.

Throughout the entire experiment you will observe several spots on the screen produced by
the reflected and/or refracted beams at the various interfaces between the air, the liquid, the
screen, and the cup. Be sure to make your measurements on the correct beam.

In rotating the liquid change the speed of rotation gradually and wait for long enough times
for the liquid to come into equilibrium before making any measurements.

IPhO2001-experimental competition

EXPERIMENT
PART 1: DETERMINATION of g USING a ROTATING LIQUID [7.5 pts]

Derive Equation 1.
Measure the height h0 of the liquid in the container and the inner diameter 2R of the
container.
Insert the screen between the light source and the container. Measure the distance H
between the screen and the turntable (see Figure 2).
Align the laser pointer such that the beam points vertically downward and hits the surface of
R
the liquid at a distance x0=
from the center of the container.
2
Rotate the turntable slowly. Be sure that the center of the rotating liquid is not touching the
bottom of the container.
R
It is known that at x0=
the height of the liquid remains the same as the original height
2
h0, regardless of the angular speed . Using this fact and measurements of the angle of the
surface at x0 for various values of , perform an experiment to determine the gravitational
acceleration g.
Prepare tables of measured and calculated quantities for each .
Produce the necessary graph to calculate g.
Calculate the value of g and the experimental error in it
Copy the values 2R, x0, h0, H and the experimental value of g and its error onto the answer
form.

IPhO2001-experimental competition
PART 2: OPTICAL SYSTEM
In this part of the experiment the rotating liquid will be treated as an image forming optical system.
Since the curvature of the surface varies with the angular speed of rotation, the focal distance of this
optical system depends on .
2a) Investigation of the focal distance [5.5 pts]

Align the laser pointer such that the laser beam is directed vertically downward at the center
of the container. Mark the point P where the beam strikes the screen. Thus the line joining
this point to the center of the cup is the optical axis of this system (see Figure 2).
Since the surface of the liquid behaves like a parabolic mirror, any incident beam parallel to
the optical axis will pass through the focal point F on the optical axis after reflection.
Adjust the speed of rotation to locate the focal point on the screen. Measure the angular
speed of rotation and the distance H between the screen and the turntable.
Repeat the above steps for different H values.
Copy the measured values of 2R and h0 and the value of at each H onto the answer form.
With the help of an appropriate graph of your data, find the relationship between the focal
length and the angular speed. Copy your result onto the answer form.

2b) Analysis of the image (what you see on the screen) [3.5 pts]
In this part of the experiment the properties of the image produced by this optical system will be
analyzed. To do so, follow the steps given below.

Remove the head of the laser pointer by turning it counterclockwise.


Mount the new head (provided in an envelope) by turning it clockwise. Now your laser
produces a well defined shape rather than a narrow beam.
Adjust the position of the laser pointer so that the beam strikes at about the center of the cup
almost normally.
Put a semitransparent sheet of paper on the horizontal screen, which is placed close to the
cup, such that the laser beam does not pass through the paper, but the reflected beam does.
Observe the size and the orientation of the image produced by the source beam and the
beam reflected from the liquid when it is not rotating.
Start the liquid rotating, and increase the speed of rotation gradually up to the maximum
attainable speed while watching the screen. As increases you might observe different
frequency ranges over which the properties of the image are drastically different. To
describe these observations complete the table on the answer form by adding a row to this
table for each such frequency range and fill it in by using the appropriate notations
explained on that page.

IPhO2001-experimental competition
PART 3: REFRACTIVE INDEX [3.5 pts]
In this part of the experiment the refractive index of the given liquid will be determined using a
grating. When monochromatic light of wavelength is incident normally on a diffraction grating,
the maxima of the diffraction pattern are observed at angles m given by the equation
m = d sin m
(4)
where, m is the order of diffraction and d is the distance between the rulings of the grating. In this
part of the experiment a diffraction grating will be used to determine the wavelength of the laser
light and the refractive index of the liquid (see Figure 3).

Use the grating to determine the wavelength of the laser pointer. Copy your result onto the
answer form.
Immerse the grating perpendicularly into the liquid at the center of the cup.
Align the laser beam such that it enters the liquid from the sidewall of the cup and strikes the
grating normally.
Observe the diffraction pattern produced on the millimetric scale attached to the cup on the
opposite side. Make any necessary distance measurements.
Calculate the refractive index n of the liquid by using your measurements. (Ignore the effect
of the plastic cup on the path of the light.)
Copy the result of your experiment onto the answer form.

IPhO2001-experimental competition

F(0,y0+C)

V(0,y0)
P(x,y)
h0

x
R
Figure 1. Definitions of the bank angle at point P(x,y), the vertex V and the focus F for the
parabolic surface produced by rotating the liquid, of initial height h0 and radius R, at a constant
angular speed around the y-axis.

IPhO2001-experimental competition

+P

22

h0
R

x
5

3
4

220V AC

Figure 2 Experimental setup for parts 1 and 2.


1. Laser pointer on a stand, 2. Transparent screen, 3. Motor, 4. Motor controller, 5. Turntable, 6.
Axis of rotation, 7. Cylindrical container.

IPhO2001-experimental competition

1
n
3
m
R

Figure 3 Top view of the grating in a liquid experiment.


1. Scaled sidewall, 2. Grating on a holder, 3. Laser pointer, 4. Cylindrical container.

IPhO2001-experimental competition
Country no

Country code

Student No.

Question No.

Page No.

Total
No. of pages

  
 
1) Determination of g using a rotating liquid
2R

x0

h0

Experimental value of g:

2a) Investigation of the focal distance


2R

h0

Relation between focal length and :

10

IPhO2001-experimental competition
Country no

Country code

Student No.

Question No.

Page No.

Total
No. of pages

2b) Analysis of the image

Use the appropriate notations explained below to describe what you see on
the screen due to reflected beam
range: For the frequency ranges only approximate values are required.
Orientation (in comparison with the object beam as seen on the transparent screen):
Inverted
: INV
Erect
: ER
Variation of the size with increasing :
Increases
:I
Decreases : D
No change : NC

For the frequency ranges you have found above:


Write R if the screen is above the focal point.
Write V if the screen is below the focal point.
Range

Orientation

Variation
of the size

image

=0

11

IPhO2001-experimental competition
3) Refractive index

Wavelength =

Experimental value for n =

12

3rd Asian Physics Olympiad


Singapore

Experimental Competition
(The Experimental Competition consists of two parts, Part A and Part B)
May 10, 2002

PART

Time Available: 2_ hours for Part A


Read This First
1 . In this experiment, you are not expected to indicate uncertainties of your
experimental results.
2. Use only the pen provided
3. Use only the front side of the answer sheets and blank work sheets furnished.
4. Express your answers primarily in equations, numbers and figures. Use as little
text as possible. Underline your final result if it is a numeric number.
5. The blank work sheets are to be used for equation derivations, measurement data
and whatever else you consider is required for the solution of the question and
that you wish to be marked. If you use some blank sheets of paper for notes that
you do not wish to be marked, put a large cross through the whole sheet and do
not include them in your numbering.
6. It is absolutely essential that you enter in the boxes at the top of answer sheets,
work sheets and graph papers: your Name, your Country and your Student No. In
addition, on the blank work sheets you should enter the Question Part No., the
progressive Page No. of each sheet and the Total Pages of blank work sheets that
you have used and wish to be marked.
7. At the end of 2_ hours for your first half of the competition, please staple your
answer sheets and graphs in order, before proceeding to the second Part of the
competition.
1

Part A
Measurement of Reflectance and Determination of Refractive Index
I. Background
The aim of this experiment is to measure the angular dependence of reflection of
polarized light and to determine the refractive index of a semiconductor wafer.
When light falls on a semiconductor surface it will be partially reflected, partially
transmitted and partially absorbed. The relative amount of light power reflected is called
reflectance R, which is defined as the ratio of the reflected power Ir over the incident
power Ii :
R=

Ir
Ii

(1)

The incident light may be resolved into two polarized components. One component is
polarized parallel (labeled as p-polarization) to the plane of incidence, and the other
polarized perpendicular (labeled as s-polarization) to the plane of incidence. For the red
laser wavelength used in this study, the effects of absorption at the semiconductor surface
are negligible. Under such condition, for an incident light from air onto a material the
reflectance Rp and R s, respectively for the p and s components, are given by the Fresnel
equations:
Rp =

ncos i cos t
cos t + ncos i

(2)

Rs =

cos i ncos t
cos i + ncos t

(3)

where n is the refractive index of the material, i is the angle of incidence, r the angle of
reflection, and t the angle of transmission (or refraction), as shown in Fig. 1.

Ii
air
Sample of
refractive
index n

Ir

i r
t

It

Fig. 1
2

Direct measurements of Rs and Rp with i = 0 are practically not feasible. However, the
Fresnel equations allow the calculation of n from Rs and R p obtained for any oblique
incident angle. A possible schematic diagram for the measurement is shown in Fig. 2
below, and a photograph of the suggested setup is shown in Fig.3.

Sample
Glass
Sample Block
Sample on
Rotary Stage

Diode Laser
on Rotary Stage

i
r
Polarizer P
On Rotary Stage

Light
Detector
Fig. 2

Laser Power
Meter

Press this Button


for measurement

Fig. 3

Note: The reference line for angular setting is on the upper left corner of the rotary
stage

II. Equipment and Materials


(1)

A diode laser mounted on rotary stage together with 3V battery supply.


The laser emits a wavelength 650 nm.

Caution: Do not look into the laser beam


Watch out for strayed or scattered laser light
Keep the glass and sample surfaces clean with tissues provided

(2)

A linear polarizer of diameter 20 mm mounted on a rotary stage.


(Note: The angular setting of 0o of the rotary stage has no bearing
on the polarization axis of the polarizer.)

(3)

A glass plate (refractive index 1.57 for light of wavelength 650 nm) and a
semiconductor wafer, fixed on the opposite sides of a rectangular sample
block.

(4)

Rotary optical stage with clamp for the sample block

(5)

A digital laser power meter, with light detector head mounted on a stand that
can be revolved about the sample.

(6)

One optical bench

(7)

Graph paper for reflected laser power vs angular setting of the rotary stage for
polarizer (2 sheets)

(8)

Graph paper for reflectance vs incident angle (4 sheets)

(9)

Light-shielding board

(10)

A torch light and a flexible ruler

III. Experiments and Calculations

(1): Determine the plane of polarization of the incident laser light.

(1 point)

The diode laser emits partially linearly polarized light at 650 nm. For best results in your
measurements to be performed, the polarization axis of the polarizer should be aligned
with the strongest linear component of the laser light.

In order to obtain Rs or Rp, one needs to determine the orientation of the polarization axis
of the polarizer in order to produce polarized light parallel or perpendicular to the plane
of incidence. The axis of polarization of the polarizer can be inferred from the laser
power reflected from the glass sample of known refractive index 1.57.
With the optics aligned as accurately as possible,
a) Determine the relative orientation of diode laser and polarizer (difference in
degree between the angular settings of the diode laser rotary stage and the
polarizer rotary stage) such that the polarizer is aligned with the strongest linear
component of the laser light. In the measurements that follow, treat the polarizer
and the source as a single system, rotating both together as necessary.
b) Mount the glass sample on the rotary stage at the Brewster angle of incidence.
Measure and plot the reflected laser power vs the angular setting (in degrees) of
the polarizer. Hence determine the orientation of the polarization axis of the
polarizer.
Note: You will have to press the button (as indicated in Fig. 3) on the laser power meter
each time to take a reading.

(2): Measure the reflectance Rp and Rs of the semiconductor wafer

(5 points)

Mount the sample block on the rotary stage so that the reflecting plane of the
semiconductor wafer can be rotated about a vertical axis on the path of the incident light.
With the optics aligned as accurately as possible,
a) Set the orientation of the incident light onto the semiconductor wafer such that it
is polarized parallel to the plane of incidence.
Measure the reflected laser power and plot the values of Rp as a function of
incident angle for a widest range of incident angles permitted by the experimental
setup.
b) Change the orientation of the incident light onto the semiconductor wafer such
that it is polarized perpendicular to the plane of incidence.
Measure the reflected laser power and plot the values of Rs as a function of
incident angle for a widest range of incident angles permitted by the experimental
setup.

(3): Calculate the refractive index of the semiconductor wafer

(a) From the Fresnel equations, show that n=

(1
(1 m

)(
)(1

(4 points)

)
R )

R p 1 + Rs
Rp

From your graphs or otherwise determine the ranges of the angle of incidence
where the signs of R p are positive and negative.
(b) Using the graphs obtained in Question (2), obtain six sets of values for Rp and Rs
at angles of incidence of 20o, 30o, 40o, 50o 60o and 80o.
Calculate six values of the refractive index n of the semiconductor wafer using
these six sets of values. Compute the mean value for n and estimate its standard
deviation.
(c) Using the graphs obtained in Question (2), determine R s and R p at normal
incidence by extrapolation. Hence calculate the average refractive index n of the
semiconductor wafer from the results of extrapolation.

IPhO2002

I. Determination of e/kB Through Electrolysis Process


Background Theory
The electrolysis of water is described by the reaction :
H2O 2H+ + O-2
2H+ + 2e- H2 ; O-2

1
O2 + 2e2

The reaction takes place when an electric current is supplied through a pair of
electrodes immersed in the water. Assume that both gases produced in the reaction are
ideal.
One of the gases produced by the reaction is kept in a test tube marked by arbitrary
scale. By knowing the total charge transferred and the volume of the gas in the test
tube the quantity e/kB can be determined, where e is the charge of electron and k B is
the Boltzmann constant.
For the purpose mentioned above, this experiment is divided into two parts.
Part A: Calibration of the arbitrary scale on the test tube by using a dynamic method.
This result will be used for part B
Part B: Determination of the physical quantity e/k B by means of water electrolysis
You are not obliged to carry out the two experiments ( part A and part B ) in
alphabetical order

The following physical quantities are assumed:

Acceleration of gravity, g = (9.78 0.01) ms-2


Ratio of internal vs external diameters of the test tube, = 0.82 0.01
The local values of temperature T and pressure P will be provided by the organizer.

List of tools and apparatus given for experiment (part A & B):

Insulated copper wires of three different diameters:


1. Brown of larger diameter
2. Brown of smaller diameter
3. Blue
A regulated voltage source (0-60 V, max.1A)
A plastic container and a bottle of water.
A block of brass with plastic clamp to keep the electrode in place without
damaging the insulation of the wire.
A digital stopwatch.
A multimeter (beware of its proper procedure).
A wooden test tube holder designed to hold the tube vertically.
A multipurpose pipette

IPhO2002
A vertical stand.
A bottle of white correction fluid for marking.
A cutter
A pair of scissors
A roll of cellotape
A steel ball
A piece of stainless steel plate to be used as electrode.
A test tube with scales.
Graph papers.
Note that all scales marked on the graph papers and the apparatus for the experiments
(e.g. the test tube) are of the same scale unit, but not calibrated in millimeter.

EXPERIMENT
Part A: Calibration of the arbitrary scale on the test tube
Determine a dynamic method capable of translating the arbitrary length scale to a
known scale available.
Write down an expression that relates the measurable quantities from your
experiment in terms of the scale printed on the test tube, and sketch the
experiment set up.
Collect and analyze the data from your experiment for the determination and
calibration of the unknown length scale.
Part B: Determination of physical quantity e/kB
Set up the electrolysis experiment with a proper arrangement of the test tube in
order to trap one of the gases produced during the reaction.
Derive an equation relating the quantities: time t, current , and water level
difference h, measured in the experiment.
Collect and analyze the data from your experiment. For simplicity, you may
assume that the gas pressure inside the tube remains constant throughout the
experiment.
Determine the value of e/k B.

IPhO2002

Country

S t u d e n t N o . Experiment No. Page No. Total P ages

ANSWER FORM
PART A
1. State the method of your choice and sketch the experimental set up of
the method: [0.5 pts]

2. Write down the expression relating the measurable quantities in your


chosen method: [0.5 pts]. State all the approximations used in
obtaining this expression [1.0 pts].

3. Collect and organize the data in the following orders : physical


quantities, values, units [1.0 pts]

4. Indicate the quality of the calibration by showing the plot relating two
independently measured quantities and mark the range of validity. [0.5
pts]

5. Determine the smallest unit of the arbitrary scale in term of mm and its
estimated error induced in the measurements. [1.5 pts]

IPhO2002

Country

Student No.

Experiment No. Page No. Total Pages

PART B
1. Sketch of the experimental set up. [1.0 pts]

2. Derive the following expression:


I t =

e 2 P (r 2 )
h
kB
T

[1.5 pts]

3. Collect and organize the data in the following format : physical


quantities (value, units) [1.0 pts]

4. Determine the value of e/kB and its estimated error [1.5 pts]

4th Asian Physics Olympiad

Experimental Competition

25 April 2003

Experimental Competition
I.

Determination of Capacitance

Background
It is known that capacitors play a significant role in the electrical circuits. There
are several methods of measurements of the capacitance of a capacitor. In this
experiment you are required to perform the experiment in order to determine the
capacitance of an AC capacitor using a simple electrical circuitry.
In Fig. 1.1 (a), a capacitor of capacitance C and a resistor of resistance R are
connected in series to the alternating voltage source of mains frequency. The electrical
power which is dissipated at the resistor R depends on the values of 0 , C, R and
frequency of the mains f. Graphical analysis of this relationship can be used to determine
C.
C
D

C
R

o sin t

220 VAC
50 Hz
E

Fig. 1.1 (a): AC Circuit for


determination of capacitance C

RC RB RA

Fig. 1.1 (b): A schematic diagram of


the equipment used

Materials and apparatus


1. capacitor
2. three resistors of known values with 5% errors ( RA = 680 , RB = 1500 and
RC = 3300 ) as shown in figure 1.1 (b).
3. step-down isolation transformer for alternating voltage source of f = 50 Hz
4. digital voltmeter
5. electrical connectors
6. linear graph papers

Warning:

The digital multimeter in this experiment will be used for


~
) across R only. Do not use it
measuring the rms voltage ( V
to measure in other modes.

4th Asian Physics Olympiad

Experimental Competition

25 April 2003

Instructions
a) Derive the expression for the average power dissipation P in resistor R in terms of
(1 point)
o , R, C and .
b) Deduce the condition for which P is a maximum.

(1 point)

c) Convert the dependence found in a) into a linear dependence of certain quantities


and .
(1 point)
d) Measure the root mean square (effective) voltage V across R for each of all possible
(2.5 points)
combinations of R A , R B and RC .
e) Plot P versus R and from this graph compute the value of capacitance C.

(2 points)

f) From c), draw the graph of versus and determine capacitance C.

(2 points)

g) Estimate the uncertainties in the values of C obtained in e) and f).


-----------------------

(0.5 point)

The 34th International Physics Olympiad


Taipei, Taiwan
Experimental Competition
Wednesday, August 6, 2003
Time Available : 5 hours
Please Read This First:
1. Use only the pen provided.
2. Use only the front side of the answer sheets and paper.
3. In your answers please use as little text as possible; express yourself primarily
in equations, numbers and figures. If the required result is a numerical
number, underline your final result with a wavy line.
4. Write on the blank sheets of paper the results of your measurements and
whatever else you consider is required for the solution of the question and that
you wish to be marked.
5. It is absolutely essential that you enter in the boxes at the top of each sheet of
paper used your Country and your student number [Student No.]. In addition,
on the blank sheets of paper used for each question, you should enter the
question number [Question No. : e.g. A-(1)], the progressive number of each
sheet [Page No.] and the total number of blank sheets that you have used and
wish to be marked for each question [Total No. of pages]. If you use some
blank sheets of paper for notes that you do not wish to be marked, put a large
cross through the whole sheet and do not include them in your numbering.
6. At the end of the exam please put your answer sheets and graphs in order.
7. Error bars on graphs are only needed in part A of the experiment.
8. Caution: Do not look directly into the laser beam. You can damage your
eyes!!

Apparatuses and materials


1. Available apparatuses and materials are listed in the following table:
Item

Apparatus & material

Quantity

Item

Apparatus & material

Quantity

Photodetector (PD)

Batteries

Polarizers with
Rotary mount

Battery box

90 TN-LC cell (yellow


wires) with rotary LC mount

K Optical bench

Function generator

L Partially transparent papers

Laser diode (LD)

M Ruler

Multimeters

Parallel LC cell
(orange wires)

O Scissors

Variable resistor

10

White tape *
(for marking on apparatus)

Graph papers

* Do not mark directly on apparatus. When needed, stick a piece of the white tape on the parts
and mark on the white tape.
HI
LO

ON
OFF

D
K

G
F

F
Fig. 1

2. Instructions for the multimeter:


DC/AC switch for selecting DC or AC measurement.
Use the V and the COM inlets for voltage and resistance measurements.
Use the mA and the COM inlets for small current measurements. The display then
shows the current in milliamperes.
Use the function dial to select the proper function and measuring range.

V is for

voltage measurement, A is for current measurement and is for resistance


measurement.

Voltage
range

Current
range

DC/AC
switch

Function
dial

Resistance
range

Current
Current
port (mA)
port (mA)

Voltage &
Resistance
Voltage &
port
Resistance
port

Common
Common
port
port

Fig. 2
3

3. Instructions for the Function Generator:


The power button may be pressed for ON and pressed again for OFF
Select the frequencies range, and press the proper button.
The frequency is shown on the digital display.
Use the coarse and the fine frequency adjusting knobs to tune the proper frequency.
Select the square-wave form by pressing the left most waveform button.
Use the amplitude-adjusting knob to vary the output voltage.

Frequency
display

Frequency
Coarse
Adjusting
knob

Power
On/Off
button

Frequency
Range
buttons

Frequency
Fine
Adjusting
knob

Output
Amplitude
Adjusting
knob
Fig.3

Waveform
buttons

Output
connector

Part A: Optical Properties of Laser Diode


I. Introduction
1. Laser Diode
The light source in this experiment is a laser diode which emits laser light with
wavelength 650 nm. When the current of the laser diode (LD) is greater than the threshold
current, the laser diode can emit monochromatic, partially polarized and coherent light.
When the current in the laser diode is less than the threshold, the emitted light intensity is
very small. At above the threshold current, the light intensity increases dramatically with
the current and keeps a linear relationship with the current. If the current increases further,
then the increasing rate of the intensity with respect to the current becomes smaller because
of the higher temperature of the laser diode. Therefore, the optimal operating current range
for the laser diode is the region where the intensity is linear with the current. In general, the
threshold current Ith is defined as the intersection point of the current axis with the
extrapolation line of the linear region.
Caution: Do not look directly into the laser beam. You can damage your eyes!!
2. Photodetector
The photodetector used in this experiment consists of a photodiode and a current
amplifier. When an external bias voltage is applied on the photodiode, the photocurrent is
generated by the light incident upon the diode. Under the condition of a constant
temperature and monochromatic incident light, the photocurrent is proportional to the light
intensity. On the other hand, the current amplifier is utilized to transfer the photocurrent
into an output voltage. There are two transfer ratios in our photodetector high and low
gains. In our experiment, only the low gain is used. However, because of the limitation of
the photodiode itself, the output voltage would go into saturation at about 8 Volts if the
light intensity is too high and the photodiode cannot operate properly any more. Hence the
appropriate operating range of the photodetector is when the output voltage is indeed
proportional to the light intensity. If the light intensity is too high so that the photodiode
reaches the saturation, the reading of the photodetector can not correctly represent the
incident light intensity.

II. Experiments and procedures


Characteristics of the laser diode & the photodetector
In order to make sure the experiments are done successfully, the optical alignment of
5

light rays between different parts of an experimental setup is crucial. Also the light source
and the detector should be operated at proper condition. Part A is related to these questions
and the question of the degree of polarization.
1. Mount the laser diode and photodetector in a horizontal line on the optical bench, as shown
in Fig. 4. Connect the variable resistor, battery set, ampere meter, voltage meter, laser
diode and photodetector according to Fig. 5. Adjust the variable resistor so that the current
passing through LD is around 25 mA and the laser diode emits laser light properly. Choose
the low gain for the photodetector. Align the laser diode and the photodetector to make the
laser light level at the small hole on the detector box and the reading of the photodetector
reaches a maximum value.
Caution: Do not let the black and the red leads of the battery contact with each other
to avoid short circuit.

LD

PD
~60cm

Fig. 4

Optical setup (LDlaser diode; PDphotodetector).

A
100

I
LD

3V

PD
V

Fig. 5

Equivalent circuit for the connection of the laser diode.

2. Use the output voltage of the photodetector to represent the laser light intensity J. Adjust
the variable resistor to make the current I of the laser diode varying from zero to a
maximum value and measure the J as I increases. Be sure to choose appropriate current
increment in the measurement.

Question A-(1) (1.5 point)


Measure, tabulate, and plot the J vs. I curve.
Question A-(2) (3.5 points)
Estimate the maximum current Im with uncertainty in the linear region of the
J vs. I curve. Mark the linear region on the J - I curve figure by using arrows () and
determine the threshold current Ith with uncertainty.
3. Choose the current of the laser diode as Ith + 2(Im Ith)/3 to make sure the laser diode and
photodetector are operated well.
4. To prepare for the part B experiment: Mount a polarizer on the optical bench close to the
laser diode as shown in Fig. 6. Make sure the laser beam passing through the center portion
of the polarizer. Adjust the polarizer so that the incident laser beam is perpendicular to the
plane of the polarizer. (Hint: You can insert a piece of partially transparent paper as a test
screen to check if the incident and reflected light spots coincide with each other.)

LD

Fig. 6

PD

Alignment of the polarizer (Ppolarizer).

5. Keep the current of the laser diode unchanged, mount a second piece of polarizer on the
optical bench and make sure proper alignment is accomplished, i.e., set up the source,
detector and polarizers in a straight line and make sure each polarizer plane is
perpendicular to the light beam.

Part B Optical Properties of Nematic Liquid Crystal :


Electro-optical switching characteristic of 90o TN LC cell
I. Introduction
1. Liquid Crystal
Liquid crystal (LC) is a state of matter that is intermediate between the crystalline
solid and the amorphous liquid. The nematic LCs are organic compounds consist of
long-shaped needle-like molecules. The orientation of the molecules can be easily aligned
and controlled by applying an electrical field. Uniform or well prescribed orientation of the
LC molecules is required in most LC devices. The structure of the LC cell used in this
experiment is shown in Fig 7. Rubbing the polyimide film can produce a well-aligned
preferred orientation for LC molecules on substrate surfaces, thus due to the molecular
interaction the whole slab of LC can achieve uniform molecular orientation. The local
molecular orientation is called the director of LC at that point.
The LC cell exhibits the so-called double refraction phenomenon with two principal
refractive indices. When light propagates along the direction of the director, all polarization
components travel with the same speed vo = c / n0 , where no is called the ordinary index of
refraction. This propagation direction (direction of the director) is called the optic axis of
the LC cell. When a light beam propagates in the direction perpendicular to the optic axis,
in general, there are two speeds of propagation. The electric field of the light polarized
perpendicular (or parallel) to the optic axis travels with the speed of vo = c / n0 (or
v e = c / ne , where ne is called the extraordinary index of refraction). The birefringence
(optical anisotropy) is defined as the difference between the extraordinary and the ordinary
indices of refraction n ne no .
PI alignment Film

Glass Substrate

Glass Substrate

ITO Electrode
Fig. 7

LC Layer

ITO Electrode

LC cell structure
8

2. 90o Twisted Nematic LC Cell


In the 90o twisted nematic (TN) cell shown in Fig. 8, the LC director of the back
surface is twisted 90o with respect to the front surface. The front local director is set
parallel to the transmission axis of the polarizer. An incident unpolarized light is converted
into a linearly polarized light by the front polarizer.

LC molecules

Light
Propagation
Direction

Polarizer PI

PI Analyzer

Fig. 8 90o TN LC cell

When a linearly polarized light traverses through a 90o TN cell, its polarization
follows the twist of the LC directors (polarized light sees ne only) so that the output beam
remains linearly polarized except for that its polarization axis is rotated by 90o (its called
the polarizing rotary effect by ne; similarly we can also find polarizing rotary effect by no).
Thus, for a normally black (NB) mode using a 90o TN cell, the analyzers (a second
polarizer) transmission axis is set to be parallel to the polarizers transmission axis, as
shown in Fig. 9. However, when the applied voltage V across the LC cell exceeds a critical
value Vc, the director of LC molecules tends to align along the direction of applied external
electrical field which is in the direction of the propagation of light. Hence, the polarization
guiding effect of the LC cell is gradually diminishing and the light leaks through the
analyzer. Its electro-optical switching slope is defined as (V90V10)/V10, where V10 and
V90 are the applied voltages enabling output light signal reaches up to 10% and 90% of its
maximum light intensity, respectively.

Polarizer

90o TN-LC

Analyzer

NB operation
E
Z

Fig. 9

NB mode operation of a 90o TN cell

II. Experiments and procedures


1. Setup a NB 90o TN LC mode between two polarizers with parallel transmission axes and
apply 100 Hz square wave voltage using a function generator onto the ITO portions of two
glass substrates and vary the applied voltage (Vrms) from 0 to 7.2 Volts.
* In the crucial turning points, take more data if necessary.
Question B-(1) (5.0 points)
Measure, tabulate, and plot the electro-optical switching curve (J vs. Vrms curve) of the
NB 90o TN LC, and find its switching slope , where is defined as (V90V10)/V10.

Question B-(2) (2.5 points)


Determine the critical voltage Vc of this NB 90o TN LC cell. Show explicitly with graph
how you determine the value Vc.
Hint:* When the external applied voltage exceeds the critical voltage, the light
transmission increases rapidly and abruptly.

10

Part C Optical Properties of Nematic Liquid Crystal :


Electro-optical switching characteristic of parallel aligned LC cell
I. Introduction
Homogeneous Parallel-aligned LC Cell
For a parallel-aligned LC cell, the directors in the front and back substrates are
parallel with each other, as shown in Fig. 10. When a linearly polarized light impinges on a
parallel-aligned cell with its polarization parallel to the LC director (rubbing direction), a
pure phase modulation is achieved because the light behaves only as an extraordinary ray.

LC molecule
Glass substrate (ITO+PI)
Fig. 10

Homogeneous parallel aligned LC

On the other hand, if a linearly polarized light is normally incident onto a parallel
aligned cell but with its polarization making = 45o relative to the direction of the
aligned LC directors (Fig. 11), then phase retardation occurs due to the different
propagating speed of the extraordinary and ordinary rays in the LC medium. In this

= 45o configuration, when the two polarizers are parallel, the normalized transmission
of a parallel aligned LC cell is given by
2
T || = cos

The phase retardation is expressed as

= 2 d n (V , ) /
where d is the LC layer thickness, is the wavelength of light in air, V is the root mean
square of applied AC voltage, and n, a function of and V, is the LC birefringence. It
should be also noted that, at V = 0, n (= neno) has its maximum value, so does . Also
n decreases as V increases.
In the general case, we have
T// = 1 sin 2 2 sin 2

11

T = sin 2 2 sin 2

where

//

and represent that the transmission axis of analyzer is parallel and

perpendicular to that of the polarizer, respectively.

II. Experiments and procedures


1. Replace NB 90o TN LC cell with parallel-aligned LC cell.
2. Set up = 45o configuration at V = 0 as shown in Fig. 11. Let the analyzers transmission
axis perpendicular to that of the polarizer, then rotate the parallel-aligned LC cell until the
intensity of the transmitted light reaches the maximum value ( T ). This procedure
establishes the = 45o configuration. Take down T value, then, measure the intensity of
the transmitted light ( T// ) of the same LC cell at the analyzers transmission axis parallel to
that of the polarizer (also at V = 0).
For T//

Homogeneous
Parallel Aligned
LC cell
Fig

L
P

Analyzer

For T

VAC
Polarizer
Fig. 11

Schematic diagram of experimental setup


(The arrow L is the alignment direction.)

Question C-(1) (2.5 points)


Assume that the wavelength of laser light 650 nm, LC layer thickness 7.7 m, and
approximate value of n 0. 25 are known. From the experimental data T and
Tobtained above, calculate the accurate value of the phase retardation and accurate
value of birefringence n of this LC cell at V=0.

12

3. Similar to the above experiment (1), in the = 45o configuration, apply 100 Hz square
wave voltage using a function generator onto the ITO portions of two glass substrates, vary
the applied voltage (Vrms) from 0 to 7 Volts and measure the electro-optical switching
curve (T) at the analyzers transmission axis parallel to the polarizers transmission axis.
(Hint: Measuring the T switching curve is helpful to increase the data accuracy of the
above T measurement; the data of T are not needed in the following questions. )
* In the crucial turning points, take more data if necessary (especially in the range of
0.5-4.0 Volts).
Question C-(2) (3.0 points)
Measure, tabulate, and plot the electro-optical switching curve for T of this parallel
aligned LC cell in the = 45o configuration.
Question C-(3) (2.0 points)
From the electro-optical switching data, find the value of the external applied voltage V.
Hint: * V is the applied voltage which enables the phase retardation of this anisotropic
LC cell become (or 180o).
* Remember that n is a function of applied voltage, and n decreases as V
increases.
* Interpolation is probably needed when you determine the accurate value of this V.

13

Experimental competition- Problem No.1

Hall effect and magnetoresistivity effect


APPARATUS AND MATERIALS
1. Three digital multimeters.
2. A Hall sensor with four pins MNPQ (M in black wire, N yellow wire, P red wire, Q
green wire), fixed on a printed circuit, a pair of conductors leading to M, N; another pair
of conductors leading to P, Q.
3. A permanent magnet in the shape of a disk, of radius r = 14 mm, of thickness t = 4 mm.
The magnetization is perpendicular to the surface of the disk. The value B0 (in Tesla) of
the magnetic field at the surface of the magnet is written on the magnet's surface.
During the experiment, keep the magnet far away from the Hall sensor whenever you
do not use it.
4. A coil of N turns is wound on a core having the shape of a toroid, made of a
ferromagnetic material. The average diameter of the core is = 25 mm. The toroid has a
gap of width d=3 mm.
5. A box with two independent 1.5 V dry cells. The cell connected in series to a 10 k
variable resistor, called battery 1, is used to supply the current to the Hall sensor. The
second cell, called battery 2, is used to supply the current to the coil only during the
measurement.
6. A protractor with a small hole at its center.
7. A piece of plexiglass with a small needle fixed on it.
8. A holder for the printed circuit with the Hall sensor.
9. A small piece of plastic used to fix the sensor on the needle.
10. Conductors with negligible resistance.
11. Graph papers.
1

10 k
variable
resistor

2
4

Figure 1

Experiment
I. Introduction
1. The magnetoresistivity effect and the Hall effect.
Consider a conductor sample in the shape of a
parallelopiped of length a, width b and thickness c (see
a M
c

Figure 2). The current I flows along the direction of a. If


P
r
the sample is placed in a magnetic field B , the magnetic
Q
b
I
field affects the resistance R of the sample. This effect is
r
N
B
called magneto-resistivity effect (MRE). Let R be the
increase of the resistance R of the sample, R0 - the value of
Figure 2
the resistance in the absence of a magnetic field, then the
magnitude of the MRE is defined by the ratio R/R0.
Assume that the applied magnetic field is uniform and the magnetic induction vector

r
B is parallel to the top face of the sample as shown in Figure 2. If the charge carriers in

the sample are electrons, the Lorentz force will bend them upward, and the top face of the
sample will be charged negatively. This effect is called the Hall effect. The voltage
appearing between electrodes M (on the top face) and N (on the bottom face) is called the
Hall voltage. This can be measured by use of a voltmeter.
The potential difference measured between the electrodes M and N is given by
U MN = U H + VMN
(1)
where, UH is the Hall voltage, VMN is the potential difference in the absence of a magnetic
field due to some undesired effects (the electrodes M and N being not exactly opposite to
each other, etc...).
Normally, the Hall voltage UH is proportional to IB.sin , and the magnitude of the

MRE is proportional to B2sin2, where is the angle between vector B and the current
direction. But when the sample has a non regular shape, the dependence of UH and R/R
on B sin may be more complicated.
The Hall effect is used to fabricate a device for measuring the magnetic field. This
device is called the Hall sensor. For Hall sensor, the expression of UH is given by:
UH = .I.B.sin
(2)
where is, by definition, the sensitivity of the Hall sensor.
II. The measuring sample:
The measuring sample in this experiment is a commercial Hall sensor. It consists of a
small thin semiconductor plate covered with plastic, with 4 ohmic electrodes, leading to
the pins M, N, P, Q (see Figure 3). It is used in this experiment to study both the MRE
and the Hall effect.
M
Q

P
N
Figure 3

Place the sensor in the magnetic field and use an ohmmeter to mesure the resistance
between pins M and N, we can deduce the magnitude of the MRE. Set a current (I 1mA)
flowing from P to Q, we can study the Hall effect by measuring the voltage between M
and N with a milivoltmeter.

III. Experiment
1. Determination of the sensitivity of the Hall sensor
Set the current through the sensor I 1 mA. Keep the distance between the sensor
and the centre of the surface of the magnet y = 2cm. Adjust the orientation of the magnet
to obtain maximal value of the Hall voltage. Measure the Hall voltage with some values of
I and determine the sensitivity of the Hall sensor.
For a magnet having the shape of a disk of radius r, thickness t , the magnetic field at a
point situated on its axis at a distance y from the center of the disk surface with y>>t is
given by the expression

1
B ( y )= B0
2

y+t

( y+t )2 +r 2

2
2
y +r

(1)

where B0 is the magnetic induction at the surface of the magnet. The value of B0 is given on
the surface of the magnet.
[2.0 pts]

2. Study of the dependence of UH on angle between B and the current direction.


Set the current through the sensor I 1 mA. Keep the distance between the sensor and
the centre of the surface of the magnet y = 2 cm. Put the magnet on the protractor so that
the plane of the magnet is perpendicular to the line connecting the sensor and centre of the
magnet.
a. Draw a sketch of the experimental arrangement.
b. Tabulate the values of UH for in the range of -90o90o.
c.Verify the proportionality between UH and sin by using a graph plotted in an
appropriate way.
[2.5 pts]

3. Study of the dependence of R/R on B, for B perpendicular to the sample plane.


The MRE is significant only at sufficiently strong magnetic field. So it is
recommended to use a magnetic field as strong as possible.
a. Draw a sketch of the experimental arrangement and explain the principle of the
measurements.
b. Perform measurements and tabulate the data.
k
c. Assume that R/R B , determine the value of k by using a graph plotted in an
appropriate way. Estimate the maximal deviation of the obtained value of k.
[4.0 pts]
4. Determination of the relative permeability of the ferromagnetic materials of the
core of the toroidal coil
Determine the relative permeability of the core material at the measured current
intensity I by following this guidance step by step:
Put the Hall sensor into the gap on the core.
Connect the coil and an ammeter to battery 2. Use only the inputs COM and
20A of the ammeter in this case.
Measure the current I in the coil and the magnetic field B in the gap.
Calculate the value of
You can use the following relation:
B .( d )
[1.5 pts]
+ B .d = 4 .107 .N .I

Experimental Competition / Question

Page 1/12

35th International Physics Olympiad


Pohang, Korea
15 ~ 23 July 2004

Experimental Competition
Monday, 19 July 2004

Please, first read the following instruction carefully:


1.
2.
3.
4.
5.
6.

7.

8.
9.

The time available is 5 hours.


Use only the pen provided.
Use only the front side of the writing sheets. Write only inside the boxed area.
In addition to the blank writing sheets, there are Answer Forms where you must
summarize the results you have obtained.
Write on the blank writing sheets the results of your measurements and whatever
else you consider is required for the solution to the question. Please, use as little text
as possible; express yourself primarily in equations, numbers, figures, and plots.
In the boxes at the top of each sheet of paper write down your country code
(Country Code) and student number (Student Code). In addition, on each blank
writing sheets, write down the progressive number of each sheet (Page Number)
and the total number of writing sheets used (Total Number of Pages). If you use
some blank writing sheets for notes that you do not wish to be marked, put a large X
across the entire sheet and do not include it in your numbering.
At the end of the experiment, arrange all sheets in the following order:
Answer forms (top)
used writing sheets in order
the sheets you do not wish to be marked
unused writing sheets
the printed question (bottom)
It is not necessary to specify the error range of your values. However, their
deviations from the actual values will determine your mark.
Place the papers inside the envelope and leave everything on your desk. You are not
allowed to take any sheet of paper or any material used in the experiment out of
the room.

Experimental Competition / Question

Page 2/12

Apparatus and materials


1. List of available apparatus and materials
Name

Quantity

Name

Quantity

Photogate timer

Philips screw driver

Photogate

Weight with a string

Connecting cable

Electronic balance

Mechanical black box


(Black cylinder)

Stand with a ruler

Rotation stage

U-shaped support

Rubber pad

C-clamp

Pulley

Ruler (0.50 m, 0.15 m)

Pin

Vernier calipers

U-shaped plate

Scissors

Screw

Thread

Allen (hexagonal, Lshaped) wrench

Spares (string, thread,


pin, screw, Allen wrench)

1 each

E
L

C
H

A
B

I J

F
G

P
M

Experimental Competition / Question

Page 3/12

2. Instruction for the Photogate Timer


The Photogate consists of an infrared LED and a photodetector. By connecting the
Photogate to the Photogate Timer, you can measure the time duration related to the
blocking of the infrared light reaching the sensor.
Be sure that the Photogate is connected to the Photogate Timer. Turn on the
power by pushing the button labelled POWER.
To measure the time duration of a single blocking event, push the button
labelled GATE. Use this GATE mode for speed measurements.
To measure the time interval between two or three successive blocking events,
push the corresponding PERIOD. Use this PERIOD mode for oscillation
measurements.
If DELAY button is pushed in, the Photogate Timer displays the result of each
measurement for 5 seconds and then resets itself.
If DELAY button is pushed out, the Photogate Timer displays the result of the
previous measurement until the next measurement is completed.
After any change of button position, press the RESET button once to activate
the mode change.
Caution: Do not look directly into the Photogate. The invisible infrared light may be
harmful to your eyes.

Photogate, Photogate Timer, and connection cable

Experimental Competition / Question

Page 4/12

3. Instruction for the Electronic Balance


Adjust the bottom legs to set the balance stable. (Although there is a level
indicator, setting the balance in a completely horizontal position is not
necessary.)
Without putting anything on the balance, turn it on by pressing the On/Off
button.
Place an object on the round weighing pan. Its mass will be displayed in grams.
If there is nothing on the weighing pan, the balance will be turned off
automatically in about 25 seconds.

Balance

Experimental Competition / Question

Page 5/12

4. Instruction for the Rotation Stage


Adjust the bottom legs to set the rotation stage stable on a rubber pad in a near
horizontal position.
With a U-shaped plate and two screws, mount the Mechanical Black Box
(black cylinder) on the top of the rotating stub. Use Allen (hexagonal, L-shaped)
wrench to tighten the screws.
The string attached to the weight is to be fixed to the screw on the side of the
rotating stub. Use the Philips screw driver.
Caution: Do not look too closely at the Mechanical Black Box while it is rotating.
Your eyes may get hurt.

Mechanical Black Box and rotation stage

Rotating stub

Weight with a string

Experimental Competition / Question

Page 6/12

Mechanical Black Box


[Question] Find the mass of the ball and the spring constants of two
springs in the Mechanical Black Box.
General Information on the Mechanical Black Box
The Mechanical Black Box (MBB) consists of a solid ball attached to two
springs in a black cylindrical tube as shown in Fig. 1. The two springs are fashioned
from the same tightly wound spring with different number of turns. The masses and the
lengths of the springs when they are not extended can be ignored. The tube is
homogeneous and sealed with two identical end caps. The part of the end caps plugged
into the tube is 5 mm long. The radius of the ball is 11 mm and the inner diameter of the
tube is 23 mm. The gravitational acceleration is given as g = 9.8 m/s2. There is a finite
friction between the ball and the inner walls of the tube.
lCM

Fig. 1 Mechanical Black Box (not to scale)


The purpose of this experiment is to find out the mass m of the ball and the spring
constants k1 and k2 of the springs without opening the MBB. The difficult aspect of this
problem is that any single experiment cannot provide the mass m or the position l of the
ball because the two quantities are interconnected. Here, l is the distance between the
centers of the tube and the ball when the MBB lies horizontally in equilibrium when the
friction is zero.
The symbols listed below should be used to represent the physical quantities of
interest. If you need to use other physical quantities, use symbols different from those
already assigned below to avoid confusion.

Experimental Competition / Question

Page 7/12

Assigned Physical Symbols


Mass of the ball: m
Radius of the ball: r (= 11 mm)
Mass of the MBB excluding the ball: M
Length of the black tube: L
Length of each end cap extending into the tube: (= 5.0 mm)
Distance from the center-of-mass of the MBB to the center of the tube: lCM
Distance between the center of the ball and the center of the tube: x (or l at
equilibrium when the MBB is horizontal)
Gravitational acceleration: g (= 9.8 m/s2)
Mass of the weight attached to a string: mo
Speed of the weight: v
Downward displacement of the weight: h
Radius of the rotating stub where the string is to be wound: R
Moments of inertia: I, Io, I1, I2, and so on
Angular velocity and angular frequencies: , 1, 2, and so on
Periods of oscillation: T1, T2
Effective total spring constant: k
Spring constants of the two springs: k1, k2
Number of turns of the springs: N1, N2

Caution: Do not try to open the MBB. If you open it, you will be disqualified
and your mark in the Experimental Competition will be zero.
Caution: Do not shake violently nor drop the MBB. The ball may be detached
from the springs. If your MBB seems faulty, report to the proctors
immediately. It will be replaced only once without affecting your
mark. Any further replacement will cut down your mark by 0.5
points each time.

Experimental Competition / Question

Page 8/12

PART-A Product of the mass and the position of the ball (m l ) (4.0 points)
l is the position of the center of the ball relative to that of the tube when the MBB
lies horizontally in equilibrium as in Fig. 1. Find the value of the product of the mass m
and the position l of the ball experimentally. You will need this to determine the value of
m in PART-B.
1. Suggest and justify, by using equations, a method allowing to obtain m l. (2.0
points)
2. Experimentally determine the value of m l. (2.0 points)

Experimental Competition / Question

Page 9/12

PART-B The mass m of the ball (10.0 points)


Figure 2 shows the MBB fixed horizontally on the rotating stub and a weight
attached to one end of a string whose other end is wound on the rotating stub. When the
weight falls, the string unwinds, and the MBB rotates. By combining the equation
pertinent to this experiment with the one obtained in PART-A, you can find an equation
for m.
Between the ball and the inner walls of the cylindrical tube acts a frictional force.
The physical mechanisms of the friction and the slipping of the ball under the rotational
motion are complicated. To simplify the analysis, you may ignore the energy dissipation
due to kinetic friction.

Fig. 2 Rotation of the Mechanical Black Box (not to scale)


The angular velocity of the MBB can be obtained
from the speed v of the weight passing through the
Photogate. x is the position of the ball relative to the
rotation axis, and d is the length of the weight.

Experimental Competition / Question

Page 10/12

1. Measure the speed of the weight v for various values of downward displacement
h of the weight. It is recommended to scan the whole range from h = 1.0 10-2 m
to 4.0 10-1 m by measuring v just once at each h with an interval of 1.0 10-2
~2.0 10-2 m. Plot the data on graph paper in a form that is suitable to find the
value of m. After you get a general idea of the relation between v and h, you may
repeat the measurement or add some data points, if necessary. When the MBB
rotates slowly, the ball does not slip from its static equilibrium position because
of the friction between the ball and the tube. When the MBB rotates sufficiently
fast, the ball hits and actually stays at the end cap of the tube because the springs
are weak. Identify the slow rotation region and the fast rotation region on the
graph. (4.0 points)
2. Show your measurements are consistent with the fact that h is proportional to v2
( h = C v2 ) in the slow rotation region. Show from your measurements that h =
A v2+B in the fast rotation region. (1.0 points)
3. The moment of inertia of a ball of radius r and mass m about the axis passing
through its center is 2mr2/5. If the ball is displaced a distance a perpendicular to
the axis, the moment of inertia increases by ma2. Use the symbol I to represent
the total moment of inertia of all the rotating bodies excluding the ball. Relate
the coefficient C to the parameters of the MBB such as m, l, etc. (1.0 points)
4. Relate the coefficients A and B to the parameters of the MBB such as m, l, etc.
(1.0 points)
5. Determine the value of m from your measurements and the results obtained in
PART-A. (3.0 points)

Experimental Competition / Question

Page 11/12

PART-C The spring constants k1 and k2 (6.0 points)


In this part, you need to perform small oscillation experiments using the MBB as a
rigid pendulum. There are two small holes at each end of the MBB. Two thin pins
inserted into the holes can be used as the pivot of small oscillation. The U-shaped
support is to be clamped to the stand and used to support the pivot. Note that the angular
frequency of small oscillation is given as = [torque/(moment of inertia
angle)]1/2. Here, the torque and the moment of inertia are with respect to the pivot.
Similarly to PART-B, consider two experimental conditions, shown in Fig. 3, to avoid
the unknown moment of inertia Io of the MBB excluding the ball.

(1)
Fig. 3

(2)

Oscillation of the Mechanical Black Box (not to scale)


The periods of small oscillation, T1 and T2, for two
configurations shown above can be measured using
the Photogate. Two pins and a U-shaped support are
supplied for this experiment.

Experimental Competition / Question

Page 12/12

1. Measure the periods T1 and T2 of small oscillation shown in Figs 3(1) and (2) and
write down their values, respectively. (1.0 points)
2. Explain (by using equations) why the angular frequencies 1 and 2 of small
oscillation of the configurations are different. Use the symbol Io to represent the
moment of inertia of the MBB excluding the ball for the axis perpendicular to the
MBB at the end. Use the symbol l as the displacement of the ball from the
horizontal equilibrium position. (1.0 points)
3. Evaluate l by eliminating Io from the previous results. (1.0 points)
4. By combining the results of PART-C 1~3 and PART-B, find and write down the
value of the effective total spring constant k of the two-spring system. (2.0 points)
5. Obtain the respective values of k1 and k2. Write down their values. (1.0 points)

EXPERIMENTAL COMPETITION

Thursday, April 28th, 2005

Before attempting to assemble your equipment, read the problem text


completely!
Please read this first:
1.
2.
3.
4.

5.

6.

7.

8.

The time available is 5 hours.


Use only the pen and equipments provided. You may use your own calculator.
Use only the one side of the provided sheets.
In addition to "blank" sheets where you may write freely, there is a set of Answer
sheets where you must summarize the results you have obtained. Numerical results
must be written with as many digits as appropriate; dont forget the units.
Please write on the "blank" sheets the results of all your measurements and whatever
else you deem important for the solution of the problem, that you wish to be evaluated
during the marking process. However, you should use mainly equations, numbers,
symbols, graphs, figures, and use as little text as possible.
It's absolutely imperative that you write on top of each sheet: your country (Country
code), your student code (as shown on your identification tag, Student ID), and
additionally on the "blank" sheets: the progressive number of each sheet (from 1 to N,
Page n.) and the total number (N) of "blank" sheets that you use and wish to be
evaluated (Page total); leave the Problem field blank.
The students should start with a new page for each section. It is also useful to write the
number of the section you are answering at the beginning of each such section. If you
use some sheets for notes that you do not wish to be evaluated by the marking team, just
put a large cross through the whole sheet, and dont number it.
When you have finished, turn in all sheets in proper order (answer sheets first, then
used sheets in order, the unused sheets and problem text at the bottom) and put them
all inside the envelope where you found them; then leave everything on your desk.
You are not allowed to take anything out of the room.

1/9

EXPERIMENTAL COMPETITION

1. DETERMINATION OF SHAPES BY
REFLECTION
INTRODUCTION
Direct visual observation, is a method where human beings used their eyes to identify an
object. However, not all things in life can be observed directly. For example, how can
you tell the position of a broken bone? Is it possible to look at a baby inside a pregnant
woman? How about identifying cancer cells inside a brain? All of these require a special
technique involving indirect observation.
In this experiment, you are to determine the shape of an object using indirect observation.
You will be given two closed cylindrical boxes and in each box, there will be an object
with unknown shapes. Your challenge is to reveal the object without opening the box.
The physics concepts for this experiment are simple, but creativity and some skills are
needed to solve it.
EXPERIMENT
APPARATUS
For this experiment, you will be given two sets of cylindrical boxes consisting of:
1. An object with unknown shape to be determined (it is a simple geometrical object
with either plane or cylindrical sides)
2. Closed cylindrical box with an angular scale on the top side (2a) and around its
circumference (2b).
3. A knob which you can rotate
4. A laser pointer
5. Spare batteries for the laser pointer
6. A ruler
EXPERIMENTAL METHOD
The students are to determine the shape of the object inside a closed cylindrical box. The
diameter of the cylinder can be measured by a ruler. Students are not allowed to open
the cylindrical box or break the seal to determine the shape of the object. The object is
an 8-mm thick metal with its sides polished so that it can reflect light likes a mirror. You
can rotate the object using the knob on the top part of the cylinder. This will rotate the
object in the same axis as the cylinders axis.
2/9

EXPERIMENTAL COMPETITION

The laser pointer can be switched on by rotating its position. You can adjust the position
of the light beam by rotating the laser pointer in either clockwise or anti-clockwise
direction. The reflection of the laser beam from the laser pointer can be observed along
the circumference of the closed cylinder. Measurement using the angular scale can be
used. By rotating the knob on the upper part while the laser pointer is switched on, you
will notice that as you rotate the object, the position of the reflected light from the object
will change. If the light from the laser pointer dim or the laser pointer fail to work, ask
the committee for replacement. By observing the correlation of the angular position of
the object and the reflection of the laser beam, you should be able to determine the shape
of the object.
For every object (the two objects are of different shapes):
A. Draw a graph of: reflection angle of the laser beam against the angular position
of the object (2 1 pt)
B. Determine the number of edges (sides) in each object (2 0.25 pt)
C. Use data from the graphic to sketch the shape of the object and find the angular
positions of changing sides correspond with the top angular scale (2 1.5 pts)
For the object with fewer sides only:
D. Draw rotating axis of the object and determine the distance to every sides (3 pts)
E. Determine the length of sides without error analysis; determine also the angles
between neighboring sides (1.5 pts)

You
must present your result on graph papers and try to deduce the mathematical equations to
determine the shape of the object.

3/9

EXPERIMENTAL COMPETITION

Remarks:
1. One of the objects has only plane sides and the second object has one curved side.
2. Sometimes you may get two reflections of the beam from the object.
3. In case of a curved side the determination of the radius of curvature is not required
but determination whether it is convex or concave with respect to the axis of rotation
is necessary.

4/9

EXPERIMENTAL COMPETITION

2. MAGNETIC BRAKING ON AN INCLINED


PLANE

INTRODUCTION
When a magnet moves near a non-magnetic conductor such as copper and aluminum it
experiences a dissipative force called magnetic braking force. In this experiment we will
investigate the nature of this force.
The magnetic braking force depends on:
- the strength of the magnet, determined by its magnetic moment ();
- the conductivity of the conductor (C);
- the size and geometry of both magnet and the conductor;
- the distance between the magnet and conducting surface (d); and
- the velocity of the magnet (v) relative to the conductor.
In this experiment we will investigate the magnetic braking force dependencies on the
velocity (v) and the conductor-magnet distance (d). This force can be written empirically
as:

FMB = k0 d p v n

(1)

where
k0
is an arbitrary constant that depends on , C and geometry of the
conductor and magnet which is fixed in this experiment.
d
is the distance between the center of magnet to the conductor surface
v
is the velocity of the magnet
p and n are the power factors to be determined in this experiment

5/9

EXPERIMENTAL COMPETITION

EXPERIMENT
In this experiment error analysis is required.

APPARATUS
(1) Doughnut-shaped Neodymium Iron Boron magnet.
Thickness:
tM = (6.30.1) mm
Outer diameter:
dM = (25.40.1) mm
The poles are on the flat faces as shown:
(2) Aluminum bar (2 pieces)
(3) Acrylic plate for the inclined plane with a linear
track for the magnet to roll
(4) Plastic stand
(5) Digital stop watch
(6) Ruler
(7) Graphic papers (10 pieces)

Additional information:
Local gravitational acceleration:
g = 9.8 m/s2
Mass of the magnet:
m = (21.50.5) g
North-South direction is indicated on the table.
You can read the operation manual of the stopwatch
This problem is divided into two sections:
(A) Setup and introduction
(B) Investigation of the magnetic braking force

Remarks:
Make sure that the plane is clean before your experiment

6/9

EXPERIMENTAL COMPETITION

QUESTIONS
Please provide sufficient diagrams in your answers so that your work can be
understood clearly

(A) Setup

Figure 1. Inclined plane setup without aluminum bars

Roll down the magnet along the track as shown. Choose a reasonably small
inclination angle so that it does not roll too fast.

[1] As the magnet is very strong, it may experience significant torque due to
interaction with earths magnetic field. It will twist the magnet as it rolls down
and may cause significant friction with the track. What will you do to minimize
this torque? Explain it using diagram(s).
[1.0 pt]

7/9

EXPERIMENTAL COMPETITION

Figure 2. A complete setup with aluminum bars

Place the two aluminum bars as shown in Figure 2 with distance approximately
d=5mm. Remember that the distance d is to the center of the magnet as shown in
the inset of Figure 2.
Again release the magnet and let it roll. You should observe that the magnet would
roll down much slower compared to the previous observation due to magnetic braking
force.
[2]. Provide diagram(s) of field lines and forces to describe the mechanism of
magnetic braking.
[1.0 pt]

8/9

EXPERIMENTAL COMPETITION

(B) Investigation of the magnetic braking force

The experimental setup remains the same as shown in Figure 2 with the same magnetconductor distance approximately d = 5 mm (about 2 mm gap between magnet and
conductor on each side).
[1] Keeping the distance d fixed, investigate the dependence of magnetic braking
force on velocity (v). Determine the exponent n of the speed dependence factor in
Equation 1. Provide appropriate graph to explain your result.
[4.0 pt]
Now vary the conductor-magnet distance (d) on both left and right. Choose a fixed
and reasonably small inclination angle.
[2] Investigate the dependence of the magnetic braking force on conductor-magnet
distance (d). Determine the exponent p of the distance dependence factor in
Equation 1. Provide appropriate graph to explain your result.

[4.0 pt]

9/9

36th International Physics Olympiad. Salamanca (Espaa) 2005


R.S.E.F.

PLANCKS CONSTANT IN THE LIGHT OF AN INCANDESCENT LAMP


In 1900 Planck introduced the hypothesis that light is emitted by matter in the form of quanta of energy h. In 1905
Einstein extended this idea proposing that once emitted, the energy quantum remains intact as a quantum of light (that later
received the name photon). Ordinary light is composed of an enormous number of photons on each wave front. They
remain masked in the wave, just as individual atoms are in bulk matter, but h the Plancks constant reveals their
presence. The purpose of this experiment is to measure Planck's constant.
A body not only emits, it can also absorb radiation arriving from outside.
Black body is the name given to a body that can absorb all radiation incident upon it,
for any wavelength. It is a full radiator. Referring to electromagnetic radiation, black
bodies absorb everything, reflect nothing, and emit everything. Real bodies are not
completely black; the ratio between the energy emitted by a body and the one that
would be emitted by a black body at the same temperature, is called emissivity, ,
usually depending on the wavelength.
Planck found that the power density radiated by a body at absolute
temperature T in the form of electromagnetic radiation of wavelength can be
written as

u =

c1

5 e c2 / T 1

T3
T2
T1

F-1

(1)

where c1 and c2 are constants. In this question we ask you to determine c2 experimentally, which is proportional to h.
For emission at small , far at left of the maxima in Figure F-1, it is permissible to drop the -1 from the denominator
of Eq. (1), that reduces to
u =

c1

5 e c 2 / T

(2)

The basic elements of this experimental question are sketched in Fig.


F-2.

The emitter body is the tungsten filament of an incandescent lamp A that


emits a wide range of s, and whose luminosity can be varied.
The test tube B contains a liquid filter that only transmits a thin band of
the visible spectrum around a value 0 (see Fig. F-3). More information
on the filter properties will be found in page 5.

F-2

Finally, the transmitted radiation falls upon a photo resistor C (also


known as LDR, the acronym of Light Dependent Resistor). Some
properties of the LDR will be described in page 6.

The LDR resistance R depends on its illumination, E, which is


proportional to the filament power energy density

E u0

R u0
R E

F-3

where the dimensionless parameter is a property of the LDR that will be determined in the experiment. For this setup we
finally obtain a relation between the LDR resistance R and the filament temperature T
R = c3 e c2 / 0T

(3)

that we will use in page 6. In this relation c3 is an unknown proportionality constant. By measuring R as a function on T one
can obtain c2, the objective of this experimental question.

Exp. Page 1 of 11

36th International Physics Olympiad. Salamanca (Espaa) 2005


R.S.E.F.

DESCRIPTION OF THE APPARATUS


The components of the apparatus are shown in Fig. F-4, which also includes some indications for its setup. Check
now that all the components are available, but refrain for making any manipulation on them until reading the instructions in
the next page.

6
3

5
1

10

11

12

13

F-4

EQUIPMENT:
1.
2.
3.
4.
5.
6.
7.
8.
9.
10.
11.
12.
13.

Platform. It has a disk on the top that holds a support for the LDR, a support for the tube and a support for an
electric lamp of 12 V, 0.1 A.
Protecting cover.
10 turns and 1 k potentiometer.
12 V battery.
Red and black wires with plugs at both ends to connect platform to potentiometer.
Red and black wires with plugs at one end and sockets for the battery at the other end.
Multimeter to work as ohmmeter.
Multimeter to work as voltmeter.
Multimeter to work as ammeter.
Test tube with liquid filter.
Stand for the test tube.
Grey filter.
Ruler.

An abridged set of instructions for the use of multimeters, along with information on the least squares method, is
provided in a separate page.
Exp. Page 2 of 11

36th International Physics Olympiad. Salamanca (Espaa) 2005


R.S.E.F.

SETTING UP THE EQUIPMENT

Follow these instructions:

Carefully make the electric connections as indicated in Fig. F-4, but do not plug the wires 6 to the
potentiometer.
By looking at Fig. F-5, follow the steps indicated below:

2
5

3
4

F-5

1.

Turn the potentiometer knob anticlockwise until reaching the end.

2.

Turn slowly the support for the test tube so that one of the lateral holes is in front of the lamp and the other in
front of the LDR.

3.

Bring the LDR nearer to the test tube support until making a light touch with its lateral hole. It is advisable to
orient the LDR surface as indicated in Fig. F-5.

4.

Insert the test tube into its support.

5.

Put the cover onto the platform to protect from the outside light. Be sure to keep the LDR in total darkness for
at least 10 minutes before starting the measurements of its resistance. This is a cautionary step, as the
resistance value at darkness is not reached instantaneously.

Exp. Page 3 of 11

36th International Physics Olympiad. Salamanca (Espaa) 2005


R.S.E.F.

Task 1
Draw in Answer Sheet 1 the complete electric circuits in the boxes and between the boxes, when the circuit is fully
connected. Please, take into account the indications contained in Fig. F-4 to make the drawings.

Measurement of the filament temperature


The electric resistance RB of a conducting filament can be given as
RB =

l
S

(4)

where is the resistivity of the conductor, l is the length and S the cross section of the filament.
This resistance depends on the temperature due to different causes such as:

Metal resistivity increases with temperature. For tungsten and for temperatures in the range 300 K to 3655 K, it
can be given by the empirical expression, valid in SI units,
T = 3.05 10 8 0.83

(5)

Thermal dilatation modifies the filaments length and section. However, its effects on the filament resistance will
be negligible small in this experiment.
From (4) and (5) and neglecting dilatations one gets
T = a R B0.83

(6)

Therefore, to get T it is necessary to determine a. This can be achieved by measuring the filament resistance RB,0 at
ambient temperature T0.

Task 2
a) Measure with the multimeter the ambient temperature T0.
b) It is not a good idea to use the ohmmeter to measure the filament resistance RB,0 at T0 because it introduces a small
unknown current that increases the filament temperature. Instead, to find RB,0 connect the battery to the potentiometer
and make a sufficient number of current readings for voltages from the lowest values attainable up to 1 V. (It will prove
useful to make at least 15 readings below 100 mV.) At the end, leave the potentiometer in the initial position and
disconnect one of the cables from battery to potentiometer.
Find RB for each pair of values of V and I, translate these values into the Table for Task 2,b) in the Answer Sheets.
Indicate there the lowest voltage that you can experimentally attain. Draw a graph and represent RB in the vertical axis
against I.
c) After inspecting the graphics obtained at b), select an appropriate range of values to make a linear fit to the data suitable
for extrapolating to the ordinate at the origin, RB,0. Write the selected values in the Table for Task 2, c) in the Answer
Sheets. Finally, obtain RB,0 and RB,0.
d) Compute the numerical values of a and a for RB,0 in and T0 in K using (6).

Exp. Page 4 of 11

36th International Physics Olympiad. Salamanca (Espaa) 2005


R.S.E.F.

OPTICAL PROPERTIES OF THE FILTER

The liquid filter in the test tube is an aqueous solution of copper sulphate (II) and Orange (II) aniline dye. The
purpose of the salt is to absorb the infrared radiation emitted by the filament.
The filter transmittance (transmitted intensity/incident intensity) is shown in Figure F-6 versus the wavelength.

% transmittance
30
25
20
15
10
5
0

450

500

550

600

650

700

750

/nm
F-6

Task 3
Determine 0 and from Fig. F-6.
Note:

2 is the total width at half height and 0 the wavelength at the maximum.

Exp. Page 5 of 11

36th International Physics Olympiad. Salamanca (Espaa) 2005


R.S.E.F.

PROPERTIES OF THE LDR


The material which composes the LDR is non conducting in darkness
conditions. By illuminating it some charge carriers are activated allowing some flow
of electric current through it. In terms of the resistance of the LDR one can write the
following relation
R = bE

(7)

where b is a constant that depends on the composition and geometry of the LDR and
is a dimensionless parameter that measures the variation of the resistance with the
illumination E produced by the incident radiation. Theoretically, an ideal LDR
would have = 1, however many factors intervene, so that in the real case < 1.

F-7

It is necessary to determine . This is achieved by measuring a pair R and E


(Fig. F-7) and then introducing between the lamp and the tube the grey filter F (Fig.
F-8) whose transmittance is known to be 51.2 %, and we consider free of error. This
produces an illumination E = 0.51 E. After measuring the resistance R
corresponding to this illumination, we have
R = bE

R' = b (0.512 E )

From this
ln

R
= ln 0.512
R'

(8)

F-8

Do not carry out this procedure until arriving at part b) of task 4 below.

Task 4
a) Check that the LDR remained in complete darkness for at least 10 minutes before starting this part. Connect the battery
to the potentiometer and, rotating the knob very slowly, increase the lamp voltage. Read the pairs of values of V and I
for V in the range between 9.50 V and 11.50 V, and obtain the corresponding LDR resistances R. (It will be useful to
make at least 12 readings). Translate all these values to a table in the Answer Sheet. To deal with the delay in the LDR
response, we recommend the following procedure: Once arrived at V > 9.5 V, wait 10 min approximately before
making the first reading. Then wait 5 min for the second one, and so on. Before doing any further calculation go to next
step.
b) Once obtained the lowest value of the resistance R, open the protecting cover, put the
grey filter as indicated in F-9, cover again - as soon as possible - the platform and
record the new LDR resistance R. Using these data in (8) compute and .
c) Modify Eq. (3) to display a linear dependence of ln R on RB0.83 . Write down that
equation there and label it as (9).
d) Using now the data from a), work out a table that will serve to plot Eq. (9).
e) Make the graphics plot and, knowing that c2 = hc/k, compute h and h by any method
(you are allowed to use statistical functions of the calculators provided by the
organization).

F-9

(Speed of light, c = 2.998 108 m s-1 ; Boltzmann constant, k = 1.38110-23 J K-1)

Exp. Page 6 of 11

36th International Physics Olympiad. Salamanca (Espaa) 2005


R.S.E.F.

36th International Physics Olympiad. Salamanca. Spain. Experimental Competition, 7 July 2005
COUNTRY NUMBER

COUNTRY CODE

STUDENT NUMBER

PAGE NUMBER

TOTAL No OF PAGES

Answer sheet 1
TASK 1 (2.0 points)
Draw the electric connections in the boxes and between boxes below.

Pm
P

Photoresistor
Incandescent Bulb
Potentiometer
Red socket
Black socket

Ohmmeter

Voltmeter

Ammeter

Platform

Pm
B

Potentiometer
Battery

Exp. Page 7 of 11

36th International Physics Olympiad. Salamanca (Espaa) 2005


R.S.E.F.

36th International Physics Olympiad. Salamanca. Spain. Experimental Competition, 7 July 2005
COUNTRY NUMBER

COUNTRY CODE

STUDENT NUMBER

PAGE NUMBER

TOTAL No OF PAGES

Answer sheet 2
TASK 2
a) (1.0 points)

T0 =
b) (2.0 points)

Vmin =

RB

* This is a characteristic of your apparatus. You cant go below it.

Exp. Page 8 of 11

36th International Physics Olympiad. Salamanca (Espaa) 2005


R.S.E.F.

36th International Physics Olympiad. Salamanca. Spain. Experimental Competition, 7 July 2005
COUNTRY NUMBER

COUNTRY CODE

STUDENT NUMBER

PAGE NUMBER

TOTAL No OF PAGES

Answer sheet 3
TASK 2
c) (2.5 points)

RB0 =

RB

RB0 =

d) (1.0 points)
a=

a =

TASK 3 (1.0 points)

0 =

Exp. Page 9 of 11

36th International Physics Olympiad. Salamanca (Espaa) 2005


R.S.E.F.

36th International Physics Olympiad. Salamanca. Spain. Experimental Competition, 7 July 2005
COUNTRY NUMBER

COUNTRY CODE

STUDENT NUMBER

PAGE NUMBER

TOTAL No OF PAGES

Answer sheet 4

TASK 4
a) (2.0 points)

b) (1.5 points)
R=

R =

c) (1.0 points)
Eq. (9)

Exp. Page 10 of 11

36th International Physics Olympiad. Salamanca (Espaa) 2005


R.S.E.F.

36th International Physics Olympiad. Salamanca. Spain. Experimental Competition, 7 July 2005
COUNTRY NUMBER

COUNTRY CODE

STUDENT NUMBER

PAGE NUMBER

TOTAL No OF PAGES

Answer sheet 5
TASK 4
d) (3.0 points)
V

e) (3.0 points)
h =

h=

Exp. Page 11 of 11

7th Asian Physics Olympiad

Experimental Problem

Page 1 of 6

Experimental Problem
This experimental problem consists of two related parts.

Part 1
Measurement of the specific heat of aluminum
in the 450 C 650 C temperature range. (10 points)
In this part, you can use the following equipment ONLY:

4
7
1

2
10

1. A plastic cup with a cap which has a hole for the thermometer;
2. A digital thermometer with accuracy of 0.10C; The temperature of the
sensor (10) is shown on the top display, and bottom display shows the
temperature of the room. Do not press max/min button: pressing max/min
button changes the readings between current, maximal and minimal
values. If water temperature exceeds 70C, the thermometer shows H
denoting it is out of its range. WARNING: DO NOT USE THE
THERMOMETER TO MEASURE THE TEMPERATURE OF
LIQUID NITROGEN! THERMOMETER CAN BE USED ONLY IN
PART 1.
3. An aluminum cylinder with a hole;
4. Electronic scales with accuracy of 1g; Make sure that the scales are
situated on the flat surface. The button Tare/Zero serves as On/Off and
sets the zero reading of the scales. Do not press any other buttons. Note:
the scales automatically turn OFF after some time; you have to turn them
ON back and reset the zero reading of the scales. Press the top of the

7th Asian Physics Olympiad

5.
6.
7.
8.
9.

Experimental Problem

Page 2 of 6

scales approximately every 50 seconds in order to prevent automatic


switch off of the scales.
A digital timer; Pressing left button shifts the timer from Clock mode to
Stopwatch mode. In Stopwatch mode the middle button serves as
Stop/Start, and the right button serves as Reset.
Dewar flask with hot water;
A jar for the used water;
Plotting (graphing) paper (2 pages);
Pieces of thread.

The results of the measurement of specific heat of aluminum will be used in Part
2 of the experimental problem.
The specific heat of aluminum should be determined from the comparison of
two experimental curves:
1) the cooling curve of hot water in a plastic cup without the aluminum cylinder
(the first experiment);
2) the cooling curve of hot water in a plastic cup with aluminum cylinder
immersed (the second experiment);
The specific heat of water is given c w = 4.20kJ/(kg K).
Density of water w = 1.00 10 3 kg / m 3 .
Density of aluminum Al = 2.70 10 3 kg / m 3 .
Warning: Be very careful with hot water. Remember that water at temperature
T>500 C can cause burns. DO NOT USE LIQUID NITROGEN IN
THIS PART!

The task
1) [1 point] Derive theoretically an expression for aluminum specific heat cAl in
terms of experimentally measured quantities : mass 1 of hot water in the first
experiment, mass m2 of hot water in the second experiment, mass m of
aluminum cylinder and the ratio of heat capacities K = C1 / C2 , where 1 is the
heat capacity of water in the first experiment, 2 is the combined heat capacity
of water and aluminum cylinder in the second experiment.
In parts 1b) and 1c) you will perform measurements to determine K. Parts 1b) and 1c)
should be performed with closed caps. Assume that in this case heat exchange of the
contents of the cup with the environment depends linearly on the difference in their
temperatures. The linearity coefficient depends only on the level of the water in the

7th Asian Physics Olympiad

Experimental Problem

Page 3 of 6

cup. Make sure that the aluminum cylinder is fully immersed into water in part 1c).
You can neglect the heat capacity of the cup.
1b) [1.5 points] Perform the first experiment investigate the relationship between
the temperature of water 1 and time t in the range of temperatures from 450C to
650C. Provide a table of measurements. Write the value of 1 on the answer
sheet.
1) [1.5 points] Perform the second experiment investigate the relationship
between the temperature of water with aluminum cylinder 2 and time t in the
range of temperatures from 450C to 650C. Provide a table of measurements.
Write the values of 2 and on the answer sheet.
1d) [4 points] Use graphs to determine the ratio of the heat capacities K = C1 / C2 and
the uncertainty K . Write the values of K and K on the answer sheet.
1e) [2 points] Determine the numerical value of cAl and estimate the uncertainty of
measurement cAl . Write the values of cAl and cAl on the answer sheet.

Part 2
Measurement of the specific latent heat of evaporation
of liquid nitrogen.
(10 points)
In this part you can use the following equipment:

7th Asian Physics Olympiad

Experimental Problem

Page 4 of 6

4
1
3
1.
2.
3.
4.
5.
6.
7.

A Styrofoam cup with a cap;


Dewar flask with liquid nitrogen;
An aluminum cylinder with a hole (item 3, Part 1);
Electronic scales with accuracy of 1g (item 4, Part 1);
A digital timer (item 5, Part 1);
Plotting (graphing) paper (2 pages);
Pieces of thread.

The specific latent heat of evaporation of water is well known, while rarely we have
to deal with one the main atmospheric gases, nitrogen, in its liquid form. The boiling
temperature of liquid nitrogen under normal atmospheric pressure is very low,
TN = 77 K = 196 0 C.

In this experiment you are asked to measure the specific latent heat of
evaporation of nitrogen. Because of heat exchange with the environment nitrogen in
a Styrofoam cup evaporates and its mass decreases at some rate. When an aluminum
cylinder initially at room temperature is immersed into nitrogen, nitrogen will boil
violently until the temperature of the aluminum sample reaches the temperature of
liquid nitrogen. The final brief ejection of some amount of vaporized nitrogen from
the cup indicates that aluminum has stopped cooling-- this ejection is caused by the
disappearance of the vapor layer between aluminum and nitrogen. After aluminum
reaches the temperature of nitrogen, the evaporation of nitrogen will continue.
When considering a wide range of temperatures, one can observe that the
specific heat of aluminum cAl depends on absolute temperature. The graph of

7th Asian Physics Olympiad

Experimental Problem

Page 5 of 6

aluminums specific heat in arbitrary units versus temperature is shown in Fig. 1. Use
the result of specific heat measurement in 450C-650C temperature range in Part 1 to
normalize this curve in absolute units.

Fig. 1. The relationship between aluminums specific heat in arbitrary units and temperature.

Warnings:
1) Liquid nitrogen has temperature N = - 196C. To prevent frostbite do not
touch nitrogen or items which were in contact with nitrogen. Make sure to
keep away your personal metal belongings such as jewelry, wrist watch, etc.
2) Do not put any irrelevant items into nitrogen;
3) Be careful while putting the aluminum cylinder into the liquid nitrogen to
prevent spurts or spilling.

The task
2a) [3 points] Measure the evaporation rate of nitrogen in a Styrofoam cup with a
closed cap, and measure the mass of nitrogen evaporated during the cooling of
the aluminum cylinder (aluminum cylinder is loaded through a hole in the cup).
Proceed in the following manner. Set up the Styrofoam on the scales, pour about
250g of liquid nitrogen in it, wait about 5 minutes and then start taking
measurements. After some amount of nitrogen evaporates, immerse the
aluminum cylinder into the cup this will result in a violent boiling. After
aluminum cylinder cools down to the temperature of nitrogen, evaporation calms
down. You should continue taking measurements in this regime for about 5
minutes until some additional amount of nitrogen evaporates. During the whole
process record the readings of the scales M (t ) as a function of time.

7th Asian Physics Olympiad

Experimental Problem

Page 6 of 6

IN NO CASE SHOULD YOU TOUCH THE ALUMINUM CYLINDER


AFTER IT WAS SUBMERGED INTO LIQUID NITROGEN.
In your report provide a table of M (t ) and mN (t ) , where mN (t ) is the mass of the
evaporated nitrogen.
2b) [1 point] Using the results of the measurements of M (t ) in 2a), plot the graph of
the mass of evaporated nitrogen mN versus time t . The graph should illustrate
all the three stages of the process the calm periods before and after immersion
of aluminum, and the violent boiling of nitrogen.
2) [3.0 points] Determine from the graph the mass m NAl of nitrogen evaporated
only due to heat exchange with the aluminum cylinder, as it is cooled down from
room temperature to the temperature of liquid nitrogen. In order to do this you
have to take into account the heat exchange with the environment through the
cup before, during and after the cooling of aluminum. Write the value of m NAl
and its uncertainty mNAl on the answer sheet.
2d) [0.5 points] Using the result of measurement of aluminums specific heat in the
temperature range of 450C 650C (Part 1), normalize the graph of the
relationship between aluminums specific heat and temperature from arbitrary
to absolute units. On the answer sheet write the value of the coefficient of
conversion from arbitrary units to absolute units:
c Al ( J /(kg K )) = c Al (arb.units )

2) [2.5 points] Using the results of measurement of the mass of nitrogen


evaporated due to cooling of the aluminum cylinder and the normalized graph of
the relationship between specific heat and temperature, determine nitrogens
specific latent heat of evaporation . Write the value of and its uncertainty
on the answer sheet.

Good Luck and Look Good!

37thInternationalPhysicsOlympiad
Singapore
817July2006

ExperimentalCompetition
Wed12 July2006

ExperimentalCompetition

Page 2

Listofapparatusandmaterials

Label
A

Component
Microwavetransmitter

Quantity
1

Label

Component
Latticestructureina
blackbox
Goniometer

Quantity
1

Microwavereceiver

Transmitter/receiver
holder
Digitalmultimeter

Prismholder

Rotatingtable

DCpowersupplyfor
transmitter
SlabasaThinfilm
sample
Reflector(silvermetal
sheet)
Beamsplitter(blue
Perspex)
Verniercaliper
(providedseparately)

Lens/reflectorholder

Planocylindricallens

Waxprism

BluTack

1pack

30cm ruler(provided
separately)

ExperimentalCompetition

Page 3

Caution:
Theoutputpowerofthemicrowavetransmitteriswellwithinstandardsafety
levels.Nevertheless,oneshouldneverlookdirectlyintothemicrowavehornat
closerangewhenthetransmitterison.
Donotopen theboxcontainingthelattice
I .
Thewaxprisms
O arefragile(usedinPart3).
Note:
Itisimportanttonotethatthemicrowavereceiveroutput(CURRENT)is
proportionaltotheAMPLITUDEofthemicrowave.
AlwaysuseLOgainsettingofthemicrowavereceiver.
Donotchangetherangeofthemultimeterduringthedatacollection.
Placetheunusedcomponentsawayfromtheexperimenttominimize
interference.
Alwaysusethecomponentlabels(
A ,
B ,
C ,) toindicatethecomponents
inallyourdrawings.

ExperimentalCompetition

Blacklead

Page 4

Redlead

The digital multimeter should be used with the two leads connected as shown in the
diagram.Youshouldusethe2mcurrentsettinginthisexperiment.

ExperimentalCompetition

Page 5

Part1:Michelsoninterferometer
1.1.Introduction
InaMichelson interferometer,abeamsplittersendsan incomingelectromagnetic(EM)
wavealongtwoseparatepaths,andthenbringstheconstituentwavesbacktogetherafter
reflectionsothattheysuperpose,forminganinterferencepattern.Figure1.1illustratesthe
setupforaMichelsoninterferometer.Anincidentwavetravelsfromthetransmittertothe
receiveralongtwodifferentpaths.Thesetwowavessuperposeandinterfereatthereceiver.
The strength of signal at the receiver depends on the phase difference between the two
waves,whichcanbevariedbychangingtheopticalpathdifference.

Receiver
Beam
splitter

Transmitter

Reflectors

Figure1.1:SchematicdiagramofaMichelsoninterferometer.
1.2.Listofcomponents
1) Microwavetransmitter
A withholder
C
2) Microwavereceiver
B withholder
C
3) Goniometer
J
4) 2reflectors:reflector
G withholder
M andthinfilm
F actingasareflector.
5) Beamsplitter
H withrotatingtable
L actingasaholder
6) Digitalmultimeter
D

ExperimentalCompetition

Page 6

1.3.Task:Determinationofwavelengthofthemicrowave

[2marks]

Using only the experimental components listed in Section 1.2, set up a Michelson
interferometerexperimenttodeterminethewavelength l ofthemicrowaveinair.Record
yourdataanddetermine l insuchawaythattheuncertaintyis0.02cm.
Notethatthethinfilmispartiallytransmissive,somakesureyoudonotstandormove
behinditasthismightaffectyourresults.

Part2:Thinfilminterference
2.1.Introduction
AbeamofEMwaveincidentonadielectricthinfilmsplitsintotwobeams,asshownin
Figure 2.1. Beam A is reflected from the top surface of the film whereas beam B is
reflectedfromthebottomsurfaceofthefilm.ThesuperpositionofbeamsAandBresults
inthesocalledthinfilminterference.

A
B

q1

q1

q2

Figure2.1:Schematicofthinfilminterference.

The difference in the optical path lengths of beam A and B leads to constructive or
destructiveinterference.TheresultantEMwaveintensity Idependsonthepathdifference
ofthetwointerferingbeamswhich inturndependsontheangleofincidence, q1,ofthe

ExperimentalCompetition

Page 7

incidentbeam,wavelength loftheradiation,andthethicknesstandrefractiveindex nof


thethinfilm.Thus,therefractiveindex n ofthethinfilmcanbedeterminedfrom I1 plot,
usingvaluesof tand l.

2.2. Listofcomponents
1) Microwavetransmitter
A withholder
C
2) Microwavereceiver
B withholder
C
3) Planocylindricallens
N withholder
M
4) Goniometer
J
5) Rotatingtable
L
6) Digitalmultimeter
D
7) Polymerslabactingasathinfilmsample
F
8) Verniercaliper
2.3.Tasks:Determinationofrefractiveindexofpolymerslab

[6marks]

1)Deriveexpressionsfortheconditionsofconstructiveanddestructiveinterferences
intermsof q1,t, landn.
[1mark]
2)UsingonlytheexperimentalcomponentslistedinSection 2.2,setupanexperiment
tomeasurethereceiveroutputSasafunctionoftheangleof incidence1 inthe
rangefrom40o to75o.Sketchyourexperimentalsetup,clearlyshowingtheangles
ofincidenceandreflectionandthepositionofthefilmontherotatingtable.Mark
all components using the labels given on page 2. Tabulate your data. Plot the
receiveroutputS versustheangleofincidence1.Determineaccuratelytheangles
correspondingtoconstructiveanddestructiveinterferences.
[3marks]
3)Assumingthattherefractiveindexofairis1.00,determinetheorderofinterference
m andtherefractiveindexofthepolymerslab n.Writethevaluesof m and n onthe
answersheet.

ExperimentalCompetition

Page 8

[1.5marks]
4)Carryouterroranalysisforyour resultsandestimatetheuncertaintyof n.Writethe
valueoftheuncertaintynontheanswersheet.
[0.5marks]

Note:
The lens should be placed in front of the microwave transmitter with the planar
surface facing the transmitter to obtain a quasiparallel microwave beam. The
distancebetweentheplanarsurfaceofthelensandtheapertureoftransmitterhorn
shouldbe3cm.
Forbestresults,maximizethedistancebetweenthetransmitterandreceiver.
Deviations of themicrowave emitted by transmitter from a plane wave may cause
extrapeaks intheobservedpattern.Intheprescribedrangefrom40o to75o,onlyone
maximumandoneminimumexistduetointerference.

Part3:FrustratedTotalInternalReflection
3.1. Introduction
Thephenomenonoftotalinternalreflection(TIR)mayoccurwhentheplanewavetravels
from an optically dense medium to less dense medium. However, instead of TIR atthe
interfaceaspredictedbygeometricaloptics,theincomingwaveinrealitypenetratesinto
thelessdensemediumandtravelsforsomedistanceparalleltotheinterfacebeforebeing
scatteredbacktothedenser medium(see Figure3.1).Thiseffectcanbedescribed bya
shiftDofthereflectedbeam,knownastheGoosHnchenshift.

Prism
n1

q1 D
n2

Air

Figure3.1:AsketchillustratinganEMwaveundergoingtotalinternalreflectionina
prism.TheshiftDparalleltothesurfaceinairrepresentstheGoosHnchenshift

ExperimentalCompetition

Page 9

Prism
n1

Transmitter

q1 D

n2

Air

n1

Receiver
Prism

Figure1.2:Asketchoftheexperimentalsetupshowingtheprismsandtheairgapof
distance d.TheshiftD paralleltothesurfaceinairrepresentstheGoosHnchenshift.
zisthedistancefromthetipoftheprismtothecentralaxisofthetransmitter.
Ifanothermediumofrefractiveindex n1 (i.e.madeofthesamematerialasthefirstmedium)
isplacedatasmalldistancedtothefirstmediumasshowninFigure3.2,tunnelingofthe
EMwavethroughthesecondmediumoccurs.Thisintriguingphenomenonisknownasthe
frustrated total internal reflection (FTIR). The intensity of the transmitted wave, It,
decreasesexponentiallywiththedistanced:

I t = I 0 exp ( -2g d )

(3.1)

whereI0 istheintensityoftheincidentwaveand gis:

g=

2p

n12 2
sin q1 -1
n22

(3.2)

where l is the wavelength of EM wave in medium 2 and n2 is the refractive index of


medium2(assumethattherefractiveindexof medium2,air,is1.00).

3.2. Listofcomponents
1) Microwavetransmitter
A withholder
C

ExperimentalCompetition

Page 10

2) Microwavereceiver
B withholder
C
3) Planocylindricallens
N withholder
M
4) 2equilateralwaxprisms
O withholder
K androtatingtable
L actingasa
holder
5) Digitalmultimeter
D
6) Goniometer
J
7) Ruler
3.3. DescriptionoftheExperiment
Using only the list of components described in Section 3.2, set up an experiment to
investigatethevariationoftheintensity It asafunctionoftheairgapseparation dinFTIR.
Forconsistentresults,pleasetakenoteofthefollowing:
Useonearmofthegoniometerfor thisexperiment.
Choosetheprismsurfacescarefullysothattheyareparalleltoeachother.
Thedistancefromthecentreofthecurvedsurfaceofthelensshouldbe2cmfrom
thesurfaceoftheprism.
Placethedetectorsuchthatitshornisincontactwiththefaceoftheprism.
Foreachvalueof d,adjustthepositionofthemicrowavereceiveralongtheprism
surfacetoobtainthemaximumsignal.
Makesurethatthedigital multimeterisonthe2mArange.Collectdatastarting
from d =0.6cm.Discontinuethemeasurementswhenthereadingofthemultimeter
fallsbelow0.20mA.

3.4. Tasks:Determinationofrefractiveindexofprismmaterial

[6marks]

Task1
Sketch your final experimental setup and mark all components using the labels
givenatpage2.Inyoursketch,recordthevalueofthedistancez(seeFigure3.2),
thedistancefromthetipoftheprismtothecentralaxisofthetransmitter.
[1Mark]
Task2
Performyourexperimentandtabulateyourdata.Perform thistask twice.
[2.1Marks]

ExperimentalCompetition

Page 11

Task3
(a) Byplottingappropriategraphs,determinetherefractiveindex, n1,oftheprism
witherroranalysis.
(b) Writetherefractiveindex n1,anditsuncertainty n1,oftheprismintheanswer
sheetprovided.
[2.9Marks]

Part4:Microwavediffractionofametalrodlattice:Braggreflection
4.1.Introduction
BraggsLaw
Thelatticestructureofareal crystalcanbeexaminedusingBraggsLaw,
2d sinq =ml

(4.1)

where d refers to the distance between a set of parallel crystal planes that reflect the
Xray mistheorderofdiffractionand q istheanglebetweentheincidentXraybeamand
thecrystalplanes.BraggslawisalsocommonlyknownasBraggsreflectionorXray
diffraction.

ExperimentalCompetition

Page 12

Metalrodlattice
BecausethewavelengthoftheXrayiscomparabletothelatticeconstantofthecrystal,
traditional Braggs diffraction experiment is performed using Xray. For microwave,
however,diffractionoccursin latticestructureswith much largerlatticeconstant,which
canbemeasuredeasilywitharuler.

a
b

x
Figure4.1:Ametalrodlatticeoflatticeconstantsaandb,andinterplanar
spacingd.

a
b
d

y
x
Figure 4.2: Topview of the metalrod lattice shown in Fig. 4.1 (not to
scale).Thelinesdenotediagonal planesofthelattice.

ExperimentalCompetition

Page 13

Inthisexperiment, theBragglawisusedtomeasurethelatticeconstantof a latticemadeof


metalrods. Anexampleofsuchmetalrodlattice isshowninFig.4.1,wherethemetalrods
are shown as thick vertical lines. The lattice planes along the diagonal direction of the
xyplaneareshownasshadedplanes. Fig.4.2showsthetopview(lookingdownalongthe
zaxis)ofthemetalrodlattice,wherethepointsrepresenttherodsand thelinesdenote the
diagonallatticeplanes.

4.2.Listofcomponents
1) Microwavetransmitter
A withholder
C
2) Microwavereceiver
B withholder
C
3) Planocylindricallens
N withholder
M
4) Sealedboxcontainingametalrodlattice
I
5) Rotatingtable
L
6) Digitalmultimeter
D
7) Goniometer
J

a
a

x
Figure4.3:Asimplesquarelattice.

Inthisexperiment,youaregivenasimplesquarelatticemadeofmetalrods,asillustrated
inFig.4.3.Thelatticeissealedinabox.Youareaskedtoderivethelatticeconstantaof

ExperimentalCompetition

Page 14

the lattice from the experiment. DO NOTopen the box. No marks will be given tothe
experimentalresultsifthesealisfoundbrokenaftertheexperiment.

4.3. Tasks:Determinationoflatticeconstantofgivensimplesquarelattice[6Marks]

Task1
Drawatopviewdiagramofthesimple squarelatticeshowninFig.4.3.Inthediagram,
indicate the lattice constant a of the given lattice and the interplanar spacing d of the
diagonalplanes.Withthehelpofthisdiagram,deriveBraggsLaw.
[1Mark]
Task 2
UsingBraggslawandtheapparatusprovided,designanexperimenttoperformBragg
diffractionexperimenttodeterminethelatticeconstanta ofthelattice.
(a) Sketchtheexperimentalsetup.Markallcomponentsusingthelabelsinpage 2 and
indicateclearlytheanglebetweentheaxisofthetransmitterandlatticeplanes, q,
andtheanglebetweentheaxisofthetransmitterandtheaxisofthereceiver, z.In
your experiment, measure the diffractionon the diagonalplanes the direction of
whichisindicatedbytheredlineonthebox.
[1.5Marks]
(b) Carryoutthediffractionexperimentfor20 q50.Inthisrange,youwillonly
observe the first order diffraction. In the answer sheet,tabulate your results and
recordboththe qand z.
[1.4Marks]
(c) Plot thequantityproportionaltotheintensityofdiffractedwaveasafunctionof q.
[1.3Marks]
(d)Determine the lattice constant a using the graph and estimate the experimental
error.
[0.8Marks]

ExperimentalCompetition

Page 15

Note:
1. Forbestresults,thetransmittershouldremainfixedduringtheexperiment. The
separationbetweenthetransmitterandthelattice,aswellasthatbetweenlattice
andreceivershouldbeabout50cm.
2. Useonlythediagonalplanesinthisexperiment.Yourresultwillnotbecorrectif
youtrytouseanyotherplanes.
3. Thefaceofthelatticebox withthereddiagonallinemustbeatthetop.
4. To determine the position of the diffraction peak with better accuracy, use a
numberofdatapointsaroundthepeakposition.

Experimental Competition

Before attempting to assemble


your apparatus, read the problem text completely!
Please read this first:
1. The time available for the Experimental problem 1 is 2 hours and 45 minutes;
and that for the Experimental problem 2 is 2 hours and 15 minutes.
2. Use only the pen and equipments provided.
3. Use only the one side of the provided sheets of paper.
4. In addition to blank sheets where you may write freely, there is a set of Answer
sheets where you must summarize the results you have obtained. Numerical
results must be written with as many digits as appropriate; do not forget the units.
5. Please write on the blank sheets the results of all your measurements and
whatever else you deem important for the solution of the problem that you wish to
be evaluated during the marking process. However, you should use mainly
equations, numbers, symbols, graphs, figures, and as little text as possible.
6. It is absolutely imperative that you write on top of each sheet: your student code
as shown on your identification tag, and additionally on the blank sheets: your
student code, the progressive number of each sheet (Page n. from 1 to N) and the
total number (N) of blank sheets that you use and wish to be evaluated (Page
total).
7. The student should start with a new page for each section. It is also useful to write
the number of the section you are answering at the beginning of each such section.
If you use some sheets for notes that you do not wish to be evaluated by the
marking team, just put a large cross through the whole sheet and do not number it.
8. When you have finished, turn in all sheets in proper order (answer sheet first, then
used sheets in order, the unused sheets and problem text at the bottom) and put
them all inside the envelope provided; then leave everything on your desk. You
are not allowed to take anything out of the room.

Experimental problem 1
Using the interference method to measure the thermal expansion
coefficient and temperature coefficient of refractive index of glass (10
points)
(1) Instructions
Optical instruments are often used at high or low temperatures. When optical
instruments are used at different temperatures, the thermal properties of the materials,
of which the optical elements were made of, including thermal expansion and the
variation of refractive index with temperature, will directly affect their optical
properties. Two parameters, i.e. the linear thermal expansion coefficient and the
temperature coefficient of refractive index , are defined as

and

respectively to describe these properties, where L stands for the length of the material,
T the temperature, and n the refractive index. The purpose of the present experiment is
to measure

and of a given glass material.

Figure 1

(2) Experimental apparatus, devices, and materials


1. SampleThe cylinder-like sample used in present experiment is made of uniform
and isotropic glass, as shown in Fig. 1. Where A represents a glass cylinder with a
segmental part parallel to its axis cut away, its top and bottom surface are
approximately parallel to each other. B and B are two circular plates made of the
same glass material, from each of which a segmental part parallel to the axis was also
cut away. The top and bottom surface of each glass plate are not parallel to each other.
A, B, and B are glued together as a whole, as shown in Fig. 1. The refractive index of

the glue is as the same as that of the glass, and its thickness can be neglected.
2. HeaterThe heater used in this experiment is schematically shown in Fig. 2. A
knob on the right of the electric oven is used to adjust the temperature of the electric
oven. A big aluminum cylinder was bored a co-axis pipe-like sample cavity in the
middle of it,. The experimental sample can be put at the bottom of this cavity. Besides,
there is a small aluminum cylinder, through which two tube-like holes of different
radius were bored. The small hole allows light to pass through, while a probe of a
thermometer can be inserted in the big hole. If you want to heat the sample, you
should first slip the sample carefully into the cavity of the big aluminum cylinder
before you heat it (in doing so, the big aluminum cylinder should be inclined
somewhat to avoid cracking the sample). Next, put the small aluminum cylinder onto
the sample already located in the cavity. Then, put the whole big aluminum cylinder
including the sample and the small cylinder inside into the steel cup on the electric
oven. (For heating, you should not put any water in the steel cup, and you shouldnt
take away the steel cup but put the whole aluminum cylinder directly on the oven,
either.)

Figure 2
3. Light source holder.

As shown in Fig. 3, a support consisting of a vertical post

and a base trestle is designed to hold the laser light source. The post and the base
trestle are fixed together at C and two tunable adjusting screws A and B are attached
to the trestle. A He-Ne laser and its power supply are held at the upper part of the post,

as shown in Fig.3. Just below the laser, a slant bracket is attached to the power
supply, on which is placed an aluminum plate with a hole through it. A piece of graph
paper is attached to the aluminum plate, which can be used as an observation screen.

Figure 3
4. Sample platformA circular platform with three adjusting screws a, b, and c is
designed to hold the heating oven or the big aluminum cylinder including the sample
and the small aluminum cylinder. The platform is placed on the experimental table,
close to the base trestle, right below the laser light source, as shown in Fig 3.
5. A digital thermometer
6.

A straight ruler

7.

A basin (containing the cooling water)

8.

A piece of towel

9.

A piece of graph paper

10.

A calculator

11. A pen and a pencil


Attention
a. Never look at the laser light source along the direction opposite to the incident
direction of the light. Otherwise, the laser light might hurt your eyes.
b. Never touch the optical surface of the sample. Take the sample gently to avoid any
damage.

(3) Experimental content


1. Answer Questions

2.4

points
1.1 When a beam of white natural light coming from a lamp and passing through a
piece of red transparent paper strikes on a thick glass slab of a thickness about 2 cm,
the beams reflected from the top and bottom surfaces of the slab meet at the
observation screen, resulting in a light spot on the screen without interference fringes,
as shown in Fig.4 (a). However, when a laser beam strikes on the same glass slab, the
reflected beams meet at the same observation screen, resulting in a light spot with
interference fringes, as shown in Fig. 4(b). What is the reason accounting for these
two different phenomena? (choose the correct one)
A. The laser beam is stronger than the red beam
B. The laser beam is more collimated than the red beam
C. The laser beam is more coherent than the red beam
D. The wavelength of the laser beam is shorter than that of the red beam

Fig.4

1.2 a As shown in Fig.5, if a laser light beam strikes approximately normally on the
region a, b, and c of the sample respectively, how many main light spots of the
reflection light will be observed (need not consider multiple reflections)? Will the
profiles of these reflected optical spots be inevitably the same as that of the incident
light?

Fig. 5
1.2 b If the profile of some spots of the reflected light is different from that of the
incident light?

Please give a reason to account for.

2. Experiment: Measurement of

and

With the wavelength of the laser light


A

of the glass

(7.6 points)

, the height of the glass cylinder

and the average refractive index of the glass corresponding to

this given wavelength and the given temperature range of measurement


measure the thermal expansion coefficient
refractive index

and the temperature coefficient of the

for the glass sample over the temperature range from

.(Within this temperature range

and

to

may be taken as constant.)

2.1 Design the Experiment, draw the experimental ray diagrams and derive the
formulae relevant to the measurement.

(3.2 points)

2.2 Carry out the experiment and record the measured data of the thermal expansion
coefficient
2.3 Calculate

and
and

(1.6 points)
of the given glass material and estimate their uncertainties.
(2.6 points)

2.4 Write the final experimental results.

(0.2 points)

Attention
1. When put the sample into the sample cavity of the big aluminum cylinder, in order
to avoid cracking the sample, incline somewhat the big aluminum cylinder, and
make the sample slip towards the bottom of the sample cavity carefully and

slowly.
2. During the process of experiment, the temperature can be increased continuously.
In order to guarantee enough time you better to measure your data when the
temperature is increasing. To heat the sample, the knob of the electric oven may
be first turned to its maximum. When the temperature approaches to near 90
( around 85 ), turn the knob to the minimum immediately to stop heating.
During heating, the indication light on the left of the oven might be off and then
on again. This indicates that the oven is controlling the temperature itself
automatically. Do not care about it.
3. After the sample in the cavity of the big aluminum cylinder is heated enough, its

natural cooling down will be very slow. To get a rapid cooling, you may put the
heated steel cup containing the big aluminum cylinder with the small aluminum
cylinder inside into the water-filled basin. After a while of coolingat least 5
minutes, carefully use the towel to wrap the big aluminum cylinder and put it
directly into the cooling water to speed up the cooling process. Be careful to avoid
burning your hand. After cooling, use the towel to dry the big aluminum cylinder,
and put it back to the steel cup, then you can heat it again. In order to avoid short
circuit, never pour any water into the electric oven.
4. When you finished the experiment, turn off the electric oven immediately to avoid

overheating.

Warning: Be careful in your experiments! If your sample or


instrument is broken, you may have difficulties to continue your
experiments, since we do not have enough backups!

Experimental Problem
Determination of energy band gap of semiconductor thin films
I. Introduction
Semiconductors can be roughly characterized as materials whose electronic properties
fall somewhere between those of conductors and insulators. To understand
semiconductor electronic properties, one can start with the photoelectric effect as a
well-known phenomenon. The photoelectric effect is a quantum electronic
phenomenon, in which photoelectrons are emitted from the matter through the
absorption of sufficient energy from electromagnetic radiation (i.e. photons). The
minimum energy which is required for the emission of an electron from a metal by
light irradiation (photoelectron) is defined as "work function". Thus, only photons
with a frequency higher than a characteristic threshold, i.e. with an energy h ( h
is the Planck s constant) more than the material s work function, are able to knock
out the photoelectrons.

Figure 1. An illustration of photoelectron emission from a metal plate: The incoming photon
should have an energy which is more than the work function of the material.

In fact, the concept of work function in the photoelectric process is similar to the
concept of the energy band gap of a semiconducting material. In solid state physics,
the band gap E g is the energy difference between the top of the valence band and the
bottom of the conduction band of insulators and semiconductors. The valence band is
completely filled with electrons, while the conduction band is empty however
electrons can go from the valence band to the conduction band if they acquire
sufficient energy (at least equal to the band gap energy).The semiconductor's
conductivity strongly depends on its energy band gap.
Conduction
band

Unfilled

E band

Filled
bands

Energy
Band Gap
Valence
band

Figure 2. Energy band scheme for a semiconductor.

Band gap engineering is the process of controlling or altering the band gap of a
material by controlling the composition of certain semiconductor alloys. Recently, it
has been shown that by changing the nanostructure of a semiconductor it is possible
to manipulate its band gap.
In this experiment, we are going to obtain the energy band gap of a thin-film
semiconductor containing nano-particle chains of iron oxide (Fe2O3) by using an
optical method. To measure the band gap, we study the optical absorption properties
of the transparent film using its optical transmission spectrum. As a rough statement,
the absorption spectra shows a sharp increase when the energy of the incident photons
equals to the energy band gap.
II. Experimental Setup
You will find the following items on your desk:
1. A large white box containing a spectrometer with a halogen lamp.
2. A small box containing a sample, a glass substrate, a sample-holder, a grating,
and a photoresistor.
3. A multimeter.
4. A calculator.
5. A ruler.
6. A card with a hole punched in its center.
7. A set of blank labels.
The spectrometer contains a goniometer with a precision of 5 . The Halogen lamp
acts as the source of radiation and is installed onto the fixed arm of the spectrometer
(for detailed information see the enclosed "Description of Apparatus").
The small box contains the following items:
1. A sample-holder with two windows: a glass substrate coated with Fe2O3 film
mounted on one window and an uncoated glass substrate mounted on the other.
2. A photoresistor mounted on its holder, which acts as a light detector.
3. A transparent diffraction grating (600 line/mm).

Note: Avoid touching the surface of any component in the small box!
A schematic diagram of the setup is shown in Figure 3:

Ohmmeter
(Max. range 200 M )

Grating
600 lines/mm
Goniometer

Optical axis

Diffusive glass

Lens

Halogen lamp

Sample
Entrance
hole
Photoresistor

Figure 3. Schematic diagram of the experimental setup.

III. Methods
To obtain the transmission of a film at each wavelength, T film

, one can use the

following formula:

T film ( )

(1)

I film ( ) / I glass ( )

where I film and I glass are respectively the intensity of the light transmitted from the
coated glass substrate, and the intensity of the light transmitted from the uncoated
glass slide. The value of I can be measured using a light detector such as a
photoresistor. In a photoresistor, the electrical resistance decreases when the intensity
of the incident light increases. Here, the value of I can be determined from the
following relation:
I ( ) C( )R 1
(2)
where R is the electrical resistance of the photoresistor, C is a -dependent
coefficient.
The transparent grating on the spectrometer diffracts different wavelengths of
light into different angles. Therefore, to study the variations of T as a function of ,
it is enough to change the angle of the photoresistor ( ) with respect to the optical
axis (defined as the direction of the incident light beam on the grating), as shown in
Figure 4.
From the principal equation of a diffraction grating:
n

d [sin(

) sin

] (3)

one can obtain the angle


corresponding to a particular : n is an integer number
representing the order of diffraction, d is the period of the grating, and o is the angle
the normal vector to the surface of grating makes with the optical axis (see Fig. 4). (In
this experiment we shall try to place the grating perpendicular to the optical axis
making o 0 , but since this cannot be achieved with perfect precision the error
associated with this adjustment will be measured in task 1-e.)

'

Grating
o

Optical axis

Figure 4. Definition of the angles involved in Equation 3.

Experimentally it has been shown that for photon energies slightly larger than
the band gap energy, the following relation holds:
h

A(h

Eg )

(4)

where
is the absorption coefficient of the film, A is a constant that depends on the
film s material, and is the constant determined by the absorption mechanism of the
film s material and structure. Transmission is related to the value of
through the
well-known absorption relation:
3

T film

exp (- t)

(5)

where t is thickness of the film.


IV. Tasks:
0. Your apparatus and sample box (small box containing the sample holder) are
marked with numbers. Write down the Apparatus number and Sample number in
their appropriate boxes, in the answer sheet.
1. Adjustments and Measurements:

1-a

Check the vernier scale and report the maximum precision


).

0.1 pt

Note: Magnifying glasses are available on request.


Step1:
To start the experiment, turn on the Halogen lamp to warm up. It would be better not
to turn off the lamp during the experiment. Since the halogen lamp heats up during
the experiment, please be careful not to touch it.
Place the lamp as far from the lens as possible, this will give you a parallel light
beam.
We are going to make a rough zero-adjustment of the goniometer without utilizing
the photoresistor. Unlock the rotatable arm with screw 18 (underneath the arm), and
visually align the rotatable arm with the optical axis. Now, firmly lock the rotatable
arm with screw 18. Unlock the vernier with screw 9 and rotate the stage to 0 on the
vernier scale. Now firmly lock the vernier with screw 9 and use the vernier fineadjustment screw (screw 10) to set the zero of the vernier scale. Place the grating
inside its holder. Rotate the grating's stage until the diffraction grating is roughly
perpendicular to the optical axis. Place the card with a hole in front of the light source
and position the hole such that a beam of light is incident on the grating. Carefully
rotate the grating so that the spot of reflected light falls onto the hole. Then the
reflected light beam coincides with the incident beam. Now lock the grating's stage
by tightening screw 12.

1-b

By measuring the distance between the hole and the grating,


estimate the precision of this adjustment ( o ).

0.3 pt

Now, by rotating the rotatable arm, determine and report the


range of angles for which the first-order diffraction of visible light
(from blue to red) is observed.

0.2 pt

Step 2:
Now, install the photoresistor at the end of the rotatable arm. To align the system
optically, by using the photoresistor, loosen the screw 18, and slightly turn the
rotatable arm so that the photoresistor shows a minimum resistance. For fine
positioning, firmly lock screw 18, and use the fine adjustment screw of the rotatable
arm.
4

Use the vernier fine-adjustment screw to set the zero of the vernier scale.

1-c

(0)
Report the measured minimum resistance value ( Rmin
).
Your zero-adjustment is more accurate now, report the
precision of this new adjustment ( o ) .

Note:
o is the error in this alignment i.e. it is a measure of
misalignment of the rotatable arm and the optical axis.

0.1 pt

0.1 pt

Hint: After this task you should tighten the fixing screws of the vernier.
Moreover, tighten the screw of the photoresistor holder to fix it and do not remove it
during the experiment.

Step 3:
Move the rotatable arm to the region of the first-order diffraction. Find the angle at
which the resistance of the photoresistor is minimum (maximum light intensity).
Using the balancing screws, you can slightly change the tilt of the grating s stage, to
achieve an even lower resistance value.
1-c

(1)
Report the minimum value of the observed resistance ( Rmin
) in
its appropriate box.

0.1 pt

It is now necessary to check the perpendicularity of the grating for zero adjustment,
again. For this you must use the reflection-coincidence method of Step 1.
Important: From here onwards carry out the experiment in dark (close the cover).
Measurements: Screw the sample-holder onto the rotatable arm. Before you start the
measurements, examine the appearance of your semiconductor film (sample). Place
the sample in front of the entrance hole S1 on the rotatable arm such that a uniformly
coated part of the sample covers the hole. To make sure that every time you will be
working with the same part of the sample make proper markings on the sample holder
and the rotatable arm with blank labels.
Attention: At higher resistance measurements it is necessary to allow the
photoresistor to relax, therefore for each measurement in this range wait 3 to 4
minutes before recording your measurement.

1-d

Measure the resistance of the photoresistor for the uncoated


glass substrate and the glass substrate coated with semiconductor
layer as a function of the angle
(the value read by the
goniometer for the angle between the photoresistor and your
specified optical axis). Then fill in Table 1d. Note that you need
at least 20 data points in the range you found in Step 1b. Carry
out your measurement using the appropriate range of your
ohmmeter.
Consider the error associated with each data point. Base your

2.0 pt

1.0 pt
5

answer only on your direct readings of the ohmmeter.


Step 4:
The precision obtained so far is still limited since it is impossible to align the
rotatable arm with the optical axis and/or position the grating perpendicular to the
optical axis with 100% precision. So we still need to find the asymmetry of the
measured transmission at both sides of the optical axis (resulting from the deviation
of the normal to the grating surface from the optical axis ( o )).
To measure this asymmetry, follow these steps:
First, measure T film at
1-e

at some other angles around 20 . Complete Table 1e (you can


use the values obtained in Table 1d).
Draw T film versus and visually draw a curve.

On your curve find the angle

20 o (

you measured at
with

20 . Then, obtain values for T film

20 as

0.6 pt

for which the value of T film is equal to the T film that


|T

film

film

( 20 )

). Denote the difference of this angle

, in other words:

20
1-e

0.6 pt

Report the value of

(6)

in the specified box.

0.2 pt

Then for the first-order diffraction, Eq. (3) can be simplified as follows:
d sin(
/ 2) , (7)
where is the angle read on the goniometer.
2. Calculations:

2-a

Use Eq. (7) to express


in terms of the errors of the other
parameters (assume d is exact and there is no error is associated
with it). Also using Eqs. (1), (2), and (5), express T film in terms
of R and

0.6 pt

R.

2-b

Report the range of values of


diffraction.

over the region of first-order

2-c

Based on the measured parameters in Task 1, complete Table


2c for each . Note that the wavelength should be calculated using
Eq. (7).

0.3 pt

2.4 pt

1
1
Plot Rglass
and R film
as a function of wavelength together on

2-d

the same diagram. Note that on the basis of Eq. (2) behaviors of
1
1
Rglass
and R film
can reasonably give us an indication of the way

1.5 pt

I glass and I film behave, respectively.


In Table 2d, report the wavelengths at which Rglass and

R film attain their minimum values.

0.4 pt
6

For the semiconductor layer (sample) plot T film as a function of


2-e

wavelength. This quantity also represents the variation of the film


transmission in terms of wavelength.

1.0 pt

3. Data analysis:
By substituting
1 2 and A 0.071 ((eV)1/2/nm) in Eq. (4) one can find values for
E g and t in units of eV and nm, respectively. This will be accomplished by plotting a
suitable diagram in an x y coordinate system and doing an extrapolation in the
region satisfying this equation.

3-a

By assuming x h and y ( t h ) 2 and by using your


measurements in Task 1, fill in Table 3a for wavelengths around
530 nm and higher. Express your results ( x and y ) with the
correct number of significant figures (digits), based on the
estimation of the error on one single data point.
Note that h should be calculated in units of eV and wavelength
in units of nm. Write the unit of each variable between the
parentheses in the top row of the table.

3-b

Plot y versus x .
Note that the y parameter corresponds to the absorption of the
film. Fit a line to the points in the linear region around 530 nm.
Specify the region where Eq. (4) is satisfied, by reporting the
values of the smallest and the largest x-coordinates for the data
points to which you fit the line.

3-c

Call the slope of this line m , and find an expression for the
film thickness ( t ) and its error ( t ) in terms of m and A (consider
A to have no error).

3-d

Obtain the values of E g and t and their associated errors in


units of eV and nm, respectively. Fill in Table 3d.

2.4 pt

2.6
pt

0.5 pt

3.0 pt

Some useful physical constants required for your analysis:


Speed of the light:

c 3.00 10 8 m/s

Plank s constant:
Electron charge:

h 6.63 10

34

e 1.60 10

19

J.s
C

This document was created with Win2PDF available at http://www.daneprairie.com.


The unregistered version of Win2PDF is for evaluation or non-commercial use only.

EXPERIMENTAL COMPETITION
Problems and tasks
9th Asian Physics Olympiad
Ulaanbaatar, Mongolia (April 24, 2008 )

THE PHYSICS OF SOUND WAVE (20 points)


Our environment is filled with sound and sound effects. This experimental problem related to
the ultrasonic and sonic effects and consists of four parts. In part 1 the characteristics of the
ultrasonic microphone system should be acquired. Afterwards we will observe and explain
interference phenomena, then will study the Doppler Effect, and finally will determine the
threshold of human hearing and resolving power.
List of Apparatus and materials
label
A
B
C
D
E
F
G
H
I
J

Component
Function Generator
Ultrasonic Amplifier
AC Millivoltmeter
Frequency counter
variable DC Power Supply
Ultrasonic transducer for
Source (Red labeled)
Ultrasonic microphone for
Detector ( Blue labeled)
Source holder
Detector holders
tal mirror/ Reflector

Qty
1
1
1
1
1
1

label
K
L
M
N
O
P

Optical bench

1
1
1

Variable resistor (rheostat)


Ruler

Component
Mirror magnetic holder with ruler
Rotating disc
A motor attached to the optical bench
Connection coaxial cables
Rotating holder with angle meter
Stereo Headphones

Qty
1
1
4
1
1

E
R

Figure 1. Apparatus and materials

Apparatuses and materials (continued)


1. Instructions for Synthesized Function Generator (Item A)
Main
Displa
y

Power
Switc
h

Frequency
Adjustment
Knob

Entry
Keys

Duty
Control

Shift Output
Key ON/OFF
key

Amplitude
Main
Control
Outpu
t
Offset
TTL
Contro
Outpu
l
t

The power button may be pressed for ON and pressed again for OFF.
Select the frequency range and use proper button: to move the cursor in the main
display (frequency editing point) use key 4 or 5 after pressing SHIFT key; to
specify the frequency unit use key 9 for kHz with SHIFT key.
The frequency will be displayed in the main display. To see voltage of the signal
produced by the generator, use V/F button with SHIFT key.

Use the frequency adjustment knob to tune proper frequency.


To change the amplitude of the signal, you can use amplitude control knob.
You should switch on Output control ON/OFF key, otherwise the generator will not
send the signal to the microphone.
Do not use the Duty control in sine wave situation.

2. Amplifier Adaptor (item B)


The amplifier itself does not do anything, it serves merely as an adaptor for connecting
plugs which otherwise would not match.

3. AC Millivoltmeter (Item C)

Connect BNC type cable from Amplifier Adaptor to INPUT terminal.


Set the RANGE to 100 V and switch on the power.
Alter the RANGE selector switch until the pointer is at a position which located at
1/3 of the full scale, the reading can be taken easily. The number written in the
selector switch indicates the maximum measurable value in that range.

4. Frequency counter (Item D)

Signal
input

Freq.
button

Time
button

On/Off
button

Connect the Signal cable to the Signal input connector


Turn ON the frequency counter
Press Freq. button and Time button
Frequency counter will be ready for measurement.

Caution: Be sure switch off the power of equipments before plug


in/out all connections, otherwise damage of equipments or sensors
will be occurred.

PART 1. Characteristics of the microphone system ( 3 points )


Introduction
The property, called piezoelectricity, provides a convenient coupling between mechanical
oscillations of crystal, which occur at a very sharply defined frequency, and the electrical
properties of a circuit of which the crystal is a part. Piezoelectric materials are used to convert
electric and sound signals into each other. But they are distinguished by having a well
specified working frequency range. Therefore, in this experimental part, we have to determine
physical properties of the microphone which is made of piezoelectric material, before using it.
List of components
1. Function Generator (FG)

5.

2.

Ultrasonic Amplifier

6.

3.
4.

AC Millivoltmeter (MV)
Rotating holder with angle meter

7.
8.

Ultrasonic transducer for Source (S) with


holder
Ultrasonic microphone for Detector (D)
with holder
Two connection coaxial cables
Calculator

Experiments and procedures


We will use two ultrasonic microphones, one of them will be used as signal source (red
labeled) and the other will be used as a signal receiver (blue labeled). The source receives the
signal from the function generator (FG) through the amplifier and converts the signal into a
sound wave. The signal receiver (it can be called as detector) that connected to a preamplifier,
receives the sound wave emitted from the source, and converts it to electrical signal. The
voltage of the output signal will be measured by AC Millivoltmeter.
Hints: The voltage of the signal given to the source (S) should be adjusted around 1V. If
it is more than this the ultrasonic wave will be saturated and it will affect your result.
The microphones are placed in the rotating holder and connected to an electrical cycle
with ultrasonic amplifier scheme. The distance between the source and the detector
should not be changed during the experiment. And please, make sure that the
microphones are placed parallel to holder base and the axes of the microphones are laid
on the same line.
1a. ( 1.5 points ) By changing the frequency of the signal from the FG, measure the voltage
of the output signal that converted by the detector. Measure in the range of input frequency
from 30 kHz to 50 kHz, and make sure the frequency range of the signal sent to the source in
a given range. Otherwise, the microphone will be damaged or out of order. Set the voltage
of the signal from the FG around 1V. Measured data will be filled into table 1A. Plot the
graph using measured data. Plot a measured rms signal voltage vs. frequency, determine the
cut-off frequencies f1, f2 where the measured rms signal voltage drops
) of the
maximum measured value. Hence, determine the bandwidth
=f2-f1. Determine the working
frequency fw (at which the voltage of the signal from the detector will be at maximum) from
the obtained results.
1b. ( 1.5 points ) Set the frequency from the FG to the working frequency. Determine the
angular dependence of the intensity of the output signal on the position of the detector relative
to the source. Write the measured data in a Table 1B. Plot a graph of the dependence of the
voltage ratio
on an angle , where A() and A(0) are voltages of the output signal
at an angle and 0, respectively. The direction
has maximal detection, and is called
the axis of the source. Find angular values at which the voltage of the detected signal
decreases 2 and 3 times.
5

PART 2. Interference of Waves ( 6 points )


Introduction
A standing wave pattern is a vibrational pattern created within a medium when the reflected
waves from mirror to interfere with incident waves from the source. The waves are interfering
in such a manner that there are points of no displacement produced at the same positions
along the medium. These points along the medium are known as nodes. There are other
points along the medium which undergo vibrations between a large positive and large
negative displacement. These points are known as antinodes.
List of components
1.
Function Generator (A)
2.
3.
4.
5.

6.

Metal mirror (J) and magnetic holder with


ruler (K)
Optical bench (R)
Two connection coaxial cables (N)
Calculator (X)

Ultrasonic Amplifier (B)


7.
AC Millivoltmeter (C)
8.
Ultrasonic transducer for Source
9.
(F) with holder (H)
Ultrasonic microphone for Detector
(G) with holder (I)

Description of the Experiments


2a. Study of the Wave Interference ( 1.6 points )
Using the instruments 1-8 shown in the above list, assemble the experimental set up shown in
Fig. 2.1 and study the interference of the wave. To reduce possible undesirable interference be
cautious and place other instruments away from detector. Connect the functional generator to
the Source and set the frequency at 40kHz.

Top view

Side view

Fig. 2.1. Study of the wave interference.


Position A: The detector is placed between Source and Mirror.
Position B: The detector is placed behind the Source.
S- Source, M-Mirror, D-Detector

2a.1. Place the detector in Position A which is shown in Figure 2.1 and observe the
dependence of the detected signal level on the positions of S, M and D. When the detector is
too close to the source there will occur unclear effects, therefore do not measure in close
distance to the source. Remember the detector has angular sensitivity. The detector should be
placed for optimum measurement.
2a.2. Place the detector in Position B which is shown in Figure 2.1 and observe the
dependence of the detected signal level on the positions of M and D. The position of S should
be fixed.
2a.3. Measurement of Wavelength.
As you did in the experiment 2a.1 fixing the positions of S and D and by moving M,
experimentally determine the wavelength of the ultrasonic wave.
2b. Find experimentally the correct answers to the following statements. ( 2.8 points )
Write down for correct answers or X for incorrect answers below the label of the
chosen statements in your Answer sheet.
a. The standing wave will be observed between the S and M (Fig. 2.1). This standing
wave will occur for any values of the distance between S and M.
b. The standing wave will be observed between the S and M. This standing wave will
occur only when the distance between S and M equal to n/2. Where n is integer.
c. In the both positions A and B the detector will detect nodes and antinodes of the
standing waves. It can be proved by moving the positions of S and M.
d. The standing wave will occur for any values of the distance between S and M. It
can be observed experimentally by moving the position of D. In position B
detector will detect the high level of signal when the distance between S and M is
SM=n/4. Where n is odd integer.
e. The positions of the nodes and antinodes are immovable with respect to the lab
frame (bench) when the S and M are moved.
f. When the distance between S and M increased the level of the reflected wave in
position B will be periodic with decreasing amplitude.
g. The standing wave will occur only between S and M, but behind the Source the
reflected waves will be observed.
2c. Study of the Standing Wave ( 1.6 points )
In position B if the M is moved detector will detect the maximum minimum values of signal
level. In these cases identify either node or antinode occurred on the surface of the Mirror and
Source. If you identify the node write down N and if you identify antinode write down A
in the table.
Condition
On the surface of Mirror
On the surface of Source
When D detects maximum
When D detects minimum

PART 3. The Doppler effect

( 8 points )

The observed and measured frequency of a signal changes by virtue of relative motion between the
source and the observer. This is known as the Doppler effect.
The observed frequency is given by the formulas
(Source and Detector receding)

(1)

(Source and Detector approaching)

(2)

where
frequency of the wave emitted by the Source
speed of sound in air
velocity of the Detector
velocity of the Source

List of components
1.
Function Generator (FG)
2.
Ultrasonic Amplifier
3.
AC Millivoltmeter (MV)
4.
Frequency counter (FC)
5.
Ultrasonic transducer for Source
(S) with holder
6.
Ultrasonic microphone for Detector
(D) with holder

7.
8.
9.
10.
11.

Mtor
Rotating disc
DC Power Supply
Optical bench
Connection coaxial cables

Description of the Experiment


The source (S) is placed so that the ultrasound strikes the tilts of the rotating disc from the
side, and the Detector is placed so that reflected ultrasound is detected more effectively. The
source is connected to the output of the Function Generator, and the Detector is to the
millivoltmeter to measure which will detect the sound intensity level, as explained in Part 1.
Before beginning the experiment, you should check changes in the sound intensity level by
moving the disk forward and back manually. If changes in the sound intensity level are small
then you should adjust the position of the sound source and the receiver properly. If the
apparatus is not adjusted properly there may occur an error in the measurements of the
sound intensity level and frequency.
Switch on the motor of the rotating disk
Observe the changes in the sound frequency by gently increasing the voltage on the
motor of the rotating disk
It is necessary to prepare the instruments to the measurement for Part 3, it is advised to
use results from Part 1 and Part 2. For example, to set the working frequency etc. If
you could not determine the working frequency use 40 kHz as a working frequency.
Make sure, the voltage of the signal from FG is not saturated.

3a. Formulas for the Doppler shift of sound from the rotating disk. ( 1.5 points )
A sound wave from the Source reflects to saw tooth of the rotating disk (Figure 1) and
the Doppler effect will occur. If we denote as the velocity of motion of the saw
tooth in R direction, obtain a formula for in terms of , and , for this case.
Make it sure in the experimental setup, an angle dependence is negligible, or an
incident and a reflected angles should be less than 5.
From obtained expression, write a simplified formulae for f/f0 in function of v/c using
the abbreviation f=|f-fo|, for the case v<<c, where f0 is the working frequency. Whole
derivation procedure should be written on the Answer sheet.

Figure 1. The saw tooth of the rotating disk


3b. Calculations for the Doppler effect from the rotating disk ( 1.4 points )
Derive the expression for the radial velocity v of the middle point of saw tooth in terms of
angular velocity of rotating disk and l. (The saw tooths height is l=4.37 0.05 mm,
for further calculation).
3c. The Doppler effect in dependence on the motor voltage ( 2.8 points )
Measure frequency of the ultrasonic signal detected by the detector as a function of the
motor voltage up to 16V. Plot a graph
vs U . From the graph at large U,
determine the value of threshold voltage Ut from which the exploration of the f goes
to zero, and the slope of the graph with an measurement error.
Hints: Make the rotating disk to rotate in Clockwise (CW) direction and dont change it
further. Choose the detector position that it measures more stable and more effective.
To save your time you can measure electric current for Task 3d, simultaneously with the
measurement of motor voltage.
Make sure equipments that you use in Experimental part, are calibrated and measured
very well before using them. Each system has own specification which is given on special
sheet. For example, at 10V voltage, the frequency shift of the Doppler effect is given in
the specification sheet. Using these experimental results, you should calibrate your
system. Otherwise, the measurement will be incorrect.
3d. Linear dependence of U ( 0.8 points )
From the specification sheet, find angular velocity , voltage U and electric current I
with its measurement errors (for CW direction) and calculate numerical coefficients
and errors for the linear dependence of U. Assume that at high voltages, is
9

approximately proportional to the voltage. You should write the obtained values from
the specification on Answer Sheet.
3e. Speed of sound in air from the Doppler effect ( 1.5 points )
Put together the functional dependences
and
and find
from here the speed of sound in air with a measurement error. Using experimental and
theoretical values of the speed of sound in air find the relative error of your
experiment:
*100 % .
where ctheor = 343 m/s speed of sound in air in audible range at 20 C. You can use
this value only in this part
PART 4: Threshold and Resolving Power of Hearing ( 3 points )
The threshold of hearing is the sound intensity in W/m2 at which our ear can barely hear it.
This threshold depends on sound frequency. The threshold of hearing at about 2000 Hz is
equal to 10-12 W/m2. The ratio log10(I/Io) is called as sound intensity level and is measured in
units of Bell, abbreviated as B. In the practice it is more convenient to use the ratio (in dB)
= 10 log10(I/Io). The human ear can hear sounds from about 20 Hz to about 20,000 Hz. It
called that the audible range.
List of Apparatuses and Accessories
The following items are required for this experiment (Figure. 4.1)
1) Function generator (a)
2) AC Millivoltmeter (b)
3) Cable connectors (c, d)
4) Variable resistor (rheostat) (e)
5) Headphone (f)
Preparation
1. Connect the experimental apparatus as showed in Figure 4.2.
2. Switch on the power of the AC millivoltmeter and Function Generator.
3. Rotate the ADJ button and set it to the middle position.
(A)

(B)
(b)
(a)
(f)

(d)
(e)

(c)

Figure 4.1 (A). Connections of the apparatuses. (B) Connections of the leads to the rheostat: plug in
the red lead from the function generator to the red socket and black lead to the black socket; connect
the red lead of AC millivoltmeter to yellow socket and the black lead to the black socket; and plug in
headphone lead to the headphone socket.

10

Function generator
input
frequency

power

MAIN
50

red

cable

Rheostat
Headphone
socket

red
yellow

red
cable

black
black

Volume controller
headphone

Figure 4.2. The connection scheme.


Measurements
4a. The frequency region of ones own hearing ( 0.5 points )
A. Determination of the lowest frequency to be heard, flow; and B. Determination of the
highest frequency to be heard, fhigh
1. Set the volume level of the headphone to the maximum using volume controller of
rheostat.
2. Find the lowest frequency, flow, to be heard. In order to do it you should change both of
sound level and frequency. To change the frequency use the frequency button of Function
Generator. To change the sound level you can use the rheostat and the AMPL ADJ button
of the Function Generator.
3. Use similar procedures and find the fhigh

4b. The frequency region to be heard the best ( 1 point )


1. Lower the sound intensity step by step. For each step find the flow and fhigh. Find out a
region where you can hear lowest level of sound intensity, as possible as can. The lower limit
of this region is lowest frequency of the frequency region to be heard the best, fth1; and upper
limit of this region is the highest frequency of the frequency region to be heard the best, fth2.
Average of this two limits or the most sensitive frequency to be heard is

11

4c. Resolving power (R) of ear for different frequencies of sound. ( 1.0 point)
Resolving power of ear is an ability to distinguish two close frequencies.
1. Set the frequency of the generator to fth.
2. While hearing with the headphone, change the intensity to the level that you can hear well.
3. Change the frequency a little. If you can not hear a difference between fth and the changed
frequency, make the difference a little more. Find a frequency that you can hear as a
different from the fth . Difference between the fth and the changed frequency is resolveable
frequency of your ear at fth.
for the fth.
4. Calculate resolveable frequency (f) and resolving power

4d. Find the minimum speed of the mirror which gives Doppler effect detected by ones own
ears in the above frequency region. ( 0.5 points )
Evaluate the speed of the mirror and error of it using the following formula:

where c = 343 m/s speed of sound in air in audible range at 20 C. You can use this value
only in this part.

12

39th International Physics Olympiad - Hanoi - Vietnam - 2008

Experimental Problem

DIFFERENTIAL THERMOMETRIC METHOD


In this problem, we use the differential thermometric method to fulfill the two
following tasks:
1. Finding the temperature of solidification of a crystalline solid substance.
2. Determining the efficiency of a solar cell.
A. Differential thermometric method
In this experiment forward biased silicon diodes are used as temperature sensors to
measure temperature. If the electric current through the diode is constant, then the voltage
drop across the diode depends on the temperature according to the relation

V (T ) = V (T0 ) (T T0 )

(1)

where V (T ) and V (T0 ) are respectively the voltage drops across the diode at
temperature T and at room temperature T0 (measured in oC), and the factor

= ( 2.00 0.03) mV/ o C

(2)

The value of V (T0 ) may vary slightly from diode to diode.


If two such diodes are placed at different temperatures, the difference between the
temperatures can be measured from the difference of the voltage drops across the two
diodes. The difference of the voltage drops, called the differential voltage, can be
measured with high precision; hence the temperature difference can also be measured
with high precision. This method is called
the differential thermometric method. The
R1
R2
electric circuit used with the diodes in this
E
experiment is shown in Figure 1. Diodes
V
D1 and D2 are forward biased by a 9V
battery, through 10 k resistors, R1 and

R2 . This circuit keeps the current in the

V1

D1

D2

V2

Figure 1. Electric circuit of the diode

two diodes approximately constant.


If the temperature of diode D1 is T1 and that of D2 is T2 , then according to (1), we
have:
1

39th International Physics Olympiad - Hanoi - Vietnam - 2008

Experimental Problem

V1 (T1 ) = V1 (T0 ) (T1 T0 )


and

V2 (T2 ) = V2 (T0 ) (T2 T0 )


The differential voltage is:

V = V2 (T2 ) V1 (T1 ) = V2 (T0 ) V1 (T0 ) (T2 T1 ) = V (T0 ) (T2 T1 )


V = V (T0 ) T

(3)

in which T = T2 T1 . By measuring the differential voltage V , we can determine


the temperature difference.
To bias the diodes, we use a circuit box, the diagram of which is shown in Figure 2.

Blue

To D1 - Blue

V
10 k

To D2 - Red
10 k

Common- Black

Red
Red
V2
Black

9V
Figure 2. Diagram of the circuit box
(top view)
The circuit box contains two biasing resistors of 10 k for the diodes, electrical leads
to the 9 V battery, sockets for connecting to the diodes D1 and D2, and sockets for
connecting to digital multimeters to measure the voltage drop V2 on diode D2 and the
differential voltage V of the diodes D1 and D2.

39th International Physics Olympiad - Hanoi - Vietnam - 2008

Experimental Problem

B. Task 1: Finding the temperature of solidification of a crystalline substance


1. Aim of the experiment
If a crystalline solid substance is heated to the melting state and then cooled down,
it solidifies at a fixed temperature Ts , called temperature of solidification, also called the
melting point of the substance. The traditional method to determine Ts is to follow the
change in temperature with time during the cooling process. Due to the fact that the
solidification process is accompanied by the release of the latent heat of the phase
transition, the temperature of the substance does not change while the substance is
solidifying. If the amount of the substance is large enough, the time interval in which the
temperature remains constant is rather long, and one can easily determine this
temperature. On the contrary, if the amount of substance is small, this time interval is too
short to be observed and hence it is difficult to determine Ts .
In order to determine Ts in case of small amount of substance, we use the
differential thermometric method, whose principle can be summarized as follows. We use
two identical small dishes, one containing a small amount of the substance to be studied,
called the sample dish, and the other not containing the substance, called the reference
dish. The two dishes are put on a heat source, whose temperature varies slowly with time.
The thermal flows to and from the two dishes are nearly the same. Each dish contains a
temperature sensor (a forward biased silicon diode). While there is no phase change in
the substance, the temperature Tsamp of the sample dish and the temperature Tref of the
reference dish vary at nearly the same rate, and thus T = Tref Tsamp varies slowly with

Tsamp . If there is a phase change in the substance, and during the phase change
Tsamp does not vary and equals Ts , while Tref steadily varies, then T varies quickly.
The plot of T versus Tsamp shows an abrupt change. The value of Tsamp corresponding
to the abrupt change of T is indeed Ts .
The aim of this experiment is to determine the temperature of solidification Ts of a

39th International Physics Olympiad - Hanoi - Vietnam - 2008

Experimental Problem

pure crystalline substance, having Ts in the range from 50oC to 70oC, by using the
traditional and differential thermal analysis methods. The amount of substance used in
the experiment is about 20 mg.
2. Apparatus and materials
1. The heat source is a 20 W halogen lamp.
2. The dish holder is a bakelite plate with a square hole in it. A steel plate is fixed on
the hole. Two small magnets are put on the steel plate.
3. Two small steel dishes, each contains a silicon diode soldered on it. One dish is
used as the reference dish, the other - as the sample dish.
Cover

Steel plate

Red
Black
Blue

Sample dish
D2

D1

Ref. dish

Magnet
s

12V/20W bulb

Figure 3. Apparatus for measuring the solidification temperature


Each dish is placed on a magnet. The magnetic force maintains the contact between
the dish, the magnet and the steel plate. The magnets also keep a moderate thermal
contact between the steel plate and the dishes.
A grey plastic box used as a cover to
protect the dishes from the outside influence.
Figure 3 shows the arrangement of the
D1
D2
dishes and the magnets on the dish holder
and the light bulb.
4. Two digital multimeters are used as
voltmeters. They can also measure room
Red
temperature by turning the Function selector
Black
to the oC/oF function. The voltage function
of the multimeter has an error of 2 on the
last digit.
Note: to prevent the multimeter (see
Figure 9) from going into the Auto power

Blue

Figure 4. The dishes on the dish holder


(top view)

39th International Physics Olympiad - Hanoi - Vietnam - 2008

Experimental Problem

off function, turn the Function selector from OFF position to the desired function while
pressing and holding the SELECT button.
5. A circuit box as shown in Figure 2.
6. A 9 V battery.
7. Electrical leads.
8. A small ampoule containing about 20 mg of the substance to be measured.
9. A stop watch
10. A calculator
11. Graph papers.
3. Experiment
1. The magnets are placed on two equivalent locations on the steel plate. The reference
dish and the empty sample dish are put on the magnets as shown in the Figure 4. We use
the dish on the left side as the reference dish, with diode D1 on it (D1 is called the
reference diode), and the dish on the right side as the sample dish, with diode D2 on it (D2
is called the measuring diode).
Put the lamp-shade up side down as shown in Figure 5. Do not switch the lamp on.
Put the dish holder on the lamp. Connect the apparatuses so that you can measure the
voltage drop on the diode D2, that is Vsamp = V2 , and the differential voltage V .
In order to eliminate errors due to the warming up period of the instruments and
devices, it is strongly recommended that the complete measurement circuit be switched
on for about 5 minutes before starting real experiments.

Figure 5.
Using the halogen lamp as a heat source
1.1. Measure the room temperature T0 and the voltage drop Vsamp (T0 ) across
diode D2 fixed to the sample dish, at room temperature T0 .

1.2. Calculate the voltage drops Vsamp 50o C , Vsamp 70o C

and Vsamp 80o C

on the measuring diode at temperatures 50oC, 70oC and 80oC, respectively.

39th International Physics Olympiad - Hanoi - Vietnam - 2008

Experimental Problem

2. With both dishes still empty, switch the lamp on. Follow Vsam. When the temperature
of the sample dish reaches Tsamp ~ 80oC, switch the lamp off.
2.1. Wait until Tsamp ~ 70oC, and then follow the change in Vsamp and V with
time, while the steel plate is cooling down. Note down the values of Vsamp and V
every 10 s to 20 s in the table provided in the answer sheet. If V varies quickly, the
time interval between consecutive measurements may be shorter. When the temperature
of the sample dish decreases to Tsamp ~ 50oC, the measurement is stopped.
2.2. Plot the graph of Vsamp versus t, called Graph 1, on a graph paper provided.
2.3. Plot the graph of V versus Vsamp , called Graph 2, on a graph paper provided.
Note: for 2.2 and 2.3 do not forget to write down the correct name of each graph.
3. Pour the substance from the ampoule into the sample dish. Repeat the experiment
identically as mentioned in section 2.
3.1. Write down the data of Vsamp and V with time t in the table provided in the
answer sheet.
3.2. Plot the graph of Vsamp versus t, called Graph 3, on a graph paper provided.
3.3. Plot the graph of V versus Vsamp , called Graph 4, on a graph paper provided.
Note: for 3.2 and 3.3 do not forget to write down the correct name of each graph.
4. By comparing the graphs in section 2 and section 3, determine the temperature of
solidification of the substance.
4.1. Using the traditional method to determine Ts : by comparing the graphs of

Vsamp versus t in sections 3 and 2, i.e. Graph 3 and Graph 1, mark the point on Graph 3
where the substance solidifies and determine the value Vs (corresponding to this point)
of Vsamp .

39th International Physics Olympiad - Hanoi - Vietnam - 2008

Experimental Problem

Find out the temperature of solidification Ts of the substance and estimate its error.
4.2. Using the differential thermometric method to determine Ts : by comparing the
graphs of V versus Vsamp in sections 3 and 2, i.e. Graph 4 and Graph 2, mark the
point on Graph 4 where the substance solidifies and determine the value Vs of Vsamp .
Find out the temperature of solidification Ts of the substance.
4.3. From errors of measurement data and instruments, calculate the error of Ts
obtained with the differential thermometric method. Write down the error calculations
and finally write down the values of Ts together with its error in the answer sheet.

C. Task 2: Determining the efficiency of a solar cell under illumination of an


incandescent lamp
1. Aim of the experiment
The aim of the experiment is to determine the efficiency of a solar cell under
illumination of an incandescent lamp. Efficiency is defined as the ratio of the electrical
power that the solar cell can supply to an external circuit, to the total radiant power
received by the cell. The efficiency depends on the incident radiation spectrum. In this
experiment the radiation incident to the cell is that of an incandescent halogen lamp. In
order to determine the efficiency of the
solar cell, we have to measure the
irradiance E at a point situated under
the lamp, at a distance d from the lamp
along the vertical direction, and the

d = 12 cm

maximum power Pmax of the solar cell


when it is placed at this point. In this
experiment, d = 12 cm (Figure 6).
Irradiance E can be defined by:

E = /S
in which is the radiant flux (radiant
power), and S is the area of the

Figure 6.
Using the halogen lamp
as a light source

illuminated surface.

39th International Physics Olympiad - Hanoi - Vietnam - 2008

Experimental Problem

2. Apparatus and materials


1. The light source is a 20W halogen lamp.
2. The radiation detector is a hollow cone made of copper, the inner surface of it is
blackened with soot (Figure 7). The cone is incompletely thermally isolated from the
surrounding. In this experiment, the detector is considered an ideal black body. To
measure temperature, we use silicon diodes. The measuring diode is fixed to the radiation
detector (D2 in Figure 1 and Figure 7), so that its temperature equals that of the cone. The
reference diode is placed on the inner side of the wall of the box containing the detector;
its temperature equals that of the surrounding. The total heat capacity of the detector (the
cone and the measuring diode) is C = ( 0.69 0.02 ) J/K . The detector is covered by a
very thin polyethylene film; the radiation absorption and reflection of which can be
neglected.

Thermal insulator

Common
Reference
diode D1

Blue

Black

Measuring
diode D2

Red

Figure 7. Diagram of the radiation detector


3. A circuit box as shown in Figure 2.
4. A piece of solar cell fixed on a plastic box
(Figure 8). The area of the cell includes some metal
connection strips. For the efficiency calculation these
strips are considered parts of the cell.
5. Two digital multimeters. When used to
measure the voltage, they have a very large internal
resistance, which can be considered infinitely large.
When we use them to measure the current, we cannot
neglect their internal resistance. The voltage function
of the multimeter has an error of 2 on the last digit.

Red

Black

Figure 8.
The solar cell

39th International Physics Olympiad - Hanoi - Vietnam - 2008

Experimental Problem

The multimeters can also measure the room temperature.


Note: to prevent the multimeter (see Figure 9) from going into the Auto power off
function, turn the Function selector from OFF position to the desired function while
pressing and holding the SELECT button.
6. A 9 V battery
7. A variable resistor.
8. A stop watch
9. A ruler with 1mm divisions
10. Electrical leads.
11. Graph papers.
3. Experiment
When the detector receives energy from radiation, it heats up. At the same time, the
detector loses its heat by several mechanisms, such as thermal conduction, convection,
radiation etc...Thus, the radiant energy received by detector in a time interval dt is equal
to the sum of the energy needed to increase the detector temperature and the energy
transferred from the detector to the surrounding:

dt = CdT + dQ
where C is the heat capacity of the detector and the diode, dT - the temperature
increase and dQ - the heat loss.
When the temperature difference between the detector and the surrounding

T = T T0 is small, we can consider that the heat dQ transferred from the detector to
the surrounding in the time interval dt is approximately proportional to T and dt ,
that is dQ = k Tdt , with k being a factor having the dimension of W/K. Hence,
assuming that k is constant and T is small, we have:

or

dt = CdT + k Tdt = Cd (T ) + k Tdt

d ( T ) k
+ T =
dt
C
C

(4)

The solution of this differential equation determines the variation of the temperature
difference T with time t, from the moment the detector begins to receive the light with
a constant irradiation, assuming that at t=0, T =0

T (t ) =
k

t
C
1 e

(5)

When the radiation is switched off, the mentioned above differential equation
becomes

39th International Physics Olympiad - Hanoi - Vietnam - 2008

Experimental Problem

d (T ) k
+ T = 0
dt
C

(6)

and the temperature difference T varies with the time according to the following
formula:

T (t ) = T ( 0 ) e

k
t
C

(7)

where T (0) is the temperature difference at t = 0 (the moment when the measurement
starts).
1. Determine the room temperature T0 .
2. Compose an electric circuit comprising the diode sensors, the circuit box and the
multimeters to measure the temperature of the detector.
In order to eliminate errors due to the warming up period of the instruments and
devices, it is strongly recommended that the complete measurement circuit be switched
on for about 5 minutes before starting real experiments.
2.1. Place the detector under the light source, at a distance of d = 12 cm to the lamp.
The lamp is off. Follow the variation of V for about 2 minutes with sampling intervals
of 10 s and determine the value of V (T0 ) in equation (3).
2.2. Switch the lamp on to illuminate the detector. Follow the variation of V . Every
10-15 s, write down a value of V in the table provided in the answer sheet. (Note:
columns x and y of the table will be used later in section 4.). After 2 minutes, switch the
lamp off.
2.3. Move the detector away from the lamp. Follow the variation of V for about 2
minutes after that. Every 10-15 s, write down a value of V in the table provided in the
answer sheet. (Note: columns x and y of the table will be used later in section 3.).
Hints: As the detector has a thermal inertia, it is recommended not to use some data
obtained immediately after the moment the detector begins to be illuminated or ceases to
be illuminated.
3. Plot a graph in an x-y system of coordinates, with variables x and y chosen
appropriately, in order to prove that after the lamp is switched off, equation (7) is satisfied.
3.1. Write down the expression for variables x and y.
3.2. Plot a graph of y versus x, called Graph 5.
3.3. From the graph, determine the value of k .
4. Plot a graph in an x-y system of coordinates, with variables x and y chosen

10

39th International Physics Olympiad - Hanoi - Vietnam - 2008

Experimental Problem

appropriately, in order to prove that when the detector is illuminated, equation (5) is
satisfied.
4.1. Write down the expressions for variables x and y.
4.2. Plot a graph of y versus x, called Graph 6.
4.3. Determine the irradiance E at the orifice of the detector.
5. Put the solar cell to the same place where the radiation detector was. Connect the solar
cell to an appropriate electric circuit comprising the multimeters and a variable resistor
which is used to change the load of the cell. Measure the current in the circuit and the
voltage on the cell at different values of the resistor.
5.1. Draw a diagram of the circuit used in this experiment.
5.2. By rotating the knob of the variable resistor, you change the value of the load.
Note the values of current I and voltage V at each position of the knob.
5.3. Plot a graph of the power of the cell, which supplies to the load, as a function of
the current through the cell. This is Graph 7.
5.4. From the graph deduce the maximum power Pmax of the cell and estimate its error.
5.5. Write down the expression for the efficiency of the cell that corresponds to the
obtained maximum power. Calculate its value and error.

11

39th International Physics Olympiad - Hanoi - Vietnam - 2008

Experimental Problem

Contents of the experiment kit (see also Figure 10)


1

Halogen lamp 220 V/ 20 W

Stop watch

Dish holder

10

Calculator

Dish

11

Radiation detector

Multimeter

12

Solar cell

Circuit box

13

Variable resistor

9 V battery

14

Ruler

Electrical leads

15

Box used as a cover

Ampoule with substance to be


measured

Note: to prevent the multimeter (see Figure 9) from going into the Auto power off
function, turn the Function selector from OFF position to the desired function while
pressing and holding the SELECT button.

Select

Function selector

Figure 9. Digital multimeter

12

39th International Physics Olympiad - Hanoi - Vietnam - 2008

Experimental Problem

8
14

10
4
2

12

13
5
11
15

Figure 10. Contents of the experiment kit

13

Experimental competition

Problem 1

28 April 2009
Page 1 of 4
-------------------------------------------------------------------------------------------------------------------------

Problem 1: The Earths Horizontal Magnetic Field


This is to determine the horizontal component of the Earths magnetic field BH using smallamplitude oscillation of a cylindrical bar magnet. The magnet is to oscillate in the combined static
fields of the Earth and that due to a square coil.
Y

supporting string

a
a

a
Z

P
a

square coil of
sides a and N turns

X
magnet of length L
and radius r

i
i

Figure 1

The experiment is to be done in three sections. Section I is a derivation of formulae to be used in


Section III.

Apparatus
Each student is provided with apparatus as shown in Figure 2:
1. a square coil of resistance 5.2 0.2 and 130 turns
2. a small cylindrical magnet of mass 15.0 0.2 g with nylon strings.
3. a voltmeter (for measuring the potential difference across the coil only)
4. a power supply (placed under the table to avoid the interference of its magnetic field)
5. a wooden stand
6. a stop watch

7. a ruler

8. a protractor

9. white label (you can write on it)

10. color clay

11. graph papers

12. an electrical cord

Experimental competition

Problem 1

28 April 2009
Page 2 of 4
-------------------------------------------------------------------------------------------------------------------------

3
1
2
12
12

10

Platform

8
1

10

not used

7
5
DO NOT change
these connections.

Figure 2
Warning
Use the multi-meter to measure only the voltage difference of the coils. Using the multi-meter
in other modes can destroy the power supply!!!

Experimental competition

Problem 1

28 April 2009
Page 3 of 4
-------------------------------------------------------------------------------------------------------------------------

Section I
[1 point]
It is given here that the magnetic flux density BP at a

perpendicular distance A from the middle of a straight

current element ia is

i
a
Figure 3

BP =

0i
2 A

( a 2)
a
A2 +
2

.. (i)

where 0 = 4 107 henry per metre, the permeability of free space.

Use this expression to show that the expression for the magnitude of the magnetic flux density
from the square coil at point P in Figure 1 is given by

0 a 2 iN
1

B px =
2
2
2 2 a 2
x + x + 2 a

2
2

.. (ii)

It is also given here that the period of a small-amplitude oscillation of the magnet in the net
magnetic field B is

= 2

I
mB

.. (iii)

where m is the magnetic moment of magnet with mass M , and I is its moment of inertia about the
axis through its centre of mass
I

L2 r 2
= M +
12 4

.. (iv)

Experimental competition

Problem 1

28 April 2009
Page 4 of 4
-------------------------------------------------------------------------------------------------------------------------

Section II

[0.8 point]

For the experiments in Section III you have to align the magnet in the position as shown in Figure 1.
If the length of the string is too small, the torsion of the string cannot be neglected in the oscillation
of the magnet. Perform appropriate measurements (say, oscillation of magnet in Earths magnetic
field alone) to justify that we can ignore the torsion of the string. You are not required to plot a
graph.

Section III
For the following experiments (in a, b, and c), you have to align the magnet in the position as shown
in Figure 1. Measure and write down the value of the distance between the centre of the magnet and
[0.2 point]

the top surface of the platform.

a) Coils magnetic field and Earths horizontal magnetic field in the same direction [5 points]
Warning
Please connect the coil to the power supply and leave it on for at least 5 minutes.
Measure periods of oscillation for different values of the combined field strength when the
coils magnetic field and Earths magnetic field are in the same direction. Draw a straight line
graph and compute the values of BH and the magnetic moment m from this graph and estimate
their errors.

b) Earths magnetic field only

[1 point]

Use the value of m from (a) and the period of oscillation of the magnet bar in the absence of
the Coils magnetic field from Section II to calculate again the value for BH and estimate its
error.

c) Coils magnetic field and Earths horizontal magnetic field in opposite directions

[2 points]

By reversing the connection at the power supply, find the equilibrium position x0 along the Xdirection between Earths magnetic field and the opposing magnetic field from the coil. Use the
value of x0 to calculate again the value for BH and estimate its error.
********************

Experimental competition

Problem 2

28 April 2009
Page 1 of 5
-------------------------------------------------------------------------------------------------------------------------

Problem 2: Oscillation of Water-Filled Vessel


The student is required to perform non-destructive measurements in order to determine the thickness
t of an aluminium vessel whose cavity is completely filled with water. The aluminium vessel is

composed of a cylinder and two end plates. The cylinder is of length L and outer radius R . The total
length of the vessel is h . The thickness of both end plates is 0.60 cm (see Figure 1). You can neglect
the error of this thickness. In this problem, please use gramme and centimetre as units for mass and
length, respectively.

0.60 cm
water

g
X

A
a

CM

0.60 cm

mass M
t cm
Figure 1

Figure 2

Figure 2 shows the so-called bifilar suspension of mass M . The two strings are each of equal
length A . The period T of a small-amplitude oscillation of M is

= 2

A I

g Ma 2

.. (i)

where I is the effective moment of inertia about the vertical axis through the centre of mass of M
and g is the acceleration due to gravity at Bangkok ( g = 978 cm s 2 ) .

This experiment consists of two parts. Section I concerns a derivation of formulae and Section II
concerns the actual experimentation.

Experimental competition

Problem 2

28 April 2009
Page 2 of 5
-------------------------------------------------------------------------------------------------------------------------

Apparatus

Each student is provided with:


1. a water-filled vessel
2. a stand
3. a stop watch
4. a ruler
5. a nylon string
6. a protractor
7. masking tapes
8. a knife (not shown in the figure below)

3
7
6
5
4

Experimental competition

Problem 2

28 April 2009
Page 3 of 5
-------------------------------------------------------------------------------------------------------------------------

Section I
[2.0 points]
The student is to derive expressions in terms of R, L, t and the density of aluminium of the
following quantities, [see Figure 1]
i)

mass ( m1 ) of the cylindrical body of the vessel,

ii)

mass ( m2 ) of each end plate,

iii)

mass ( m3 ) of water in the whole cavity,

iv)

the total mass ( M ) of the water-filled vessel, and

v)

the effective moment of inertia, I y , about the Y-axis, of this water-filled vessel (see
Figure 1), assuming that the water is ideal fluid.

Then perform measurements of R, h, L . By substituting the values, derive expressions in terms of t


for the quantities i)-v) above. The aluminium density = 2.70 g/cm3 and the water density is 1.00
g/cm3.

Hint:

Y
R2

R1 O

I =m

X
m

1
m ( R22 + R12 ) ,
2
1
I x = m ( R22 + R12 )
4
Iy =

L2
12

Thin rod of length

Thin cylinder of inner radius R1 and outer radius R2

Figure 3

Experimental competition

Problem 2

28 April 2009
Page 4 of 5
-------------------------------------------------------------------------------------------------------------------------

Section II

Figure 4

Figure 5

a) Angular oscillation about the axis of symmetry

[4.0 points]
For one fixed value of A , perform precise measurements of the period Ty for a small-amplitude
oscillation as in Figure 4. Then compute the value of the thickness ( t ) of the cylindrical wall.
Estimate the experimental error t for the thickness.
Compute also the values of m1 , m2 , m3 , and M using this value of t .

Experimental competition

Problem 2

28 April 2009
Page 5 of 5
------------------------------------------------------------------------------------------------------------------------b) Angular oscillation about the central axis perpendicular to the length

[2.8 points]
Change the bifilar suspension of the vessel to that of Figure 5 and make similar measurements
as in (a).
Then use the value of the period of oscillation just found together with the values of t , m1 , m3 , M
found in (a) to compute the value of the effective moment of inertia I xExp of the vessel about the
X-axis (see Figure 2 and Figure 5).
Compute also the theoretical estimate of the value of I xTheo based on the value of t found in (a)
assuming that the whole of the computed mass of water found in (a) is now constrained to take
part in the oscillatory motion of the vessel.

c) Comparing experimental and theoretical values of the moment of inertia

What is the difference ( I x ) between the values of I xTheo and I xExp ?

[1.2 points]

Do you consider this difference statistically significant?


Estimate the percentage of the mass of water that takes part in the oscillatory motion in (b),
assuming this water to be circular discs adhering to the end plates.

Hint:

Theo
x

L2 R 2 + ( R t )2
( 0.6 cm )2 R 2 L 0.6 cm 2
L2 ( R t )2
= m1 +
+
+ +
+ 2m2

+ m3 +
4
12
4 2
2
4
12

12

*********************

EXPERIMENTAL PROBLEM 1
DETERMINATION OF THE WAVELENGTH OF A DIODE LASER
MATERIAL
In addition to items 1), 2) and 3), you should use:
4) A lens mounted on a square post (LABEL C).
5) A razor blade in a slide holder to be placed in acrylic support, (LABEL D1) and
mounted on sliding rail (LABEL D2). Use the screwdriver to tighten the support if
necessary. See photograph for mounting instructions.
6) An observation screen with a caliper scale (1/20 mm) (LABEL E).
7) A magnifying glass (LABEL F).
8) 30 cm ruler (LABEL G).
9) Caliper (LABEL H).
10) Measuring tape (LABEL I).
11) Calculator.
12) White index cards, masking tape, stickers, scissors, triangle squares set.
13) Pencils, paper, graph paper.

Razor blade in a slide holder to be placed in acrylic support (LABEL D1) and mounted on
sliding rail (LABEL D2).

EXPERIMENT DESCRIPTION
You are asked to determine a diode laser wavelength. The particular feature of this
measurement is that no exact micrometer scales (such as prefabricated diffraction gratings)
are used. The smallest lengths measured are in the millimetric range. The wavelength is
determined using light diffraction on a sharp edge of a razor blade.

Figure 1.1 Typical interference fringe pattern.

Once the laser beam (A) is reflected on the mirror (B), it must be made to pass through a
lens (C), which has a focal length of a few centimeters. It can now be assumed that the focus
is a light point source from which a spherical wave is emitted. After the lens, and along its
path, the laser beam hits a sharp razor blade edge as an obstacle. This can be considered to
be a light source from which a cylindrical wave is emitted. These two waves interfere with
each other, in the forward direction, creating a diffractive pattern that can be observed on a
screen. See Figure 1.1 with a photograph of a typical pattern.

There are two important cases, see Figures 1.2 and 1.3.

Figure 1.2. Case (I). The razor blade is before the focus of the lens. Figure is not at scale. B
in this diagram is the edge of the blade and F is the focal point.

Figure 1.3. Case (II). The razor blade is after the focus of the lens. Figure is not at scale. B
in this diagram is the edge of the blade and F is the focal point.

EXPERIMENTAL SETUP
Task 1.1 Experimental setup (1.0 points). Design an experimental setup to obtain the
above described interference patterns. The distance L0 from the focus to the screen should
be much larger than the focal length.
Make a sketch of your experimental setup in the drawing of the optical table
provided. Do this by writing the LABELS of the different devices on the drawing of
the optical table. You can make additional simple drawings to help clarify your
design.
You may align the laser beam by using one of the white index cards to follow the
path.
Make a sketch of the laser beam path on the drawing of the optical table and write
down the height h of the beam as measured from the optical table.
WARNING: Ignore the larger circular pattern that may appear. This is an effect due
to the laser diode itself.
Spend some time familiarizing yourself with the setup. You should be able to see of the
order of 10 or more vertical linear fringes on the screen. The readings are made using the
positions of the dark fringes. You may use the magnifying glass to see more clearly the
position of the fringes. The best way to observe the fringes is to look at the back side of
the illuminated screen (E). Thus, the scale of the screen should face out of the optical table.
If the alignment of the optical devices is correct, you should see both patterns (of Cases I
and II) by simply sliding the blade (D1) through the rail (D2).

THEORETICAL CONSIDERATIONS
Refer to Figure 1.2 and 1.3 above. There are five basic lengths:

L0 : distance from the focus to the screen.


Lb : distance from the razor blade to the screen, Case I.
La : distance from the razor blade to the screen, Case II.
LR (n) : position of the n-th dark fringe for Case I.
LL (n) : position of the n-th dark fringe for Case II.
The first dark fringe, for both Cases I and II, is the widest one and corresponds to n = 0.
Your experimental setup must be such that LR (n) << L0 ,Lb for Case I and LL (n) << L0 ,La
for Case II.
The phenomenon of wave interference is due to the difference in optical paths of a wave
starting at the same point. Depending on their phase difference, the waves may cancel each

other (destructive interference) giving rise to dark fringes; or the waves may add
(constructive interference) yielding bright fringes.
A detailed analysis of the interference of these waves gives rise to the following condition to
obtain a dark fringe, for Case I:

5
I (n) = n +

with

n = 0, 1, 2,

(1.1)

and for Case II:

II (n) = n +

with

n = 0, 1, 2,

(1.2)

where is the wavelength of the laser beam, and I and II are the optical path differences
for each case.
The difference in optical paths for Case I is,

I (n) = (BF + FP) BP

for each

n = 0, 1, 2,

(1.3)

for each

n = 0, 1, 2,

(1.4)

while for Case II is,

II (n) = (FB + BP) FP

Task 1.2 Expressions for optical paths differences (0.5 points). Assuming LR (n) << L0 ,Lb
for Case I and LL (n) << L0 ,La for Case II in equations (1.3) and (1.4) (make sure your setup
satisfies these conditions), find approximated expressions for I (n) and II (n) in terms of
r
L0, Lb , La , LR (n) and LL (n) . You may find useful the approximation (1+ x ) 1+ rx if
x << 1 .
The experimental difficulty with the above equations is that L0 , LR (n) and LL (n) cannot be
accurately measured. The first one because it is not easy to find the position of the focus of
the lens, and the two last ones because the origin from which they are defined may be very
hard to find due to misalignments of your optical devices.
To solve the difficulties with LR (n) and LL (n) , first choose the zero (0) of the scale of the
screen (LABEL E) as the origin for all your measurements of the fringes. Let l0 R and l0 L be
the (unknown) positions from which LR (n) and LL (n) are defined. Let lR (n) and lL (n) be
the positions of the fringes as measured from the origin (0) you chose. Therefore

LR (n) = lR (n) l0 R

and

LL (n) = lL (n) l0L

(1.5)

PERFORMING THE EXPERIMENT. DATA ANALYSIS.


Task 1.3 Measuring the dark fringe positions and locations of the blade (3.25 points).
For both Case I and Case II, measure the positions of the dark fringes lR (n) and
lL (n) as a function of the number fringe n. Write down your measurements in Table
I; you should report no less than 8 measurements for each case.
Report also the positions of the blade Lb and La , and indicate with its LABEL the
intrument you used.
IMPORTANT SUGGESTION: For purposes of both simplification of analysis and
better accuracy, measure directly the distance d = Lb La with a better accuracy than
that of Lb and La ; that is, do not calculate it from the measurements of Lb and La .
Indicate with its LABEL the instrument you used.

Make sure that you include the uncertainty of your measurements.


Task 1.4 Data analysis. (3.25 points). With all the previous information you should be able
to find out the values of l0 R and l0 L , and, of course, of the wavelength .
Devise a procedure to obtain those values. Write down the expressions and/or
equations needed.
Include the analysis of the errors. You may use Table I or you can use another one to
report your findings; make sure that you label clearly the contents of the columns of
your tables.
Plot the variables analyzed. Use the graph paper provided.
Write down the values for l0 R and l0 L , with uncertainties.

Task 1.5 Calculating . Write down the calculated value for . Include its uncertainty and
the analysis to obtain it. SUGGESTION: In your formula for , wherever you find
(Lb La ) replace it by d and use its measured value. (2 points).

Question Number

Experimental Competition
27 April 2010

Page 1 of 12

__________________________________________________________________________________________

Experimental components
Set-C for common components
Label

Items

Quantity

C-A

Optical track (60 cm)

C-B

Optical clamps

C-C-#

Collimated laser diode (CLD)

C-D-#

Sine wave generator (sine wave, DC 5V output)

C-E-#

Variable resistor (5 k)

C-F-#

Connecting wires

C-F-6

Component stand

C-G

Ruler

Note: # is the serial number for the component. This number is for examiners use.

Question Number

Experimental Competition
27 April 2010

Page 2 of 12

__________________________________________________________________________________________

Set-I for Experiment-I


Label

Items

I-H-#

Black box on a 1-D translational stage

I-I-#

Brass reed attached to a driving box*

I-J

Screen for amplitude measurement

I-K-#

Vertical slider (with a ruler and a magnet)

Quantity

The brass reed with a fixed end inside a box is attached to a piezo driven by an AC voltage.

Question Number

Experimental Competition
27 April 2010

Page 3 of 12

__________________________________________________________________________________________

Instructions for the Sine Wave Generator:


The power button, not shown in the picture, is on the right-hand side of the
instrument.
The Display Panel shows the frequency of the output sine wave.
Use the Sine Wave BNC connector for supplying a sine wave voltage.
Use the 5V DC banana connectors for supplying a constant voltage of 5V.
The frequency of the sine wave can be changed by turning the FREQUENCY knob,
faster for coarse adjustment and slower for fine adjustment. Ignore the fine and
coarse labels next to the FREQUENCY knob.
The amplitude of the sine wave voltage can be adjusted by turning the AMPL ADJ
knob.
The RESET bottom may be pushed to reset frequency to 0.00 Hz.

Question Number

Experimental Competition
27 April 2010

Page 4 of 12

__________________________________________________________________________________________

Set-II for Experiment -II


Label

Items

II-L-#

Uncollimated laser diode (ULD)

II-M

Holders for ULD

II-P-#

Polarizer with indicator (PR2)

II -Q-#

Polarizer (PR1)

II-R

Holder for PR1 and PR2

II-T

Holder for light filter

II-U

Light filters

II-V

Beam viewing box (screen)

II-W-#

Photoconductor (PC)

II-X

Digital multimeter

II-Y

Digital multimeter

Items II-L-#, II-M, and II-V are not used.

Quantity

Question Number

Experimental Competition
27 April 2010

Page 5 of 12

__________________________________________________________________________________________

Instructions for the digital multimeter:


You can turn the digital multimeter on or off by pressing the power button.
Use the V and the COM inlets for voltage and resistance measurements.
Use the mA and the COM inlets for small current measurements.
Use the function dial to select the proper function and measuring range. V is for
voltage measurement, A is for current measurement and is for resistance
measurement.
Do not press the HOLD button, which will hold the display reading and stop the
measurement function. You can release it by pressing the button again.

Question Number

Experimental Competition
27 April 2010

Page 6 of 12

__________________________________________________________________________________________

Experiment I. Magnetic force probe


Introduction
As shown in Fig. I-1, the free end of a reed can oscillate in the vertical direction when it
is driven by an external oscillating force, and its frequency is determined by the external
driver. If we plot the average power dissipated in the vibrating reed, which has certain
damping mechanisms, as a function of frequency, we can find a maximum dissipated power at
a certain frequency called the resonance frequency fR, as illustrated in Fig. I-2. The
sharpness of the resonance is described by the quality factor Q as:
f
Q= R
f
where f is the full width at half maximum of the Pav-f curve, as shown in Fig. I-2, i.e. f
= f2 - f1 with f1 and f2 corresponding to Pmax 2 on the lower side and the higher side of the
resonance frequency respectively.

Fig. I-1. A vibrating reed.

Fig. I-2. Plot of the average dissipated power versus the driving frequency.
Besides the oscillating driving force, if the free end of the reed is subjected to a uniform
force, its resonance frequency, amplitude and quality factor remain the same. On the other
hand, under a non-uniform force, many properties of the vibrating reed, such as the resonance
frequency fR, the maximum amplitude A, and the quality factor Q, may vary with the position
of its free end.

Question Number

Experimental Competition
27 April 2010

Page 7 of 12

__________________________________________________________________________________________

In this experiment, a small magnet adhered to the free end of the reed serves as a probe
tip as shown in Fig. I-3, while a target magnet underneath the tip magnet produces a
non-uniform magnetic field and exerts a non-uniform force on the tip magnet. When the tip
magnet approaches the target magnet underneath with same pole opposing each other, then
the repulsive force becomes stronger. Thus the resonance frequency fR of the reed varies with
the distance between the tip magnet and the target magnet. The resonance frequency increases
with decreasing separation distance between the two repulsing magnets. However, when
moving the tip magnet horizontally away from the target magnet as shown in Fig. I-4, it may
sense a weak attractive force at a certain distance. The resonance frequency shifts to lower
values when the non-uniform force is attractive. We shall use this property, that the resonance
frequency of the reed sensitively depends on the separation between the tip magnet and target
magnet, to locate the hidden magnets inside a black box.

Fig. I-3. Near a pole of a target magnet, the magnetic field is non-uniform.

Fig. I-4. Moving a tip magnet horizontally may cause it to sense an attractive or repulsive
force.

Question Number

Experimental Competition
27 April 2010

Page 8 of 12

__________________________________________________________________________________________

Experimental procedures
Error analysis is not required in any parts of Experiment I.
Exp. I-AMeasuring the resonance frequency
Carefully take out the experimental components from Set-C and Set-I, and set up
the experimental apparatus as shown in Fig. I-A-1. The schematic plot is shown in Fig.
I-A-2. Connect the 5V-DC voltage source to the laser box (C-C-#). Connect the
oscillating output of the sine wave generator to the driving box of the brass reed (I-I-#).
Turn on the power and fix the output voltage of the sine wave generator. Direct the laser
beam into the mirror at the free end of the brass reed so that the reflected beam spot on the
screen (I-J) can be used to determine the vibrating amplitude of the reed.
Caution: 1) Carefully remove the paper protected cover before using the brass reed (I-I-#) for
experimental measurements. The resonance frequency of the brass reed is very
sensitive to its shape, and any deformation of reed during the experiment may give
an inaccurate result.
2) Do not directly look into the laser beam, which can damage your eyes.

Fig. I-A-1. Experimental setup for finding the resonance frequency.

Question Number

Experimental Competition
27 April 2010

Page 9 of 12

__________________________________________________________________________________________

Fig. I-A-2. The schematic plot of Fig. I-A-1.


(1) Measure the amplitude A of the oscillating laser beam spot by changing the
frequency of the sine wave generator. Record the measured amplitude as a
function of frequency in the data table on the answer sheet.
(0.8 points)
(2) Make a proper plot on one of the supplied graph papers to determine the resonance
frequency fRO and quality factor Q. Also record the obtained fRO and Q in the
proper blank spaces on the answer sheet.

(1.2 points)

Exp. I-BResonance frequency versus the external force.


In this part of experiment, the resonance frequency under the influence of a
non-uniform force is investigated. The non-uniform force is provided by a small 3-mm
cylindrical metallic calibration magnet MC fixed on a vertical slider (I-K-#) with its N
pole pointing upward. The tip magnet MT, adhered at the free end of the oscillating reed,
has its N pole pointing downward. The pole axes of both magnets should be aligned along
the same vertical line.
Set up the experiment as shown in Fig. I-B-1. The schematic plot is shown in Fig.
I-B-2.

Question Number

Experimental Competition
27 April 2010

Page 10 of 12

__________________________________________________________________________________________

Fig. I-B-1. Experimental setup of finding the relation of resonance frequency with the
nominal distance between two magnets, MC and MT.

Fig. I-B-2. The schematic plot of Fig. I-B-1.


(1) On the scale of the vertical slider, read out the position z0 of the bottom plane for the
tip magnet MT without the interaction of MC by properly moving MC away from MT.
Record the measured z0 in the data table.
(0.2 points)
(2) Adjust the position of the magnet MC to be right underneath MT. The pole axes of
both magnets should be aligned along the same vertical line. Determine the position
z of the top plane of the N-pole of MC. Calculate the nominal distance d by
defining d = z0 z . Record z and d in the data table. (Note: The equilibrium
separation between the two magnets is not the same as d because the two magnets
repel each other.)
(3) Determine the resonance frequency fR for the distance d by tuning the frequency of

Question Number

Experimental Competition
27 April 2010

Page 11 of 12

__________________________________________________________________________________________

the sine wave generator until the maximum amplitude is reached, plotting amplitude
versus frequency is not necessary for determining fR of each distance d. Record the
determined resonance frequency fR in the data table.
(4) Change the vertical position of the magnet MC and repeat the steps (2) and (3) for a
number of measurements of different distance d and the corresponding resonance
frequency fR.
(1.2 points)
(5) Plot a graph of fR as a function of distance d using a graph paper. Guiding by eyes,
draw the best line through the data points.
(1.2 points)
(6) Define fR = fR - fRO, and plot ln(fR) as a function of d using another graph paper.
Guiding by eyes, draw the best line through the data points.

(1.0 points)

Exp. I-CFinding the positions and depths of the magnets inside a black box.
There are two magnets MA and MB buried in the black box (I-H-#) which is fixed on
a 1D translational stage. The N poles of both magnets are pointed upward. Magnets MA and
MB, and MC used in Exp. I-B are very close in size, shape, and magnetic properties. The
depths of the magnets MA and MB may be different. Magnet MA is located at the
intersection of the two lines marked on the top surface of the black box. Magnet MB is
located somewhere along the longer line as shown in Fig. I-C-1. The horizontal distance
between magnets MA and MB is denoted by AB .

Fig. I-C-1. Magnet MA is located beneath the intersection of the two lines marked on the
top surface while the magnet MB is located somewhere along the longer line.
(1) On the scale of the vertical slider, read out the position z0 (in this part, z0 may be
different from the z0 in Exp. I-B) of the bottom plane for the tip magnet MT without
the interaction of the magnets inside the black box. On the scale of the vertical slider,
read out the position zbox of the top plane of black box. Record z0 and zbox on the
answer sheet.

(0.2 points)

Question Number

Experimental Competition
27 April 2010

Page 12 of 12

__________________________________________________________________________________________

(2) Move the black box along the longer line and observe the variation in resonance
frequency fR of the reed to find the position of MB. Record the measured distances y
and their corresponding resonance frequencies fR in the data table.
(1.4 points)
(3) Plot fR as a function of y on a graph paper to determine the position of magnet MB.
Mark the positions of magnets MA and MB on the y-axis of your graph, and write
(1.2 points)
down the value of AB on the answer sheet.
(4) Determine the depths dA and dB of the magnets MA and MB from the top surface of the
black box using the results in Exp. I-B. Write down the values of dA and dB on the
answer sheet.

(1.6 points)

41st International Physics Olympiad, Croatia Experimental Competition, July 21st, 2010

1/4

Experimental problem 1
There are two experimental problems. The setup on your table is used for both problems. You have 5
hours to complete the entire task (1&2).

Experimental problem 1: Elasticity of sheets


Introduction
Springs are objects made from elastic materials which can be used to store mechanical energy. The
most famous helical springs are well described in terms of Hookes law, which states that the force
with which the spring pushes back is linearly proportional to the distance from its equilibrium length:

F kx , where k is the spring constant, x is the displacement from equilibrium, and F is the
force [see Fig. 1(a)]. However, elastic springs can have quite different shapes from the usual helical
springs, and for larger deformations Hookes law does not generally apply. In this problem we
measure the properties of a spring made from a sheet of elastic material, which is schematically
illustrated in Fig. 1(b).

Figure 1. Illustrations of (a) a helical spring and (b) a spring made from a sheet of elastic material
rolled up into a cylinder. When the latter spring is sufficiently compressed, its shape can be
approximated as a stadium with two semicircles of radii R0 (see text).

Transparent foil rolled into a cylindrical spring


Suppose that we take a sheet of elastic material (e.g. a transparent foil) and bend it. The more we
bend it, the more elastic energy is stored in the sheet. The elastic energy depends on the curvature
of the sheet. Parts of the sheet with larger curvature store more energy (flat parts of the sheet do
not store energy because their curvature is zero). The springs used in this experiment are made from
rectangular transparent foils rolled into cylinders (see Fig. 2). The elastic energy stored in a cylinder is

Eel

1
2
c

2R

A,

(1)

41st International Physics Olympiad, Croatia Experimental Competition, July 21st, 2010

2/4

where A denotes the area of the cylinder's side (excluding its bases), Rc denotes its radius, and the
parameter , referred to as the bending rigidity, is determined by the elastic properties of the
material and the thickness of the sheet. Here we neglect the stretching of the sheets.

Figure 2. A schematic picture of an elastic sheet rolled into a cylinder of radius Rc and length l .

Suppose that such a cylinder is compressed as in Fig. 1(b). For a given force applied by the press ( F ),
the displacement from equilibrium depends on the elasticity of the transparent foil. For some
interval of compression forces, the shape of the compressed transparency foil can be approximated
with the shape of a stadium, which has a cross section with two straight lines and two semicircles,
both of radius R0 . It can be shown that the energy of the compressed system is minimal when

R02

l
.
2F

(2)

The force is measured by the scale calibrated to measure mass m , so F mg ,

m/s2.

Experimental setup (1st problem)


The following items (to be used for the 1st problem) are on your desk:
1. Press (together with a stone block); see separate instructions if needed
2. Scale (measures mass up to 5000 g, it has TARA function, see separate instructions if needed)
3. Transparency foils (all foils are 21 cm x 29.7 cm, the blue foil is 200 m thick, and the
colorless foil is 150 m thick); please, do ask for the extra foils if you need them.
4. Adhesive (scotch) tape
5. Scissors
6. Ruler with a scale
7. A rectangular wooden plate (the plate is to be placed on a scale, and the foil sits on the plate)
The setup is to be used as in Fig. 3. The upper plate of the press can be moved downward and
upward using a wing nut, and the force (mass) applied by the press is measured with the scale.
Important: The wing nut moves 2 mm when rotated 360 degrees. (Small aluminum rod is not used
in Experiment 1.)

41st International Physics Olympiad, Croatia Experimental Competition, July 21st, 2010

3/4

Figure 3. The photo of a setup for measuring the bending rigidity.

Tasks
1. Roll the blue foils into cylinders, one along the longer side, and the other along its shorter side;
use the adhesive tape to fix them. The overlap of the sheet should be about 0.5 cm.
(a) Measure the dependence of the mass read by the scale on the separation between the
plates of the press for each of the two cylinders. (1.9 points)
(b) Plot your measurements on appropriate graphs. Using the ruler and eye as the guide,
draw lines through the points and determine the bending rigidities for the cylinders.
Mark the region where the approximate relation (the stadium approximation) holds.
Estimate the value of

R0
below which the stadium approximation holds; here Rc is the
Rc

radius of the non-laden cylinder(s). (4.3 points)


The error analysis of the results is not required.
2. Measure the bending rigidity of a single colorless transparent foil. (2.8 points)
3. The bending rigidity depends on the Youngs modulus Y of elasticity of the isotropic
material, and the thickness d of the transparent foil according to

41st International Physics Olympiad, Croatia Experimental Competition, July 21st, 2010

Yd 3
,
12(1 2 )

4/4

(3)

where is the Poisson ratio for the material; for most materials 1 / 3 . From the previous
measurements, determine the Youngs modulus of the blue and the colorless transparent
foil. (1.0 points)

Experimental Question 1: Levitation of Conductors in an Oscillating Magnetic Field


In an oscillating magnetic field of sufficient strength, levitation of a metal conductor becomes possible. The levitation
occurs due to a non-zero mean magnetic force exerted on currents in the conductor. The currents are induced by the
alternating field itself. The Lorentz force doesnt average to zero because of a phase shift between the current
oscillations and the magnetic field oscillations. This phase shift is a result of the self-inductance of the current loops
within the conductor.
In this experiment, we study this phenomenon and deduce the self-inductance of an aluminum ring from measuring
the vertical force applied to it by a solenoid with an oscillating current. For measurement convenience, the mean force
on the ring will be directed downwards, so levitation will not be observed.
On your desk, you have the following items (Figure 1):
(1)

An AC power supply operating on 50Hz. The power supply has two pairs of terminals. The two smaller
sockets (1a) supply a voltage of about 24V; use these only for running current through the solenoid. The two
larger sockets (1b) supply a voltage of about 0.7V. The power supply is turned on only when the green button
(1c) is pressed this is in order to prevent overheating the system by accidentally leaving the current on, as
further explained below. The red light bulb indicates when the power supply is on.

(2)

A cylindrically symmetric solenoid filled with iron rods. The solenoid is connected to the 24V terminals of the
power supply. The solenoid can be raised and lowered using a long screw. The screws vertical step is
.

(3-5)

Three metal rings made of exactly the same material (an alloy of aluminum). One ring is closed. The second
ring is identical to the first except for a short segment which was removed, making the ring open. The third
ring is also open, and is much thinner than the first two.

(6-7)

Two multimeters. They will be used as a voltmeter and an ammeter. The AC voltmeters sensitivity is 0.1mV.
The ammeter can measure currents of up to 20A (also in AC mode). Note: in AC mode, the multimeters
display the RMS (root-mean-square) of the measured quantity, i.e. the amplitude divided by . See Figures 3
and 4 for detailed instructions.

(8)

A battery-powered digital scale with sensitivity 0.01g. When the scale experiences a rapidly oscillating force,
it displays the time-averaged force. Note: the scale has a tare option, which calibrates its reading under a
given weight to zero. See Figure 2.

(9)

An 8cm7cm7cm polystyrene block which can be used as a stand for the rings.

(10)

Electric wires with various connectors.

(11)

A ruler.

(12)

Millimeter graph paper.

(13)

A desktop lamp which can be turned on or off for your convenience.

Caution: when the closed ring is exposed to the solenoids magnetic field, a large current flows through it, heating it
up. As a consequence, the rings electrical properties may change slightly. To avoid this, dont run a current through
the solenoid for long periods of time.
The earths gravity field in Tel Aviv is

.
Page 1 of 5

Figure 1 - Summary of the equipment.


(2)

(1)

(1)
(1b)

(1a)
(8)

(9)

(1c)

(3)

(6)

(7)

(4)

(5)

(10)

Figure 2: The digital scale


(1) On/Off button
(2) Tare button sets the
current weight as 0.

(2)

(1)

Page 2 of 5

(2)
(3)
(4)
(5)
(6)
(7)
(8)
(9)
(10)

50Hz power supply.


(1a) 24V terminals.
(1b) 0.7V terminals.
(1c) Power button.
Solenoid on a vertical screw.
Broad closed ring.
Broad open ring.
Thin open ring.
Voltmeter.
Ammeter.
Scale.
Polystyrene block.
Wires.

Figure 3: The voltmeter.


(1) On/Off button.
(2) The dial is set to 200mV AC.
(3) Connect your wires to the COM
and V/ terminals.

(1)

(2)
)

(3)
)
Figure 4: The ammeter.

(1) On/Off button.


(2) The dial is set to 20mA/20A AC.
(3) Connect your wires to the COM
and A terminals.
(1)

(2)

(3)
)

Page 3 of 5

Theory (1.3 points)


Consider a conducting ring of radius placed in the solenoids magnetic field. The symmetry axes of the solenoid and
the ring coincide. Denote the rings inductance by , its resistance by , and the angular frequency of the solenoids
current by . Define as the coordinate along the common symmetry axis of the solenoid and the ring.
In this part only, you may neglect the small effect of the rings magnetic field on the solenoid and the iron. Also,
neglect the thickness of the ring.
In this part, well use Faradays law, and the magnetic version of Gausss law:

Faradays law: The induced EMF (electromotive force) on a loop generated by a changing magnetic flux is
.
The magnetic Gauss law: the total magnetic flux through a closed surface is zero.

A current loop placed in a cylindrically symmetric magnetic field experiences a total force
( )

( )

( )

where is the current in the loop, and


is the radial component (in the direction of the loops radius) of the external
magnetic field in the loops vicinity. The positive direction of the force is downwards in the direction. The
positive direction of the current is shown in Figure 5.
( )
a. (0.2 pts.) Consider an oscillating external magnetic flux
( ) through the ring.
Find ( ) - the EMF induced by the given flux only, and ( ) - the current induced in the ring, as functions of
, , , and .
Hint: The EMF amplitude and the current amplitude on an AC circuit element with both resistance and

inductance are related by

, and the current is delayed by a phase

with respect

to the EMF.
b. (0.6 pts.) Find

, where is the coordinate along the axis perpendicular to the rings

in terms of and

plane.
c. (0.5 pts.) Show that

(
(

, where is the time averaged value of

( ) is the RMS

(root-mean-square, i.e., amplitude divided by ) of the EMF on a loop at height . Find the constant
not find , in later parts, take the magnitude of to be 1).
Hint: you may find the following identities useful:
(
)
(
)
(

))

))

Figure 5: The metal ring in the solenoids magnetic field.


Page 4 of 5

(if you do

Measurements (5.1 points)


In all of the following measurements and analysis, take into account that results with higher precision will receive
higher grades. In all of your measurements and results, specify error estimations.
Resistance measurements (2.6 points)
In this part, you should use the 0.7V terminals of the AC power supply. Using the supplied wires to short-circuit the
two 0.7V terminals should result in a current of
, depending on the contacts. Note that the three shorter
wires achieve a better contact with the ammeter than the two longer ones. CAUTION: Dont use the 24V terminals, to
avoid overheating the components.
d. (1.3 pts.) Find the resistance
of the thin ring. Draw your circuit on the answer form.
Hint: the resistance of each of the rings is much smaller than
. For the thin ring, you can neglect the
inductive impedance with respect to the resistance.
e. (1.3 pts.) Find the resistance

of the closed ring. Make additional measurements as necessary.

Measurements of the induced EMF (1.5 points)


f. (1.5 pts.) Connect the solenoid to the 24V terminals of the power supply. Place the broad open ring so that its axis
coincides with the axis of the solenoid. Measure the induced EMF
on the ring at different heights , i.e., at
different distances from the solenoid. Record your measurements in the provided table on the answer form. Plot a
graph of
as a function of (with a trend line).
Measurements of the force (1 point)
g. (1 pt.) Place the broad closed ring so that its axis coincides with the axis of the solenoid. Measure the timeaveraged magnetic force on the ring at different heights , i.e. at different distances from the solenoid. Record
your measurements in the provided table on the answer form.

Analysis (3.6 points)


| of
h. (1.4 pts.) Find the absolute value of the derivative |
with respect to , for values of where you
measured the force in part (g). Record your values in the provided table on the answer form. Error estimations are
not required in this part.
i.

(2.2 pts.) Analyze your results using a linear graph to find - the inductance of the closed ring. You may use the
fact that
.
Note: Despite the noticeable thickness of the closed ring, the formula you derived in part (c) still applies with a
high accuracy. Use it as an operational definition for the inductance of a broad ring.
Hint: When the ring is too close to the iron, the measurements will be distorted. Try to avoid this complication in
your analysis.

Page 5 of 5

Experimental Competition:

14 July 2011

Problem 1

Page 1 of 4

1. Electrical Blackbox: Capacitive Displacement Sensor


For a capacitor of capacitance C which is a component of a relaxation oscillator whose frequency
of oscillation is f , the relationship between f and C is as follows:

CS

where is a constant and C S is the stray capacitance of our circuits. The frequency f can be
monitored using a digital frequency meter.
The electrical blackbox given in this experiment is a parallel plate capacitor. Each plate consists of
a number of small teeth of the same geometrical shape. The value of C can be varied by displacing
the upper plate relative to the lower plate, horizontally. Between the two plates there is a sheet of
dielectric material.
Equipment: a relaxation oscillator, a digital multimeter for measuring frequency of the relaxation
oscillator, a set of capacitors of known capacitances, an electrical blackbox and a
battery.
Caution: Check the voltage of the battery and ask for a new one if the voltage is less than 9 V.
Do not forget to switch on.
Electrical connectors to
the plates
Battery

Relaxation oscillator
Electrical blackbox:
Parallel plate capacitor

Sliding upper plate

Switch
Frequency output

Connectors to capacitor
FIGURE 1

Experimental Competition:

14 July 2011

Problem 1

Page 2 of 4

FIGURE 2 Capacitors

The position for frequency


measurements

FIGURE 3 Digital multimeter for measuring frequency

TABLE 1 Nominal Capacitance values


Code
33J
68
82J
151

Capacitance value
(pF)
34 1
68 1
84 1
150 1

Experimental Competition:

14 July 2011

Problem 1

Page 3 of 4

Part 1. Calibration
Perform the measurement of f using the given capacitors of known capacitances. Draw appropriate
graph to find the value of

and C S . Error analysis is not required.

[3.0 points]

Part 2. Determination of geometrical shape of a parallel plate capacitor


[6.0 points]
Given the three possible geometrical shapes as Pattern I, Pattern II and Pattern III as follows:

lower plate

Top view

upper plate

slide in and out

Pattern I

lower plate

upper plate

Pattern II

Top view

slide in and out

Experimental Competition:

14 July 2011

Problem 1

Page 4 of 4

lower plate

upper plate

Top view

slide in and out

Pattern III

For each pattern, draw qualitatively an expected graph of C versus the positions of the upper plate
but label the x-axis. Then, perform the measurement of f versus the positions of the upper plate.
Plot graphs and, from these graphs, deduce the pattern of the parallel plate capacitor and its
dimensions (values of b and w ). The separation d between the upper and lower plates is 0.20 mm.
The dielectric sheet between the plates has a dielectric constant K
space

8.85 10

12

1.5 . The permittivity of free

Fm-1 . Error analysis is not required.

Part 3. Resolution of digital calipers

[1.0 point]

As the relative position of the parallel plates is varied, the capacitance changes with a pattern. This
set-up may be used as digital calipers for measuring length. If the parallel plate capacitor in this
experiment is to be used as digital calipers, estimate from the experimental data in Part 2 its
5 kHz . An error
resolution: the smallest distance that can be measured for the frequency value f
estimate for the final answer is not required.

The 43rd International Physics Olympiad Experimental Competition


Tartu, Estonia Thursday, July 19th 2012
The examination lasts for 5 hours. There are 2 problems
worth in total 20 points. There are two tables in your
disposal (in two neighbouring cubicles), the apparatus of
Problem E1 is on one table and the apparatus of Problem E2 is on the other table; you can move freely between
these tables. However, you are not allowed to move
any piece of experimental setup from one table to
the other.
Initially the experimental equipment on one table is
covered and on the other table is boxed. You must
neither remove the cover nor open the box nor
open the envelope with the problems before the
sound signal of the beginning of competition
(three short signals).
You are not allowed to leave your working place
without permission. If you need any assistance (malfunctioning equipment, broken calculator, need to
visit a restroom, etc), please raise the corresponding ag
(help or toilet with a long handle at your seat)
above your seat box walls and keep it raised until an organizer arrives.
Use only the front side of the sheets of paper.
For each problem, there are dedicated Solution Sheets
(see header for the number and pictogramme). Write
your solutions onto the appropriate Solution Sheets. For
each Problem, the Solution Sheets are numbered; use the
sheets according to the enumeration. Copy the nal answers into the appropriate boxes of the Answer Sheets.

There are also Draft papers; use these for writing things
which you dont want to be graded. If you have written
something what you dont want to be graded onto the
Solution Sheets (such as initial and incorrect solutions),
cross these out.
If you need more paper for a certain problem, please raise
the ag help and tell an organizer the problem number; you are given two Solution sheets (you can do this
more than once).
You should use as little text as possible: try to
explain your solution mainly with equations, numbers,
tables, symbols and diagrams.
Avoid unnecessary movements during the experimental
examination and do not shake the walls of your cubicle the laser experiment requires stability.
Do not look into the laser beam or its reections! It may
permanently damage your eyes!
The rst single sound signal tells you that there are 30
min of solving time left; the second double sound signal
means that 5 min is left; the third triple sound signal
marks the end of solving time. After the third sound
signal you must stop writing immediately. Put all
the papers into the envelope at your desk. You are not
allowed to take any sheet of paper out of the room.
If you have nished solving before the nal sound signal,
please raise your ag.

page 1 of 5

Problem E1. The magnetic permeability of water The remaining legend for the gure is as follows: 6 the point
(10 points)
where the laser beam hits the screen; 11 the LCD screen of
The eect of a magnetic eld on most of substances other
than ferromagnetics is rather weak. This is because the energy
density of the magnetic eld in substances of relative magnetic
B2
permeability is given by the formula w = 2
, and typically
0
is very close to 1. Still, with suitable experimental techniques
such eects are rmly observable. In this problem we study the
eect of a magnetic eld, created by a permanent neodymium
magnet, on water and use the results to calculate the magnetic
permeability of water. You are not asked to estimate any
uncertainties throughout this problem and you do not
need to take into account the eects of surface tension.
The setup comprises of 1 a stand (the highlighted numbers
correspond to the numbers in the g.), 3 a digital caliper,
4 a laser pointer, 5 a water tray and 7 a cylindrical
permanent magnet in the water tray (the magnet is axially
magnetised). The water tray is xed to the base of the stand
by the magnets pull. The laser is xed to the caliper, the base
of which is fastened to the stand; the caliper allows horizontal
displacement of the laser. The on-o button of the laser can be
kept down with the help of 13 the white conical tube. Do not
leave the Laser switched on unnecessarily. The depth of the
water above the magnet should be reasonably close to 1 mm (if
shallower, the water surface becomes so curved that it will be
dicult to take readings from the screen). 15 A cup of water
and 16 a syringe can be used for the water level adjustment
(to raise the level by 1 mm, add 13 ml of water). 2 A sheet of
graph paper (the screen) is to be xed to the vertical plate
with 14 small magnetic tablets. If the laser spot on the screen
becomes smeared, check for a dust on the water surface (and
blow away).

the caliper, 10 the button which switches the caliper units


between millimeters and inches; 8 on-o switch; 9 button
for setting the origin of the caliper reading. Beneath the laser
pointer, there is one more button on the caliper, which temporarily re-sets the origin (if you pushed it inadvertently, push
it once again to return to the normal measuring mode).
Numerical values for your calculations:
Horizontal distance between the magnets centre and the
screen L0 = 490 mm. Check (and adjust, if needed) the alignment of the centre of the magnet in two perpendicular directions. The vertical axis of the magnet must intersect with the
laser beam, and it must also intersect with 12 the black line
on the support plate.
Magnetic induction (magnetic eld strength) on the
magnets axis, at a height of 1 mm from the at surface,
B0 = 0.50 T
Density of water w = 1000 kg/m3
Acceleration of free-fall g = 9.8 m/s2
Permeability of a vacuum 0 = 4 107 H/m
WARNINGS:
The laser orientation is pre-adjusted, do not move it!
Do not look into the laser beam or its reections!
Do not try to remove the strong neodymium magnet!
Do not put magnetic materials close to the magnet!
Turn o the laser when not used, batteries drain in 1 h!

page 2 of 5

Part A. Qualitative shape of the water surface (1 points)


iv. (1.4 pts) please note that the slope (tan ) of the water
When a cylindrical magnet is placed below water surface, the surface can be expressed as follows:
latter becomes curved. By observation, determine the shape
of the water surface above the magnet. Based on this observacos2 0 y y0 (x x0 ) tan 0
tion, decide if the water is diamagnetic ( < 1) or paramagnetic
tan

,
2
L0 + x x0
( > 1).
where y0 is the vertical position of the laser spot on the screen
when the beam is reected from the water surface at the axis
of the magnet, and x0 is the respective position of the caliper.
Calculate the values of the slope of the water surface and enter
them into the Table on the Answer Sheet. Please note that it
may be possible to simplify your calculations if you substitute
Write the letter corresponding to the correct option into the some combination of terms in the given expression for the slope
Answer Sheet, together with an inequality > 1 or < 1.
with a reading from the last graph.
For this part, you do not need to justify your answer.

v. (1.6 pts) Calculate the height of the water surface relative


to the surface far from the magnet, as a function of x, and
write it into the Table on the Answer Sheet.

Part B. Exact shape of the water surface (7 points)


Curving of the water surface can be checked with high sensitivity by measuring the reection of the laser beam from the vi. (1.0 pts) Draw the graph of the latter dependence. Insurface. We use this eect to calculate the dependence of the dicate on it the region where the beam hits the water surface
depth of the water on the horizontal position above the magnet. directly above the magnet.
i. (1.6 pts) Measure the dependence of the vertical position
y of the laser spot on the screen on the caliper reading x (see
gure). You should cover the whole usable range of caliper
displacements. Write the results into the Table in the Answer
Sheet.

Part C. Magnetic permeability (2 points)


Using the results of Part B, calculate the value of 1 (the
so-called magnetic susceptibility), where is the relative magnetic permeability of the water. Write your nal formula and
the numerical result into the Answer Sheet.

ii. (0.7 pts) Draw the graph of the measured dependence.


iii. (0.7 pts) Using the obtained graph, determine the angle
0 between the beam and the horizontal surface of the water.

page 3 of 5

Problem E2. Nonlinear Black Box (10 points)


In simple problems, electrical circuits are assumed to consist of linear elements, for which electrical characteristics are
directly proportional to each other. Examples include resistance (V = RI), capacitance (Q = CV ) and inductance
(V = LI = L dI
dt ), where R, C and L are constants. In this
problem, however, we examine a circuit containing nonlinear
elements, enclosed in a black box, for which the assumption of
proportionality no longer holds.
The setup
comprises a multimeter (labelled IPhOmeasure), a black box that acts as a current source, a black
box containing nonlinear elements, and four test leads with
stackable connectors for wiring. Be careful not to break the
seal on the black box.
The multimeter can measure current and voltage simultaneously. You can store with it up to 2000 data points, each
consisting of: voltage V , current I, power P = IV , resistance
R = V /I, voltage time derivative V (= dV
dt ), current time derivative I (= dI
)
and
time
t.
See
multimeter
manual for details.
dt
If you go beyond 2000 stored data points, the oldest data will
be overwritten.
IN

OUT

GND

ours on the black box and the current source are connected (you are allowed to use negative voltages).

C(V)

Nonlinear
device

Black box
It is safe to discharge the capacitor in the black box by shorting
its inputs, either by itself or through the IN and OUT terminals on multimeter: the internal resistance of this capacitor is
enough to keep the current from damaging anything.
You are not asked to estimate any uncertainties
throughout this problem.
Part A. Circuit without inductance (7 points)
In this part, keep the switch on the black box closed (push I
down), so that the inductance is shorted. Please note that
some measurements may take a considerable time, therefore it
is recommended that you read through all the tasks of part A
to avoid unnecessary work.

i. (1.0 pts) Conrm that the output current of the current


source is approximately 6 mA, and determine the range within
which it varies for voltages between 0 and +480 mV. Document
The constant current source supplies stable current as long as the circuit diagram used.
the voltage across its terminals stays between 0.6125 V and
0.6125 V. When switched o, the constant current source be- ii. (1.2 pts) Show that the dierential capacitance C(V ) used
in the black box is approximately 2 F by measuring its value
haves as a large (essentially innite) resistance.
for a single voltage of your choice C(V0 ) = C0 . Document the
circuit diagram.
-

Multimeter

+
I=6mA
U=-612.5mV...612.5mV

iii. (2.2 pts) Neglecting the nonlinearity of the capacitance


[C(V ) C0 ], determine the currentvoltage characteristic of
the nonlinear element used in the black box. Plot the I(V )
curve for obtainable positive voltages on the black box onto
the answer sheet. Document the circuit diagram.

iv. (2.6 pts) Using measurements taken from the whole range
of obtainable voltages, calculate and plot the C(V ) curve for
The black box contains an electric double layer capacitor obtainable positive voltages from the black box on the answer
(which is a slightly nonlinear high capacitance capacitor), an sheet. Write down the minimal and maximal values of dierunknown nonlinear element, and an inductor L = 10 H of ential capacitance Cmin , Cmax . Document the circuit diagram.
negligible resistance, switchable as indicated on the circuit diagram. The nonlinear element can be considered as a resistance Part B. Circuit with inductance (3 points)
with a nonlinear dependence between the voltage and the cur- Enable the inductance by opening the switch on the black box
rent [I(V ) is a continuous function of V with I(0) = 0]. Like- (push 0 down). Using the same method as in pt. A-iii, measwise, for the capacitor, the dierential capacitance C(V ) = ure and plot the current-voltage characteristic of the nonlinear element. Describe any signicant dierences between the
dQ/dV is not exactly constant.
We say that the voltage on the black box is positive curves of parts A and B and suggest a reason using qualitative
when the potential on its red terminal is higher than arguments. You need to know that the nonlinear element also
the potential on the black terminal. Positive voltage has a capacitance ( 1 nF) which is connected in parallel to
will be acquired when the terminals of matching col- the nonlinear resistance.

Current source

page 4 of 5

IPhO-measure: short manual

Display

IPhO-measure is a multimeter capable of measuring voltage V


and current I simultaneously. It also records their time derivat their product P = V I, ratio R = V /I, and time
ives V and I,
t of the sample. Stored measurements are organized into separate sets; every stored sample is numbered by the set number
s and a counter n inside the set. All saved samples are written
to an internal ash memory and can later be retrieved.
Electrical behaviour
The device behaves as an ammeter and a voltmeter connected
as follows.
Internal
Range resistance
Voltmeter
0...2V
1 M
A
V
Voltmeter 2 . . . 10 V
57 k
Multimeter
Ammeter
0...1A
1
IN

OUT

GND

Basic usage
Push Power to switch the IPhO-measure on. The
device is not yet measuring; to start measuring, push
start. Alternatively, you can now start browsing your
stored data. See below.
To browse previously saved samples (through all sets),
press Previous or Next. Hold them down longer
to jump directly between sets.
A displayed sample consists of nine variables:
While not measuring, push Start to start measuring
a new set.
1. index n of the sample in the set;
2. index s of the set;
While measuring, push Sample each time you want to
3. time t since starting the set;
store a new set of data (i.e. of the readings shown on the
4. voltmeter output V ;
display).
5. rate of change of V (the time derivative V ); if derivative
cannot be reliably taken due to uctuations, +nan/s is
While measuring, you can also browse other samples of
shown;
the current set, using Previous and Next.
6. ammeter output I;
Press Stop to end a set and stop measuring. The
if derivative
7. rate of change of I (the time derivative I);
device is still on. You are ready to start a new meascannot be reliably taken due to uctuations, +nan/s is
uring session or start browsing stored data.
shown;
8.
product P = V I;
Pushing Power turns the device o. The device will
9.
ratio R = V /I.
show text my mind is going ...; do not worry, all the
data measurements will be stored and you will be able to
browse them after you switch the device on again. Saved If any of the variables is out of its allowed range, its display
samples will not be erased.
shows +inf or -inf.

page 5 of 5

Speed of light
Notice: All measurements and calculated values must be presented with SI units with an
appropriate number of significant digits. Uncertainties required only when explicitly asked for.

1.0 Introduction
Experiments with a laser distance meter (LDM)

Figure 1.1 Equipment for the first experiments 1.1 and 1.2.
A: Laser distance meter
B: Fiber optic cable (approximately 1 m)
C: Self-adhesive black felt pads with hole
D: Tape measure
E: Tape
F: Scissors

G: Lid from the black box


A laser distance meter (LDM, see Fig. 1.2 and Fig. 1.3) consists of an emitter and a receiver. The
emitter is a diode laser that emits a modulated laser beam, i.e. a laser beam for which the amplitude
varies at a very high frequency. When the laser beam hits an object, light is reflected in all
directions from the laser dot. Some of this light returns to the instruments receiver which is situated

Page 1 of 9

E1

Speed of light
immediately next to the emitter. The instruments telescope optics is focused on the laser dot and
receives the light returned from the laser dot. The electronics of the instrument measures the time
difference in the modulation of the received light signal relative to the emitted light signal. The
delay in the modulation is exactly the time it takes for the light to travel from emitter to receiver.
The measured time is then converted to a value
1

= +
2

This value is shown in the instruments display. Here, = 2.998 108 ms 1 is the speed of light.
The constant depends on the instrument setting; on the instrument you can switch between
measuring the distance either from the rear end or from the front end of the instrument. When the
laser distance meter is turned on, the default setting is to measure from the rear. This setting shall
be maintained during all measurements.
Due to parallax, the LDM cannot measure any distance shorter than 5 cm. The maximum distance
that can be measured is around 25 m. The shape of the instrument is such that the rear side is
perpendicular to the laser beam as well as the front side. When the instrument is lying on the table
the polarization is vertical (perpendicular to the display)
The diode laser is of class 2 with power < 1 mW and wavelength 635 nm. Manifacturer
uncertainty for measurements is +/- 2 mm.
Warning: The instruments diode laser can damage your eyes. Do not look into the laser beam
and do not shine it into other peoples eyes!
Settings for LDM
The above calculation of the distance of course assumes that the light has been travelling at
speed . At the level of accuracy in this experiment, there is no need to distinguish between the
speed of light in vacuum and in air, since the refractive index for dry, atmospheric air at normal
pressure and temperature is 1.000 29 1.000.

Page 2 of 9

E1

Speed of light
Figure 1.2 The unlabeled six buttons are irrelevant (they are used to calculate area and volume). The
relevant buttons are:
A: On/off
B: Switch between measurement from the rear and the front of the instrument.
C: Indicator for measurement from the rear/front
D: Turn on laser/start measurement
E: Continuous measurement
F: Indicator for continuous measurement

Figure 1.3 The laser distance meter seen from the front end:
A: Receiver: Lens for the telescope focused on the laser dot
B: Emitter: Do not look into the laser beam!

1.1 Measurement with the laser distance meter


The instrument will perform a measurement when you press the button D, see Fig. 1.2.

1.1

Use the LDM to measure the distance from the top of the table to the floor. Write
0.4
down the uncertainty . Show with a sketch how you perform this measurement.

Page 3 of 9

E1

Speed of light
1.2 Experiment with the fiber optic cable

Figure 1.4 Diagram of a fiber optic cable.

You have been given a fiber optic cable of length approximately 1 m and diameter approximately
2 mm. The cable consists of two optical materials. The core (diameter approximately 1 mm) is
made from a plastic with a high refractive index. The core is surrounded by a cladding made from a
plastic with a slightly lower refractive index, and this is covered by a protective jacket of black
plastic. Core and cladding serve as a wave guide for light shone into the cable, since the boundary
between core and cladding will cause total reflection and thereby prevent the light from leaving
the core as long as the angle of incidence is larger than the critical angle for total reflection. The
light will therefore follow the core fiber, even if the cable bends, as long as it is not bent too much.
The LDM should now be set for continuous measurement (E, see Fig. 1.2), so that the display
indication updates approximately once per second. The LDM will automatically go into sleep
mode after a few minutes. It can be reactivated by pushing the red start button.
Carefully and gently cover the lens of the receiver with one small, black felt pad (the other is a
backup) with a hole of diameter 2 mm (see figure 1.3A). The adhesive side of the pad should be
pressed softly against the lens. Insert a fiber optic cable of length in the hole in the pad so that it
touches the lens,
see Fig. 1.5.

Page 4 of 9

E1

Speed of light

Figure 1.5 (a) Felt pad and fiber optic cable. (b) Attaching the fiber optic cable.

The other end of the cable should be held against the emitter, so that it touches the glass in the
middle of the laser beam. Now read off the -value from the display. The supplied scissors should
be used to cut the fiber optic cable into different lengths .
Think very carefully before cutting the fiber optic cable, as you cannot get any more cable!!

Notice also that the LDM display might show a thermometer icon after a while in the continuous
mode due to excessive heating of the electronics. If this happens, turn off the LDM for a while to
cool off the instrument.

1.2a

Measure corresponding values of and . Set up a table with your measurements. Draw
1.8
a graph showing as a function of .

Use the graph to find the refractive index co for the material from which the core of
1.2b the fiber optic cable is made. Calculate the speed of light co in the core of the fiber 1.2
optic cable.

Page 5 of 9

E1

Speed of light
1.3 Laser distance meter at an angle from the vertical
In this part of the experiment you will need the equipment shown in Fig. 1.6.

Figure 1.6 Equipment for experiment 1.3 shown in the figure:


A: Optical vessel with water and measuring tape
B: Magnet to secure the angle iron on top of the black box. (You find magnet placed on the angle iron).
C: Angle iron with self-adhesive foam pads
D: Self-adhesive foam pads

Remove the black felt pad from the lens. The LDM should now be placed in the following set-up:
Place two self-adhesive foam pads on the angle iron, see A on Fig. 1.7.

Page 6 of 9

E1

Speed of light

Figure 1.7 How to place the two self-adhesive foam pads on the angle iron.

The LDM should be carefully placed on the angle iron as shown in Fig. 1.8.

Figure 1.8 How to place the laser distance meter on the angle iron.

The angle iron with the LDM should be mounted on the black box as shown in Fig. 1.9. Secure the
angle iron to the box with a magnet placed below inside the box. (The tiny magnet is found on the
angle iron). It is important to mount the LDM exactly as in the photo, since the side of the box
facing upwards slants by approximately 4 degrees. The laser beam should now be pointing
unobstructedly downwards at an angle.

Page 7 of 9

E1

Speed of light

Figure 1.9 The experimental set-up. (The black box only serves as a support. The equipment behind the
bottle is not used, though).
A: Important: The bottom of the black box must face forward as shown. The side that faces upwards
is slanting approximately 4 degrees with respect to the horizontal plane. Make sure that the angle is
the same all the time

When the LDM is turned on and mounted as explained above, the laser beam will form an angle 1
with respect to the vertical direction. This angle, which must be the same throughout this
experiment, must now be determined. The optical vessel is not needed here, so put it aside so far.
Measure with the LDM the distance 1 to the laser dot where the laser beam hits the
table top. Then move the box with the LDM horizontally until the laser beam hits the
1.3a
0.2
floor. Measure the distance 2 to the laser dot where the laser beam hits the floor. State
the uncertainties.
1.3b

Calculate the angle 1 using only these measurements 1 , 2 and (from problem 1.1).
0.4
Determine the uncertainty 1 .

Page 8 of 9

E1

Speed of light

E1

1.4 Experiment with the optical vessel


Place the optical vessel so that the laser beam hits the bottom of the vessel approximately in the
middle, see Fig. 1.10. Pour some water into the vessel. The depth of the water is . Read off on
the display of the LDM.
LDM

1 1
2

Figure 1.10 Diagram of laser beams in the optical vessel with water of depth .

1.4a

Measure corresponding values of and . Set up a table with your measurements. Draw
1.6
a graph of as a function of .

1.4b Use equations to explain theoretically what the graph is expected to look like.

1.2

1.4c Use the graph to determine the refractive index w for water.

1.2

Page 9 of 9

Experimental competition. Thursday, 17 July 2014

1/9

Experiment. To see invisible! (20 points)


Introduction
Many substances exhibit optical anisotropy resulting in that the refractive index turns out dependent
on the direction of light propagation and its polarization. Optical anisotropy can occur even in isotropic
media in the presence of mechanical stresses, nonuniform heating or application of external electric fields.
The direction in which the light propagates without the birefringentis called the optic axis of a crystal.
Consider a traditional optic scheme of experiments for studying the optical anisotropy (see Fig. 1),
which is to be used in this experimental problem.

Fig. 1. Optic scheme of an experiment for studying the optical anisotropy.


Let a light beamfall onto polarizer 1 whose transmission plane intersects its ownplane along the
straight line 1 1 . After passing polarizer 1 the light beam becomes linearly polarized and its electric field
strength vector 0 oscillatesexactly in the transmition plane of polarizer 1. Then, the light beam falls onto
the anisotropic plate P oriented such that its optical axislies in the plate plane to makethe angle 45 with
the transmission plane of polarizer 1. Two kinds of light waves are then generated in the plate P: ordinary
, polarized perpendicular to the optical axis of the plate, and extraordinary , polarized along the optical
axis of the plate. Refractive indices for these two waves are different and their difference is denoted as
= . This results in the appearanceof the phase difference = 2/( being the plate
thickness, being the wavelength of the incident light in vacuum) between the two waves on leaving the
plate. Therefore, the polarization of the outgoing light beam changes to be elliptically polarized. The light
beam then falls onto polarizer 2, whosetransmission plane 2 2 is perpendicular to the transmission plane of
polarizer 1.
A simple derivation shows that the intensity of the light beam transmitted through the plate P and
polarizer 2 is determined as

2 = 0 sin2 2 ,
(1)
where0 stands for the light intensity falling onto the plate, denotes the light transmittance coefficient of the
plate P and polarizer 2, and designates the phase difference between the ordinary and extraordinary
waves after passing the plate P.

In this experiment do not evaluate errors unlessasked to do so!


The description of the equipment in Appendix A

Experimental competition. Thursday, 17 July 2014

2/9

Part 1.Qualitative observations!(3.5 points)


Part 1.1.Polarizers(0.8 points)
1.1

Find the orientation(i.e. which of the diagonals) of the transmission plane of polarizer 1 and
polarizer 2. Show these planes in the figure in the Answer Sheet. (0.8 points)

Part 1.2.Rulers(1.0 points)


In this part of the experiment uselight emitting diodes(LED) as a light source.
Fix LED on a stand and connect it to its power supply. Set up both polarizers by their face sides
(indicated bynumbers 1 and 2) pointing towards the light source. Make sure that the polarizers are crossed,
i.e. the light beamcannot pass through them. Block the first polarizer by a sheet of white paperplacing it on
the face side as shown in Fig.1B in Appendix B.
Place the plastic ruler1between the polarizers. You can move the ruler with your hands.
1.2.1 Find the possible directions of the optical axis in thecenter of the plastic ruler. Show these
directions in the figure on the Answer Sheet. (0.4 points)
1.2.2 Determine approximately at what distance along the ruler 1 and along the two rulersstacked
together, the phase difference for the blue light changes to 2. (0.6 points)

Part 1.3.Strip(0.8 points)


1.3.1

Find the possible directions of the optical axis of the strip. Show qualitatively them in the figure in
the Answer Sheet. (0.4 points)

Using the clamps fix a long flexible plastic strip on the screen so that the strip edges coincide with the
sreenedges. The stripshould be curved (see Fig. 3B). Place the screen with the strip between the polarizers.
Shifting the screen, observe the color change of the strip. Measure x coordinates of strip points on the screen
scale, use the left edge of the screenholder as an origin as shown in Figure 3B.
Hereinafter coordinates are measured by the scale in the screen. As a pointer, use the left edge of the
holder, which indicated in Fig. 3B by the arrow!
1.3.2

Measure the coordinates of the middle points of two dark bands, the left and the right ,
visible on the strip. (0.4 points)

Part 1.4.Liquid crystal cell(0.9 points)


Liquid crystal (LC) is a state of matter that is
intermediate between the crystalline solid and the amorphous
liquid. The orientation of its molecules can be easily aligned
and controlled by applying an electrical field. The LC cell
exhibits the optical anisotropy phenomenon with two principal
refractive indices. The magnitude of this effect depends on the
applied AC voltage. The Liquid Crystal Cell (LCC) is
composed of two glass plates 1 whose inner surface is coated
with a transparent conductive layer 2. Between the plates there
is a thin (approximately 10 microns) layer of the solution 3
which is in a liquid crystal state. Leads are soldered to the
plates for connecting to the AC power supply.
Place the liquid crystal cell (LCC) between the polarizers. Plug it into its power supply. Varying the
voltage across the LCC observe the changing colors of the light transmitted through it.
1.4.1 Find the possible directions of the optical axis of the LCC at zero and maximum voltageapplied
across it. Show these directions in the figure on the Answer Sheet. Z-axis is directed vertically.(0.6
points)

Experimental competition. Thursday, 17 July 2014

3/9

1.4.2 Measure the voltage across the cell at which abrupt,with respect to voltage, reorientation of
liquid crystal molecules by 90 occur.Make sure that multimeter is on AC voltage mode.(0.3
points).

Part 2.Measure!(16.5 points)


Disconnect the LED from the power supply and remove it. Remove the sheet of white paper.
In this part of the work use laser as a light source, make sure you connect it to its power supply!
Fixthe laser, polarizer 1, the screen with the slit and a photodetector (a photodiode) in the holders.
Adjust the setting so that the laser beam passes through the polarizer andthe slit of the screen to fall strictly
onto the photodiode. Using the screw 5c for the beam width adjustment, make sure that the spot size on the
photodetectoris about 5-6 mm.
The laser emits a linearly polarized light. With the ring 5a designed for the laser reorientation,make
sure that the laser beam almost completely passes through the first polarizer and the major axis of the
elliptically-shaped spot is vertical. In the following,the orientations of the laser and the photodetectormust be
fixed with the screws 5d and15c. Set up polarizer 2. Make sure that the polarizers are crossed. Fig. 4B shows
the whole setup with the screen mounted.

Part2.1.Investigating a photodiode(3.2 points)


For measuringthe light intensity,of use is the photodiode
EMF which is a rather complicated function of the incident light
intensity itself. Therefore, for measuring the light intensity the
circuit shown in Fig. 2 is used. The DC voltage measured by a
multimeter depends on the incident light intensity and the
resistance of a resistor. The main objective is to choose such an
optimum value of the resistance thatthe voltage across the
resistor is to be proportional to the intensity of the light
incident on the photodiode.

Fig. 2. Circuit for measuring the


photodiode emf.

For measurements in this part, remove the second polarizer and the screen from the optical bench.
The filters that attenuate the beam intensity should be necessarilyfixed with the clamps on the back side
of the polarizer as shown in Fig. 5B.
The maximum values of the measured voltages must be at least 300 mV.
Using a multimeter, you can measure the resistance of the resistor and the voltage across it (of
course, you have to properlyadjust the register of the multimeter). Add, in an appropriate place,the switch
provided so that you could measure both the resistance and the voltage with the single multimeter, i.e.
without disconnecting the circuit by just shorting/unshorting the switch and the multimeter register
adjustment.
2.1.1 Draw a circuit with the switch installed for measuring of the voltage across the resistor and its
resistance. (0.2 points)
2.1.2 Measure the voltage across the resistor as a function of its resistance for two values of the incident
light intensity: maximum (with the number of filters = 0) and the minimum (with the number of
filters = 5). Plot the corresponding graphs in the same figure. Specify the range of the resistance
for which the difference between the voltages is maximal. (1.0 point)
2.1.3 Measure the voltage across the resistor as a function of the number of filters = 0,1,2,3,4,5,
attenuating the intensity of the incident light on the photodiode. Measurements must be carried out
at three fixed values of the resistance, approximately equal to = 30 , = 20 and
= 10 .Plot the corresponding graphs in the same figurechoosing the scale such that it
would be possible to verify whether the voltage across the resistor depends linearly on the intensity
of the incident light registered by the photodiode. From the three above mentioned values of the
resistance choose an optimum one at which further measurements of the light intensity should
be made. (1.0 point)
2.1.4 Using this data obtained, calculate the transmittance of the filter = / and evaluate its

Experimental competition. Thursday, 17 July 2014

4/9

error,with being the intensity of the transmitted light, and being the intensity of the incident
light. You can make additional measurements if necessary. (1.0 point)
All subsequent measurements must be made at the optimal value of the resistor chosen!
It is assumed in what follows that the light intensityin relative units is equal to the voltage across the
resistor in mV.

Part 2.2 Light transmission through plastic rulers(5.4 points)


Place the plastic ruler between the polarizers. You can move the ruler with your hands. Then, secure
it with the clamps on the screen with a slit (see Fig. 2B). The lower edge of the ruler should coincide with a
line drawn on the screen, and its scale should be at the top. Make sure that both of the rulers provided
demonstrate the birefringence phenomenon. Observe a picture emerging when you put one ruler on the top
of the other so that light is to pass through both of them.
In this part use an optical scheme shown in Fig. 4B. Make sure that the rulersare fixed on the screen
at the position described in Part 1.2.
2.2.1 Measure the intensity of the transmitted light (in mV) as a functionof the coordinate of the point
of the light incidence on the ruler in the range from 0 to 10 cm.. Measurements must be carried out
for each ruler provided and for the two rulers stacked together. In each case, measure the
maximum value of the voltage. Plot the corresponding graphs in the same figure. (2.0 points)
2.2.2 For each of the two rules calculate the values of the phase shift between the ordinary and
extraordinary waves in the range of from 0 to 7 cm.Plotthe correspondinggraphs().Put down
the formula you have used for calculations. (1.2 points)
Note that the phase difference cannot unambiguously be determined from formula
(1),additional physical assumptions should be applied to determine it correctly.
2.2.3 Assuming that () is linear for each ruler
1 = 1 + 1 ,
2 = 2 + 2 ,
calculate the numerical values of theabove coefficients for rulers 1 and 2. (1.0 point)
2.2.4 Using those data obtained in parts 2.2.1-2.2.3, calculate the theoretical values of the intensity of
light passing through the two rulersstacked together. Put down the formula that you have used for
calculations. Plot the theoretical dependence in the same figure from part 2.2.1. (1.2 point)

Part 2.3 Liquid crystal cell(4.5 points)


Light transmission through LCC
Place the LCC between the polarizers as shown in Fig. 6B.
The experimental dependence under investigation is strongly nonmonotonicwith domains of quite abrupt
changes. Take this into account when taking measurements.
To measure the AC voltage of the cellpower supply and the DC voltage of the photodetector, connect the
appropriate leads directly to the multimeter.
2.3.1
2.3.2
2.3.3

Measure the intensity of the transmitted light as a function of the voltage across the LCC. Plot
the corresponding graph. (2.0 points)
Calculate the phase difference between the ordinary and extraordinary waves 0 when the
power supply is disconnected with the LCC. (1.5 points)
In a sufficiently wide range of the voltage dropacross the LCC, the phase difference between the
ordinary and extraordinary waves depends on the voltage applied by the power law
= .
Using the data obtained, plot the graphwhich allows one to determine the applicability range of
the above formula and calculate the exponent. Specify that range of applicability and evaluate
the numerical value of the parameter. (1.0 point)

Experimental competition. Thursday, 17 July 2014

5/9

Part 2.4.Light transmission through a curved strip (3.4 points)


Secure the plastic strip on the screen as described in part 1.3.
2.4.1 Measure the intensity of light, transmitted through the optical system,as a function of the
coordinatexof thepoint of light penetration into the strip in the range of 20 from its center.
Plot the corresponding graph. (1.2 point)
2.4.2 Calculate the phase shift between the ordinary and extraordinary waves0 , passing through
anuncurved strip. It is known that 0 lies in the range of10to 12. (1.2 points)
Near the center of the strip its shape may be approximated a circular arc of radius R. The theoretical
dependence of the phase shift on the distance z from the center of the strip, zR, has the form:
2

=0 1 + 2 2 2 ,
where = 1.4, is the refractive index of the strip material
2.4.3 Using the data obtained in the previous parts, calculate the radius of strip curvature near its
center. The refractive index of the strip material is equal to = 1.4. (1.0 points)

Experimental competition. Thursday, 17 July 2014


Appendix A. Experimental equipment
Optical bench 1with holders:
1a support for the light source with the
screw;
1b support for the photodetector with the
screw;
1c, 1d stands for the polarizers;
1 stand for the screen and the liquid
crystal cell (LCC).
2a, 2b polarizers on the stands. There are
numbers 1 and 2 on their face sides.
The polarizers should be installed with
their face sides towards the light source!
The transmission planes of the polarizers
makes the angle 45with the horizontal.
3 the screen with the slit (3a) and the scale
(3b)

Light sources:
4 light emitting diode (LED):
4a leads for the power supply;
4b fixing screw;
5 laser:
5a ring for turning the laser with the
scale(scale is not used);
5b leads for the power supply;
5c the screw for the beam width
adjustment on the front side;
5d fixing screw;
6 power supply for light sources:
6a switch;
6b leads for light source.
Keep the source operating only while
making measurements!
Do not point the laser beam in youror
anyones eyes, it is very dangerous!
7 variable resistor:
7a, 7b, 7c terminals for connection to a
circuit
7d knob for changing resistance.
8 switch:
8a, 8b terminals for connection to a circuit;

6/9

Experimental competition. Thursday, 17 July 2014


9a liquid crystal cell (LCC) 9ain a holder
(9b), 9 lead for connecting to the power
supply;
10 the power supply for LCC:
10a connector for the LCC;
10b leads for measuring the output
voltage;
10c knob to adjust the output voltage;
10d power on/off.
Keep the source operating only while
making measurements!
11 multimeter;
Do not press the HOLD button
11a register to measure resistance
(200 kOhm);
11b register to measure DC voltage (2V);
11c register to measure AC voltage (20V);
11d, 11e connectors for test leads;
11f power on/off.
If the display multimeter is in a "sleep"
mode - double-press power on/off!
When measuring the resistance with the
multimeter, the element must be
disconnected with a power supply!

Optical elements to be investigated


12 plastic rulers:
12a No. 1 (with the scalefrom 0 to 14 cm)
12b No. 2 (with the scalefrom20 to34cm)
13 flexible strip;
14 set of identical filters;
The filters and the strip are provided in a
separate envelope!
The plastic rulers and the strip exhibit
birefringence, their optical axes lie in
theirown planes.
15 Photodetector (photodiode)
15a - input window;
15b - leads for measuring the output voltage;
15c fixing screw.

Connecting wires, clamps, paper napkin,


a piece of paper.

7/9

Experimental competition. Thursday, 17 July 2014

8/9

The liquid crystal cell is composed of two glass plates


1 whose inner surface is coated with a transparent conductive
layer 2. Between the plates there is a thin (approximately 10
microns) layer of the solution 3 which is in a liquid crystal
state. Leads are soldered to the plates for connecting to the
AC power supply. In the absence of voltage long molecules
of liquid crystal are oriented parallel to the plates. The
direction of molecular orientation coincides with the optical
axis of the crystal.
Try not to touch those parts of the optical elements
through which the light passes! If necessary, wipe them
with a paper napkin!

Appendix B. Photos of the experimental setups

Fig. 1B Setup for the observation of birefringence Fig. 2B Mounted ruler on the screen
in the ruler

Fig. 3B Fixing flexible plastic strip to the screen.

Fig. 4B Setup for measuring the transmittance of light


through plastic ruler

Experimental competition. Thursday, 17 July 2014

Fig. 5B Mounted filters on the polarizer

9/9

Figure 6B Setup for measuring the characteristics of


the LCC

E.P.1.
The following devices and materials are given:

1. Balance (without weights)


2. Calorimeter
3. Thermometer
4. Source of voltage
5. Switches
6. Wires
7. Electric heater
8. Stop-watch
9. Beakers
10.Water
11.Petroleum
12.Sand (for balancing)

Determine specific heat of petroleum. The specific heat of water is 1


cal/(gC). The specific heat of the calorimeter is 0.092 cal/(gC).
Discuss assumptions made in the solution.
E.P.2.
Three closed boxes (black boxes) with two plug sockets on each are
present for investigation. The participants have to find out, without
opening the boxes, what kind of elements are in them and measure their
characteristic properties. AC and DC meters (their internal resistance and
accuracy are given) and AC (5O Hz) and DC sources are put at the
participants disposal.
E.P.3.
Determine the focal distances of lenses.

List of instruments: three different lenses installed on posts, a screen


bearing an image of a geometric figure, some vertical wiring also fixed
on the posts and a ruler.
E.P.4.
Apparatus: dc source, ammeter, voltmeter, rheostat (coil of high
resistance wire with sliding contact), and connecting wires.
Problem: Construct appropriate circuit and establish the dependence of
the electric power P dissipated in the rheostat as a function of the current
I supplied by the dc source.
1. Make a plot of P versus I.
2. Find the internal resistance of the dc source.
3. Determine the electromotive force E of the source.
4. Make a graph of the electric power P versus resistance R of the
rheostat.
5. Make a graph of the total power Ptot dissipated in the circuit as a
function of R.
6. Make a graph of the efficiency of the dc source versus R.
E.P.5.
There are given two cylindrical bodies (having identical external
shapes and from the same material), two measuring rules, one
graduated and other un-graduated, and a vessel with water.
It is known that one of the bodies is homogenous and the other has
an internal cavity with the following characteristics:
- the cavity is cylindrical
- has the axis parallel with the axis of the body
- its length is practically equal with that of the body
Determine experimentally and justify theoretically:
a) The density of the material the two bodies consist of.
b) The radius of the internal cavity.
c) The distance between the axis of the cavity and the axis of the
cylinder.

d) Indicate the sources of errors and appreciate which of them


influences more the final results.
Write all the variants you have found.
E.P.6.
In a "black box" there are two identical semiconducting diodes and
one resistor connected in some unknown way. By using
instruments provided by the organizers find the resistance of the
resistor.
Remark: One may assume that the diode conducts current in
one direction only.
List of instruments: two universal volt-ammeters (without
ohmmeters), battery, wires with endings, graph paper, resistor with
regulated resistance.
E.P.7.
At the workplace there are beyond other devices a test tube with 12
V electrical heating, a liquid with known specific heat (c0 = 2.1
J/gC) and an X material with unknown thermal properties. The X
material is insoluble in the liquid.
Examine the thermal properties of the X crystal material between
room temperature and 70 C. Determine the thermal data of the X
material. Tabulate and plot the measured data.
(You can use only the devices and materials prepared on the table.
The damaged devices and the used up materials are not
replaceable.)
E.P.8.
Define the electrical circuit scheme in a black box and determine
the parameters of its elements.
List of instruments: A DC source with tension 4.5 V, an AC source
with 50 Hz frequency and output voltage up to 30 V, two
multimeters for measuring AC/DC current and voltage, variable
resistor, connection wires.
E.P.9.
Materials and Instruments: elastic rubber cord (the length of free
cord is l0 = 150 mm), vertically hanged up to a stand, set of weights

from 10 g to 100 g, pan for the weights with mass 5 g, chronometer,


ruler, millimeter (scaled) paper.
Note: The Earth Acceleration is g = 10 m/s2. The mass of the
rubber cord can be neglected.

Make the following study:


1. Load the rubber cord with weights in the range 15 g to
105 g. Put the data obtained into a table. Make a graph (using
suitable scale) with the experimentally obtained dependence of the
prolongation of the cord on the stress force F.
2. Using the experimental results, obtained in p.1, calculate
and put into a table the volume of the cord as a function of the
loading in the range 35 g to 95 g. Do the calculations consequently
for each two adjacent values of the loading in this range. Write
down the formulas you have used for the calculations. Make an
analytical proposition about the dependence of the volume on the
loading.
Assume that Youngs modulus is constant: E = 2.106 Pa.
Take in mind that the Hookes law is only approximately valid and
the deviations from it can be up to 10%.
3. Determine the volume of the rubber cord, using the
chronometer, at mass of the weight equal to 60 g. Write the
formulas used.
E.P.10.
You have at your disposal the following material:

(1) A sine wave voltage generator set to a frequency of 0,20 kHz.


(2) A dual ray oscilloscope.
(3) Millimeter graph paper.
(4) A diod.
(5) A capacitor of 0.10

F (square and black).

(6) An unknown resistor R (red).

(7) A coupling plate.


(8) Coupling wires.

Build the circuit shown in the


figure.

A
0.1 F

Connect the terminals A and B to the sine wave generator set to a


frequency of 0.20 kHz. Determine experimentally the mean power
developed in the resistor R when the amplitude of the generator
voltage is 2.0 V (that is the peak-to-peak voltage is 4.0 V).
E.P.11.
Material:
(1) A glow discharge lamp connected to 220 V, alternating current.
(2) A laser producing light of unknown wavelength.
(3) A grating.
(4) A transparent micro-ruler, 1 mm long with 100 subdivisions,
the ruler is situated exactly in the centre of the circle.
(5) A 1 m long ruler
(6) Writing material.

The spectrum of the glow discharge lamp has a number of spectral


lines in the region yellow-orange-red. On of the yellow lines in the
short wavelength part of this spectrum is very strong. Determine
the wavelength of this spectral line. Estimate the accuracy of your
measurement.

Note: If you happen to know the wavelength of the laser light


beforehand you are not allowed to use that value in your
computation.

Warning. Do not look into the laser beam. Do not touch the surface
of the grating or the surface of the transparent micro-ruler.
E.P.12.
APPARATUS
1.

Spectrometer with collimator and telescope.

2.
B.

3 syringes; one for water, one for liquid A and one for liquid

3.
A beaker of water plus two sample tubes containing liquids
A and B.
4

3 retort stands with clamps.

5.

12V shielded source of white light.

6.

Black card, plasticine, and black tape.

7.
2 plastic squares with holes to act as stops to be placed over
the ends of the telescope, with the use of 2 elastic bands.
8.

Sheets of graph paper.

9.
Three dishes to collect water plus liquids A and B lost from
syringes.
Please complete synopsis sheet in addition to answering this
experimental problem.

Pendant drop

Collimator
Drop

Light

Plan of Apparatus

Telescope

INSTRUCTIONS AND INFORMATION


1. Adjust collimator to produce parallel light. This may be
performed by the following sequence of operations:
(a) Focus the telescope on a distant object, using adjusting knob on
telescope, so that the cross hairs and object are both in focus.
(b) Position the telescope so that it is opposite the collimator with
slit illuminated so that the slit can be viewed through the telescope.
(c) Adjust the position of the collimator lens, using the adjusting
knob on the collimator, so that the image of the slit is in focus on
the cross hairs of the telescope's eyepiece.
(d) Lock the spectrometer table, choosing an appropriate 'zero' on
the vernier scale, so that subsequent angular measurements of the
telescope's position can conveniently be made.

2.
Remove the eyepiece from telescope and place black plastic
stops symmetrically over both ends of the telescope, using the
elastic bands, so that the angle of view is reduced.
3.

Open up collimator slit.

4.
Use the syringes to suspend, vertically, a pendant drop
symmetrically above the centre of the spectrometer table so that it
is fully illuminated by the light from the collimator and can be
viewed by telescope.
5.
The central horizontal region of the suspended drop will
produce rainbows as a result of two reflections and k (k = 1,2,...)
internal reflections of the light. The first order rainbow corresponds
to one internal reflection. The second order rainbow corresponds to
two
Internal reflections. The k'th order rainbow corresponds to k
internal reflections. Each rainbow contains all the colours of the
spectrum. These can be observed directly by eye and their angular
positions can bed accurately measured using the telescope. Each
rainbow is due to white light rays incident on the drop at a well
determined angle of incidence, that is different for each rainbow.

6.
The first order rainbow can be recognized as it has the
greatest intensity and appears on the right hand* side of the drop.
The second order rainbow appears with the greatest intensity
on the left hand* side of the drop. These two rainbows are within an
angular separation of 20 of each other for water droplets. The
weak intensity fifth order rainbow can be observed on the right
hand side of the drop located somewhere between the other two,
'blue', extreme ends of the first and second order rainbows.
7.
Light reflected directly from the external surface of the drop
and that refracted twice but not internally reflected, will produce
bright white glare spots that will hinder observations.
8.

The refractive indices, n, of the liquids are:

Water nw = 1.333
Liquid A

nA = 1.467

Liquid B

nB = 1.534

In addition to the experimental report please complete the summary


sheet.

Footnote: This statement is correct if the collimator is to the left of


the telescope, as indicated in the diagram. If the collimator is on
the righthand side of the telescope the first order rainbow will
appear on the lefthand side of the drop and the second order
rainbow on the righthand side of the drop.

Measurements

1) Observe, by eye, the first and second order water rainbows.


Measure the angle through which the telescope has to be rotated,
from the initial direction for observing the parallel light from the
collimator, to observe, using a pendant water droplet, the red light
at the extreme end of the visible spectrum from:

(a)

the first order rainbow on the right of the drop (k = 1);

(b)

the second order rainbow on the left of the drop (k =

2);
(c) the weak fifth order rainbow (k = 5), between the
first and second order rainbows.

One of these angles may not be capable of measurement by the


rotation of the telescope due to the mechanical constraints limiting
the range of . If this is found to be the case, use a straight edge in
place of the telescope to measure .
(Place the appropriate dish on the spectrometer table to catch any
falling droplets.)
Deduce the angle of deviation, , that is the angle the incident
light is rotated by the two reflections and k reflections at the drop's
internal surface, for (a), (b) and (c). Plot a graph of against k.

2.Determine for the second order rainbows produced by liquids


A and B using the red visible light at the extreme end of the visible
spectrum. (Place respective dishes on table below to catch any
falling liquid as the quantities of liquid are limited).
cos

1
n

Using graph paper plot 6 against , n being the refractive index,


for all three liquids and insert the additional point for n = 1. Obtain

the best straight line through these points; measure its gradient and
the value of for which n = 2.
E.P.13.
Apparatus
RML Nimbus computer
Ten sheets of graph paper.
Please complete synopsis sheet in addition to answering this
experimental problem.

INFORMATION

The microcomputer has been programmed to solve the Newtonian


equations of motion for a two-dimensional system of 25 interacting
particles, in the xy plane. It is able to generate the positions and
velocities of all particles at discrete, equally spaced time intervals.
By depressing appropriate keys (which will be described), access
to dynamic information about the system can be obtained.
The system of particles is confined to a box which is initially (at
time t = 0) arranged in a two-dimensional square lattice. A picture
of the system is displayed on the screen together with the
numerical data requested. All particles are identical; the colours are
to enable the particles to be distinguished. As the system evolves in
time the positions and velocities of the particles will change. If a
particle is seen to leave the box the program automatically
generates a new particle that enters the box at the opposite face
with the same velocity, thus conserving the number of particles in
the box.
Any two particles i and j, separated by a distance rij interact with a
well-defined potential Uij,

It is convenient to use dimensionless quantities throughout the


computation. The quantities given below are used throughout the
calculations.

Variable

Symbol

Distance
Velocity

r*
v*

Time

t*

Energy
Mass of
particle
Potential

E*
M* = 48

Temperatur
eKinetic
Energy

T*

Uij*
Ek *

1
m * v *2
2

INSTRUCTIONS

The computer program allows you to access three distinct sets of


numerical information and display them on the screen. Access is
controlled by the grey function keys on the left-hand side of the
keyboard, labelled F1, F2, F3, F4, and F10. These keys should be
pressed and released - do not hold down a key, nor press it
repeatedly. The program may take up to 1 second to respond.

FIRST INFORMATION SET. PROBLEMS 1 5

v x , n

1 25 * n
vix
25 i 1

v y , n

1 25 * n
viy
25 i 1

and
U

1 25

25 j 1

25

U
i 1

*
ij

i j

where
vix*

is the dimensionless x component of the velocity for the ith


particle,
viy*

is the dimensionless y component of the velocity for the ith


particle,
and n is and integer with

n 1.

[Note: the summation over

U iy*

excludes the cases in which i = j]

After depressing F1 it is necessary to input the integer n (n 1) by


depressing one of the white keys in the top row of the keyboard,
before the information appears on the screen.

The information is displayed in dimensionless time intervals t at


dimensionless times

S t**

(S = 0, 1, 2, .....)

t** is set by the computer program to the value t** = 0100000.

The value of S is displayed at the bottom right hand of the screen.


Initially it has the value S = 0. The word "waiting" on the screen

indicates that the calculation has halted and information concerning


the value of S is displayed.

Depressing the long bar (the "space" bar) at the bottom of the
keyboard will allow the calculation of the evolution of the system
to proceed in time steps **t. The current value of S is always
displayed on the screen. Whilst the calculation is proceeding the
word "running" is displayed on the screen.

Depressing F1 again will stop the calculation at the time integer


indicated by S on the screen, and display the current values of
<vx,n>, <vy,n> and <U>
after depressing the integer n. The evolution of the system
continues on pressing the long bar.
The system can, if required, be reset to its original state at S = 0 by
pressing F10 TWICE.

SECOND INFORMATION SET: PROBLEM 6

Depressing F2 initiates the computer program for the compilation


of the histogram in problem 6. This program generates a histogram
table of the accumulated number N, of particle velocity
components as a function of dimensionless velocity. The
dimensionless velocity components, vx and vy are referred to
collectively by vc. The dimensionless velocity range is divided into
equal intervals vc = 0.05. The centres of the dimensionless
velocity "bins" have magnitudes

vc Bvc
*

(B 0,1 2,..........)

When the long bar on the keyboard is pressed the 2 x 25


dimensionless velocity components are calculated at the current
time step, and the program adds one, for each velocity component,
into the appropriate velocity 'bin'. This process is continued, for
each time step, until F3 is depressed. Once F3 is depressed the
(accumulated) histogram is displayed. The accumulation of counts
can then be continued by pressing the long bar. (Alternatively if
you wish to return to the initial situation, with zero in all bins, press
F2).
The accumulation of histogram data should continue for about 200
time steps after initiation.

In the thermodynamic equilibrium the histogram can be


approximated by the relation
N Ae

24 ( v * ) 2
c

where is a constant associated with the temperature of the system,


and A depends on the total number of accumulated velocity
components.

THIRD INFORMATION SET: PROBLEM 7

Depressing F4 followed by the long bar at any time during the


evolution of the system will initiate the program for Problem 7.
The program will take some 30 seconds, in real time, before
displaying a table containing the two
Quantities

1
RX ,2
25

x (S ) x (SR)
25

*
i

*
i

i 1

and
1
RY ,2
25

y (S ) y (SR)
25

*
i

*
i

i 1

where xi* and yi* are the dimensionless position components for the
i'th particle. S is the integer time unit and SR is the fixed initial
integer time at which the programme is initiated by depressing F4.
It is convenient to introduce integer
SZ = S - SR..

The programme displays a table of <RX,2> and <RY,2> for


SZ = 0, 2, 4..24.

Prior to the display appearing on the screen a notice 'Running' will


appear on
the screen indicating that a computation is proceeding. Depressing
F4, followed by the long bar, again will initiate a new table with SR
advanced to the point at which F4 was depressed.

COMPUTATIONAL PROBLEMS

1
Verify that the dimensionless total linear momentum
of the system is conserved for the times given by
S = 0, 40, 80, 120, 160.
State the accuracy of the computer calculation.
2.
Plot the variation in dimensionless kinetic energy of the
system with time using the time sequence
S = 0, 2, 4, 6, 12, 18, 24, 30, 50, 70, 90, 130, 180.

3.
Plot the variation in dimensionless potential energy of the
system with time using the time sequence in 2.

4.
Obtain the dimensionless total energy of the system at times
indicated in 2. Does the system conserve energy? State the
accuracy of the total energy calculation.

5.
The system is initially (at S = 0) NOT in thermodynamic
equilibrium. After a period of time the system reaches
thermodynamic equilibrium in which the total dimensionless
*
kinetic energy fluctuates about a mean value of E k . Determine this
*
value of E k and indicate the time, SD, after which the system is in
thermodynamic equilibrium.

6.
Using the dimensionless accumulated velocity data, during
thermodynamic equilibrium, draw up a histogram giving the
number N of velocity components against dimensionless velocity
component, using the constant velocity component interval Vc* =
0.05, specified in the table available from the SECOND
INFORMATION SET. Data accumulated from approximately 200
time steps should be used and the starting time integer S should be
recorded.

Verify that N satisfies the relation


N Ae

* 2
24 vC

where C and A are constants. Determine the value of

7.
For the system of particles in thermodynamic equilibrium
2
2
evaluate the average value of R , R , where R is the straight line
distance between the position of a particle at a fixed initial time
number SR and time number S. The time number difference SZ = (S
- SR) takes the values
SZ = O, 2, 4, .. 24.
Plot <R2> against SZ for any appropriate value of SR. Calculate the
gradient of the function in the linear region and specify the time
number range for which this gradient is valid.
In order to improve the accuracy of the plot repeat the previous
calculations for three (additional) different values of SR and
determine the AVERAGE <R2> for the four sets of results together
with the 'linear' gradient and time number range.
Deduce, with appropriate reasoning, the thermodynamic
equilibrium state of the system, either solid or liquid.
E.P.14.
Find the refractive indices of a prism, np, and a liquid, nl. Ignore
dispersion.
a) Determine the refractive index np of a single prism by two
different experimental methods. Illustrate your solution with
accurate diagrams and deduce the relations necessary to calculate
the refractive index. (One prism only should be used).
b) Use two identical prisms to determine the refractive index nL of
a liquid with nL < np.
Illustrate your solution with accurate diagrams and deduce the
relations necessary to calculate the refractive index.

Apparatus: Two identical prisms with angles of 30, 60 and 90; a


set square, a glass dish, a round table, a liquid, sheets of graph
paper, other sheets of paper and a pencil.
Formulae: sin(a b) = sina * cosb cosa * sinb
Additional remarks: You may mark the opaque sides of the prisms
with a pencil. The use of the lamp is optional.
E.P.15.
Equipment:
one electric tungsten bulb made of frosedt-surface glass complete
with mounting stand, 1 set
3 wooden clamps, each of which contains a slit for light
experiment
2 glass plates; one of which is rectangular and the other one is
square-shaped
1 polaroid sheet (circular-shaped)
1 red film or filter
1 roll self adhesive tape
6 pieces of self-adhesive labelling tape
1 cellophane sheet
1 sheet of black paper
1 drawing triangle with a handle
1 unerasable luminocolour pen 312, extra fine and black colour
1 lead pencil type F
1 lead pencil type H
1 pencil sharpener
1 eraser
1 pair of scissors

Important Instructions to be Followed

1. There are 4 pieces of labelling tape coded for each contestant.


Stick the tape one each on the instrument marked with the sign #.
Having done this, the contestant may proceed to perform the
experiment to answer the questions.
2. Cutting, etching, scraping or folding the polaroid is strictly
forbidden.
3. If marking is to be made on the polaroid, use the lumino-colour pen
provided and put the cap back in place after finishing.
4. When marking is to be made on white paper sheet, use the white
tape.
5. Use lead pencils to draw or sketch a graph.
6. Black paper may be cut into pieces for use in the experiment, but
the best way of using the black paper is to roll it into a cylinder as
to form a shield around the electric bulb. An aperture of proper size
may be cut into the side of the cylinder to form an outlet for light
used in the experiment.
7. Red piece of paper is to be folded to form a double layer.

The following four questions will be answered by performing the


experiment:

Question 1.1

1.1.a

Locate the axis of the light transmission of the polaroid film. This
may be done by observing light reflected from the surface of the
rectangular glass plate provided. (Light transmitting axis is the
direction of vibration of the electric field vector of light wave
transmitted through the polaroid). Draw a straight line along the
light transmission axis as exactely as possible on the polaroid film.
(#)

1.1.b
Set up the apparatus on the graph paper for the experiment to
determine the refractive index of the glass plate for white light.

When unpolarized light is reflected at the glass plate, reflected light


is partially polarized. Polarization of the reflected light is a
maximum if the tangens of incident angle is equal to the refractive
index of the glass plate, or: tan = n.
Draw lines or dots that are related to the determination of the
refractive index on the graph paper. (#)

Question 1.2
Assemble a polariscope to observe birefringence in birefringent
glass plate when light is normally incident on the plastic sheet and
the glass plates.

A birefringent object is the object which splits light into two


components, with the electric field vectors of the two components
perpendicular to each other. The two directions of the electric field
vectors are known as birefringent axes characteristic of birefringent
material. These two components of light travel with different
velocity.

Draw a simple sketch depicting design and functions of the


polariscope assembled.

Insert a sheet of clear cellophane in the path of light in the


polariscope. Draw lines to indicate birefringent axes (#). Comment
briefly but concisely on what is observed, and describe how
berefringent axes are located.

Question 1.3

1.3.a
Stick 10 layers of self-adhesive tape provided on the glass plate as
shown below. Make sure that each layer recedes in equal steps.

fig. 19.10

G square glass plate as a substrate for the cellophane layers


T 10 layers of cellophane sheet
S steps about 3 mm up to 4 mm wide

Insert the assembled square plate into the path of light in the
polariscope. Describe conditions for observing colours. How can
these colours be changed ? Comment on the observations from this
experiment.

1.3.b
Prepare monochromatic red light by placing doubly-folded red
plastic sheet in the path of white light. Mark on the assembled

square plate to show the steps which allow the determination of the
difference of the optical paths of the two components of light from
berefringent phenomenon, described under 1.2 (#).
Estimate the difference of the optical paths from two consecutive
steps.

Question 1.4

1.4.a
With the polariscope assembled, examine the central part of the
drawing triangle provided. Describe relevant optical properties of
the drawing triangle pertaining to berefringence.

1.4.b
Comment on the results observed. Draw conclusions about the
physical properties of the material of which the triangle is made.

Additional Cautions

Be sure that the following items affixed with the coded labels
provided accompany the report.
1. (#) Polarized film with the position of the transmission axis clearly
marked.
2. (#) Graph paper with lines and dots denoting experimental setup
for determining refractive index.
3. (#) Sheet of cellophane paper with marking indicating the positions
of birefringent axis.
4. (#) Square glass plate affixed with self-adhesive tape with
markings to indicate the positions of birefringent axis.

E.P.16.
The following equipment is provided:
1. Two piezoelectric discs of thickness 10 mm with evaporated
electrodes (Fig. 4.1) fixed in holders on the jaws of the calipers;
10 mm

Electrodes

Fig. 4.1

2. The calibrated sine wave oscillator with a photograph of the


control panel, explaining the functions of the switches and
regulators;
3. A double channel oscilloscope with a photograph of the control
panel, explaining the functions of the switches and regulators;
4. Two closed plastic bags containing liquids;
5. A beaker with glycerin (for wetting the discs surfaces to allow
better mechanical coupling);
6. Cables and a three way connector;
7. A stand for support the bags with the liquids;
8. Support and calipers.

A piezoelectric material changes its linear dimensions under the


influence of an electric field and vice-versa, the distortion of a
piezoelectric material induces an electrical field. Therefore, it is
possible to excite the mechanical vibrations in a piezoelectric
material by applying an alternating electric field, and also to induce
an alternating electric field by mechanical vibrations.
A. Knowing that the velocity of longitudinal ultrasonic waves in
3
the material of the disc is about 4 10 m/s, estimate roughly the
resonant frequency of the mechanical vibrations parallel to the disc
axis. Assume that the disc holders do no restrict the vibrations.
(Note that other types of resonant vibrations with lower or higher
frequencies may occur in the discs.)
Using your estimation, determine experimentally the
frequency for which the piezoelectric discs work best as a
transmitter-receiver set for ultrasound in the liquid. Wetting
surfaces of the discs before putting them against the bags improves
penetration of the liquid in the bag by ultrasound.
B. Determine the velocity of ultrasound for both liquids without
opening the bags and estimate the error.
C. Determine the ratio of the ultrasound velocities for both liquids
and its error.
Complete carefully the synopsis sheet. Your report should, apart
from the synopsis sheet, contain the descriptions of:
- method of resonant frequency estimation;
- methods of measurements;
- methods of estimating errors of the measured quantities and of final
results.
Remember to define all the used quantities and to explain the
symbols.

Synopsis Sheet1

In the real Synopsis Sheet the students had more space for filling.

Formula for estimating the resonant


frequency:

Results (with units):

A
Measured best transmitter frequency (with units):

Error:

Definition of measured
quantity:

Error:

S
y
m
b
ol
:

Res
ults:

Final formula for ultrasound velocity in liquid:


B
Velocity of ultrasound (with units):

Error:

Liq
uid
A
Liq
uid
B
Ratio of velocities:

Error:

E.P.17.
Inside a black box provided with three terminals labeled A, B and
C, there are three electric components of different nature. The
components could be any of the following types: batteries, resistors
larger then 100 ohm, capacitors larger than 1 microfarad and
semiconductor diodes.
a)Determine what types of components are inside the black box
and its relative position to terminal A, B and C. Draw the

exploring circuits used in the determination, including those used


to discard circuits with similar behaviour
b)If a battery was present, determine its electromotive force. Draw
the experimental circuit used.
c)If a resistor was present, determine its value. Draw the
experimental circuit used.
d)If a capacitor was present, determine its value. Draw the
experimental circuit used.
e)If a diode was present, determine Vo and Vr, where Vo the forward
bias threshold voltage and Vr is the reverse bias breakdown voltage.
f)Estimate, for each measured value, the error limits.

The following equipments and devices are available for your use:
1 back box with three terminals labeled A, B and C;
1 variable DC power supply;
2 Polytest 1 W multimeters;
10 connection cables;
2 patching boards;
1 100 k, 5 % value resistor;
1 10 k, 5 % value resistor;
1 1 k, 5 % value resistor;
1 100 F, 20 % value capacitor;
1 chronometer;
2 paper sheets;
1 square ruler;
1 interruptor.
Voltmeter internal resistance.
Scale

Value in k

0-1 V

3,2

1%

0-3 V

10

1%

0-10 V

32

1%

0-20 V

64

1%

0-60 V

200

1%

Ammeter internal resistance.


Scale

Value in

0-0,3 mA

1 000 1 %

0-1 mA

263 1 %

0-3 mA

94 1 %

0-20 mA

30,4

1%

0-30 mA

9,84

1%

0-100 mA

3,09

1%

0-300 mA

0,99

1%

0-1 mA

0,31

1%

Notice: Do not use the Polystes 1 W as an ohmmeter. Protect your


circuit against large currents, and do not use currents larger than 20
mA.

Give your results by means of tables or plots.

When drawing the circuits, use the following symbols:

E.P.18.
Determination of light reflectivity of a transparent dielectric
surface.

Experimental Apparatus
1. He-Ne Laser(1.5mW).The light from this laser is not linearly
polarized.
2. Two polarizers (P1, P2) with degree scale disk (Fig. 1), one (P1)
has been mounted in front of the laser output window as a polarizer,
and another one can be fixed in a proper place of the drawing board
by push-pins when it is necessary.
3. Two light intensity detectors (D1, D2) which consisted of a
photocell and a microammeter (Fig. 2).
4. Glass beam splitter(B).
5. Transparent dielectric plate, whose reflectivity and refractive
index are to be determined.
6. Sample table mounted on a semicircular degree scale plate with
a coaxial swivel arm(Fig. 3).
7. Several push-.pins for fixing the sample table on the drawing
board and as its rotation axis.

8. Slit aperture and viewing screen for adjusting the laser beam in
the horizontal direction and for alignment of optical elements.
9. Lute for adhere of optical elements in a fixed place.
10. Wooden drawing board.
11. Plotting papers

Experiment Requirement
1. Determine the reflectivity of the p-component as a function of
the incident angle (the electric field component, parallel to the
plane of incidence is called the p-component).
(a) Specify the transmission axis of the polarizer (A) by the
position of the marked line on the degree scale disk in the pcomponet measurement(the transmission axis is the direction of
vibration of the electric field vector of the transmitted light).
(b) Choose any one of the light intensity detector and set its microammeter at the range of "5". Verify the linear relation ship
between the light intensity and the micro-ammeter reading. Draw
the optical schematic diagram. Show your measured data and
calculated results(including the calculation formula)in the farm of
a table. Plot the linear relationship curve.

(c) Determine the reflectivity of the p-component as a function of


the incident angle. Draw the optical schematic diagram. Show your
measured data and calculated reflectivity(including the calculation
formula)in the form of a table. Plot the reflectivity as a function of
the incident angle.

2. Determine the refractive index of the sample as accurate as


possible.
Explanation and Suggestion
1. Laser radiation avoid direct eye exposure.
2. Since the output power of the laser beam may fluctuate from
time to time, the fluctuation of light output has to be monitored

during the performance of the experiment and a correction of the


experimental results has to be made.
3. The laser should be lighting all the time, even when you finish
your experiment and leave the examination hall, the laser should be
keeping in work.
4. The reflected light is totally plane polarized at an incident angle
B while tg B n (refractive index).

Fig. 1 polarizers with degree scale disk

Fig. 2 Light intensity detector


(1) Insert the plug of photocell into the INPUT socket of
microammeter
(2) Switching on the microammeter.
(3) Blocd off the light entrance hole in front of the photocell and
adjust the scale reading of micro ammeter to 0.
(4) Set the Multiple knob to a proper range.

Fig.3 Sample table mounted on a semicircular degree scale plate


E.P.19.
Given a black box with two similar terminals. There are no more
than three passive elements inside the black box. Find the values of
elements in the equivalent circuit between the terminals. This box
is not allowed to be opened.
Experimental Apparatus
1. Double channel oscilloscope with a panel illustration, showing
the name and function of each knob
2. Audio frequency signal generator with a panel illustration,
showing the name and function of each knob
3. Resistance box with a fixed value of l00 ohm(< 0.5%)
4. Several connecting wires
5. For the coaxial cables, the wire in black color at the terminal is
grounded.
6. Log-log paper, semi-log paper, and millimeter paper are
provided for use if necessary
Note: The knobs, which were not shown on the panel illustration of
the signal generator and oscilloscope, have been set to the
correct positions. It should not be touched by the student.
Experimental Requirements
1. Draw the circuit diagram in your experiment.

2. Show your measured data and the calculated results in the form
of tables. Plot the experimental curves with the obtained results on
the coordinate charts provided(indicate the title of the diagram and
the titles and scale units of the coordinate axes)
3. Given the equivalent circuit of the black box and the names of
the elements with their values in the equivalent circuit(write down
the calculation formulas).
Instructions
1. Do your experiment in the frequency range between 100 Hz and
50kHz.
2. The output voltage of the signal generator should be less than
1.0V (peak-to-peak). Set the Out Attenuation switch to 20 db
position and it should not be changed.
3. On connecting the wires, be careful to manage the wiring so as
to minimize the 50Hz interference from the electric mains.
Instruction for Using XD2 Type Frequency Generator
1. Set the Out Attenuation to 20 db position and it should not
be changed.
2. Set the Damping Switch to Fast position.
3. The indication of the voltmeter of the signal generator is the
relative value, but not the true value of the output.
4. Neglect the error of the frequency readings.
Note: For XD22 Type Audio Frequency generator, there is no
Damping Switch, and the output switch should be set to the
sine ~ position.

Instruction for Using SS-5702 Type Oscilloscope


1. Keep the V mode switch in Dual position.
2. The Volts/div (black) and the variable control (red) vary the
gain of the vertical amplifier, and when the variable control (red)
is ill the fully clockwise position, the black setting are calibrated.

3. The Times/div (Black) varies the horizontal sweep rate from


0.5s/div to 0.2s/div, and they are calibrated when the variable
control (red) is in the fully clockwise CAL position.
4. The Trigging Source (Trigging sweep signal) is used to select
the trigging signal channel and the" level" control is used to adjust
the amplitude of the trigging signal.
5. Measuring accuracy: 4%.
Instruction for Using Resistance Box
The resistance of the Resistance Box has been set to a value of
100ohm, and it should not be changed.
E.P.20.
Warning: Do not look directly into the laser beam or into the laser
beam reflected from the mirror - it may damage your vision.
The bimorph consists of two layers of piezoelectric material
bonded together. Metal electrodes have been evaporated onto the
two outer surfaces to allow the application of an electric field. (See
Fig. 1). The layers are chosen in such a way that when an electric
field is perpendicular to the outer surfaces, one of them expands
(along L), while the other contracts (along L). Reversing the field
reverses the effect on the layers: the one which previously
contracted expands while the other one contracts. Assume that
upon the application of the field the bimorph bends into a circular
arc.

Note: Piezoelectric materials change their dimensions while in an


electric field and produce an electric potential when under
mechanical strain. The relative change of a given dimension in the
electric field is, in first approximation, proportional to the field;
there is however some hysteresis which means that if one applies
the field and then reduces it back to zero the dimensions will not
return to exactly the original values. One has to apply some small
field in the opposite direction to restore the dimensions to the
original value. The force expanding or contracting the piezoelectric
material is, in first approximation, proportional to the field.
INSTRUCTIONS
1. Determine the dependence of the displacement of the bimorph's
free end as a function of the applied voltage in the range from
+36V down to -36V and back up to +36V. During these
measurements, change the voltage only in the indicated direction
(for example, when you measure in the range from -36V to +36V
always increase the voltage and never decrease it; if you miss a
point, skip it). Demonstrate this dependence with a graph.
During one cycle of applied voltage from +36V to -36V and back
to +36V a certain amount of energy is dissipated in the bimorph
itself. Identify and calculate a quantity which is proportional to this
amount of energy.

2. For a given bimorph, if the hysteresis is neglected, the


displacement of the bimorph's free end is given by the formula d =
A Vm l n where V is the applied voltage, l the length of the
bimorph's free end (measured from the edge of the contacts in the
holder) and m, n and A are constants. Find, by performing the
necessary measurements and calculations, the constants m, n and A.
3. Measure the capacitance of the bimorph.
Warning: Do not look directly into the laser beam or into the laser
beam reflected from the mirror - it may damage your vision.
Apparatus:

A bimorph of length L=(38+-1) mm, with a small mirror attached


to one end, clamped in the holder (clothes pin) with attached
contacts and leads. You can change the length of the free part of
the bimorph l by moving it in the holder. Be careful, the bimorph is
delicate!

A laser pointer (with a rubber band around it, which can be used to
keep the switch on during the measurements).

Black modeling clay to fix the bimorph holder and the laser pointer
on the table.

A screen (use the graph paper which you can attach to the partition
around the table).

A multimeter with cables (To measure DC voltage with this


multimeter rotate the switch to the position in which the little circle
points at 200 in the area marked DCV. The input impedance for the
voltage measurements is equal to 1 M, accuracy of the
measurement is 0.1V).

A variable resistor (potentiometer) 2.5M (to control the bimorph's


voltage) with three leads. The red lead is connected to the central
(moving) electrode of the potentiometer.

A pack of four 9V batteries with leads (Note: The resistance of the


battery pack was increased by adding a 5k resistor in series with
one of the leads. This is to limit the current and protect the circuit.
DO NOT short or bypass this resistor).

A (1.00+-0.05) G resistor (note: the resistance of this resistor can


be affected by the residue from your skin; do not touch the body of
the resistor, only the metal wires).

A stop watch.

A ruler.

Masking tape.
E.P.21.
The aim is to produce a graph showing how the conductance* of a
light-dependent resistor (LDR) varies with wavelength across the
visible spectrum.
*conductance G = 1/resistance (units: siemens, 1 S = 1 W -1)
There are five parts to this experiment:

Using a concave reflection grating (made from a strip of CDROM)


to produce a focused first order spectrum of the light from bulb A
(12 V 50W tungsten filament).

Measuring and plotting the conductance of the LDR against


wavelength as it is scanned through this first order spectrum.

Showing that the filament in bulb A behaves approximately as an


ideal black body.

Finding the temperature of the filament in bulb A when it is


connected to the 12 V supply.

Correcting the graph of conductance against wavelength to take


account of the energy distribution within the spectrum of light
emitted by bulb A.
Precautions

Beware of hot surfaces.

Bulb B should not be connected to any potential difference


greater than 2.0 V.

Do not use the multimeter on its resistance settings in any live


circuit.
Procedure

(a) The apparatus shown in Figure 1 has been set up so that light
from bulb A falls normally on the curved grating and the LDR has
been positioned in the focused first order spectrum. Move the
LDR through this first-order spectrum and observe how its
resistance (measured by multimeter X) changes with position.
(b) (i) Measure and record the resistance R of the LDR at different
positions within this first-order spectrum. Record your data in the
blank table provided.
(ii) Plot a graph of the conductance G of the LDR against
wavelength l using the graph paper provided.
Note The angle q between the direction of light of wavelength l in
the first-order spectrum and that of the white light reflected from
the grating (see Figure 1) is given by:
sin q = l /d where d is the separation of lines in the grating.
The grating has 620 lines per mm.
The graph plotted in (b)(ii) does not represent the sensitivity of the
LDR to different wavelengths correctly as the emission
characteristics of bulb A have not been taken into account. These
characteristics are investigated in parts (c) and (d) leading to a
corrected curve plotted in part (e).

Note for part (c) that three multimeters are connected as


ammeters. These should NOT be adjusted or moved. Use the
fourth multimeter (labelled X) for all voltage measurements.
(c) If the filament of a 50 W bulb acts as a black-body radiator it
can be shown that the potential difference V across it should be
related to the current I through it by the expression:
V3 = CI 5 where C is a constant.
Measure corresponding values of V and I for bulb A (in the
can). The ammeter is already connected and should not be adjusted.
(i) Record your data and any calculated values in the table provided
on the answer sheet.
(ii) Plot a suitable graph to show that the filament acts as a blackbody radiator on the graph paper provided.

(d) To correct the graph in (b)(ii) we need to know the working


temperature of the tungsten filament in bulb A. This can be found
from the variation of filament resistance with temperature.

You are provided with a graph of tungsten resistivity (m W cm)


against temperature (K).
If the resistance of the filament in bulb A can be found at a known
temperature then its temperature when run from the 12 V supply
can be found from its resistance at that operating potential
difference. Unfortunately its resistance at room temperature is too
small to be measured accurately with this apparatus. However, you
are provided with a second smaller bulb, C, which has a
larger, measurableresistance at room temperature. Bulb C can be
used as an intermediary by following the procedure described
below. You are also provided with a second 12V 50W bulb (B)
identical to bulb A. Bulbs B and C are mounted on the board
provided and connected as shown in Figure 2.
(i) Measure the resistance of bulb C when it is unlit at room
temperature (use multimeter X, and take room temperature to be
300 K). Record this resistance RC1 on the answer sheet.
i.

Use the circuit shown in Figure 2 to compare the filaments


of bulbs B and C. Use the variable resistor to vary the
current through bulb C until you can see that overlapping
filaments are at the same temperature. If the small filament is
cooler than the larger one it appears as a thin black loop.
Measure the resistances of bulbs B and C when this
condition has been reached and record their
values, RC2 and RB, on the answer sheet. Remember, the
ammeters are already connected.

(iii) Use the graph of resistivity against temperature (supplied) to


work out the temperature of the filaments of B and C when they are
matched. Record this temperature, T2V, on the answer sheet.
(iv) Measure the resistance of the filament in bulb A (in the can)
when it is connected to the 12 V a.c. supply. Once again the
ammeter is already connected and should not be adjusted. Record
this value, R12V on the answer sheet.
(v) Use the values for the resistance of bulb A at 2 V and 12 V and
its temperature at 2 V to work out its temperature when run from
the 12 V supply. Record this temperature, T12V in the table on the
answer sheet.

You are provided with graphs that give the relative intensity of
radiation from a black-body radiator (Planck curves) at 2000
K, 2250 K, 2500 K, 2750 K, 3000 K and 3250 K.
(e) Use these graphs and the result from (d)(v) to plot a corrected
graph of LDR conductance (arbitrary units) versus wavelength
using the graph paper provided. Assume that the conductance of
the LDR at any wavelength is directly proportional to the intensity
of radiation at that wavelength (This assumption is reasonable at
the low intensities falling on the LDR in this experiment). Assume
also that the grating diffracts light equally to all parts of the first
order spectrum.
.
Figure 1 - Experimental arrangement for (a)

Figure 1: Detail - the grating:

Figure 1: Detail - LDR and Multimeter:

Figure 2

Note that this diagram does not show meters


Graph 1

Graph 2a

Graph 2b

E.P. 22.
Aim
To investigate the forces on a puck when it slides down the
slope.
Warning
Do not touch the circular flat faces of the puck or the paper surface
of the slope with your hands. Use the glove provided. The faces
have different coloured paper stickers for convenience but the
frictional characteristics of the paper faces may be assumed to be
the same.
Timing
The sensors underneath the track trigger electronic gates in the box
and the green LED will light when the puck is between the
sensors. The multimeter measures the potential difference across
a capacitor, which is connected to a constant-current source (whose
current is proportional to the voltage of the battery) whilst the
green light is on. The reading of the multimeter is therefore a
measure of the time during which the puck is between the sensors.
This reading can give a value for the speed of the puck in arbitrary
units.

Operating the timer


i)
Press and hold down the black push button on the
side of the box. This switches the electronics on.
ii)
If the green light goes on slide the puck (light face up)
past the lower sensor. The green light should go off.
iii)
The potential difference across the capacitor can be
reduced to zero before the puck is released by pressing the red
button for at least 10s.
iv)
The battery potential difference can be measured by
connecting the multimeter across the terminals marked with the
cell symbol.

Definitions
(i)
A moving body sliding down an inclined plane
experiences a tangential retarding force F and a normal
reaction N. Define

(ii)
When the retarding force is due to friction
alone, x equals ms and is called the dynamic coefficient of friction
for the surface. It is independent of speed.
(iii)
define

When the blue (dark) side is in contact with the plane

where the tangential force Fd is partly due to the surface friction


and partly due to magnetic effects.
(iv)
The variable xds which gives the magnetic effects only
is defined by

Important hints and advice


(i)
You will find it helpful initially to investigate the
behaviour of the puck qualitatively.

(ii)
Think about the physics before you do a quantitative
investigation. Remember to use graphical presentation where
possible.
(iii)
Do not attempt to take too many experimental
readings unless you have plenty of time.
(iv)
You are measuring the potential difference across an
electrolytic capacitor. This does not behave quite like a simple air
capacitor. Slow leakage of charge is normal and the potential
difference will not remain completely steady.
(v)
You are given one puck and one 9.0 V
battery. Conserve the battery! The constant current filling the
capacitor is proportional to the battery potential difference. It is
therefore advisable to monitor the battery potential difference. In
addition, the sensors may not be reliable if the potential difference
of the battery falls below 8.4 V. If this happens, ask for another
battery.
(vi)
Your answer pack contains 4 sides of graph paper
only. You will not be given further sheets. You may keep the
puck at the end of your experiment.
(vii)
If you have trouble operating the multimeters ask an
invigilator.
Data

Weight of puck = 5.8410-2 N

The voltmeter reading indicates the time of travel of the


puck. When the potential difference of the battery is 9.0 V then 1V
corresponds to 0.213 s

Distance between sensors = 0.294 m

Experiment
Using only the apparatus provided investigate how xds depends on
the speed vq of the puck for track inclinations q to the horizontal.
State on the answer sheet the algebraic equations/relations used in
analysing your results and in plotting your graphs.
Suggest a quantitative model to explain your results. Use the data
which you collect to justify your model.

Experimental Question 2
Diffraction and Scattering of Laser Light
The aim of this experiment is to demonstrate and quantify to some extent the reflection, diffraction, and
scattering of light, using visible radiation from a Laser Diode source. A metal ruler is employed as a
diffraction grating, and a perspex tank, containing water and diluted milk, is used to determine reflection
and scattering phenomena.

Section 1 (6 marks)
Place the 150 mm length metal ruler provided so that it is nearly normal to the incident laser beam, and
so that the laserr beam illuminates several rulings on it. Observe a number of spots of light on the
white paper screen provided, caused by the phenomenon of diffraction.
Draw the overall geometry you have employed and measure the position and separation of these spots
with the screen at a distance of approximately 1.5 metres from the ruler.
Using the relation
N = h sin
where

is
is
is
is

the
the
the
the

order of diffraction
radiation wavelength
grating period
angle of diffraction

and the information obtained from your measurements, determine the wavelength of the laser radiation.

Section 2 (4 marks)
Now insert the empty perspex tank provided into the space between the laser and the white paper screen.
Set the tank at approximately normal incidence to the laser beam.
(i) Observe a reduction in the emergent beam intensity, and estimate the percentage value of this reduction. Some calibrated transmission discs are provided to assist with this estimation. Remember
that the human eye has a logarithmic response.
This intensity reduction is caused primarily by reflection losses at the aid/perspex boundaries, of
which there are four in this case. THe reflection coefficient for normal incidence at each boundary, R,
which is the ratio of the reflectied to incident intensities, is given by
R = {(n1 n2 )/(n1 + n2 )}2
where n1 and n2 are the refractive indices before and after the boundary. The corresponding transmission
coefficient, assuming zero absorption in the perspex, is fiven by
T =1R .
(ii) Assuming a refractive index of 1.59 for the perspex and neglecting the effect of multiple reflections
and cogerence, calculate the intensity transmission coefficient of the empty perspex tank. Compare
this result with the estimate you made in Part (i) of this Section.

Section 3 (1 mark)
Without moving the perspex tank, repeat the observations and calculations in Section 2 with the 50 mL
of water provided in a beaker now added to the tank. Assume the refractive index of water to be 1.33.

Section 4 (10 marks)

(i) Add 0.5 mL (12 drops) of milk (the scattering material) to the 50 mL of water in the perspex tank,
and stir well. Measure as accurately as possible the total angle through which the laser light is
scattered, and the diameter of the emerging light patch at the exit face of the tank, noting that these
quantities are related. Also estimate the reduction in transmitted intensity, as in earlier sections.
(ii) Add a further 0.5 mL of milk to the tank, and repeat the measurements requested in part (i).
(iii) Repeat the process in part (ii) until very little or no transmitted laser light can be observed.
(iv) Determine the relationship between scattering angle and milk concentration in the tank.
(v) Use your results, and the relationship
I = I0 ez = Tmilk I0
where

I0
I
z

Tmilk

is
is
is
is
is

the
the
the
the
the

input intensity
emerging intensity
distance in the tank
attenuation coefficient and equals a constant times the concentration of the scatterer
transmission coefficient for the milk

to obtain an estimate for the value of for a scatterer concentration of 10%.

S-ar putea să vă placă și